◆ わからない問題はここに書いてね 31 ◆

このエントリーをはてなブックマークに追加
1( ○ ´ ー ` ○ )
   / ̄   ̄ ヽ
  / ,,w━━━.、)   / ̄ ̄ ̄ ̄ ̄ ̄ ̄ ̄ ̄ ̄ ̄ ̄ ̄ ̄ ̄ ̄ ̄
  ! .fw/f_」」_|_|_i_)  | ここは分からない問題について質問するさくらちゃんスレですわ
  ヽ|:::(6||f;j' ,fj'||)  | スレッドや業務連絡,記号の書き方例は >>2-13 辺りに。
 ∠|::i:!::|:|、_ワノ:i、 <  ローマ数字や丸付き数字などの機種依存文字はお勧め出来ませんわ
  .|::|< |::|ヽーノ`l:i;ヽ  _________________
  .ノ:ノ' i:::l `只´|:|i)::)
 (::(:i  |:::|ノ ) j:j|:(

    (⌒, -- 、⌒)     / ̄ ̄ ̄ ̄ ̄ ̄ ̄ ̄ ̄ ̄ ̄ ̄ ̄ ̄ ̄ ̄
  _  Y      Y  _ < 自分でどこまで考えたのか、途中でもいいから
 ミ  | ・  . ・| / 彡 | 書いてくれればこっちも答えやすくて助かるわー
    @ゝ.  ^  ノ@    | 質問者も解答者もくれぐれもトラブルは起こさんといてなー
                 ________________

【前のスレッド】
◆ わからない問題はここに書いてね 30 ◆
http://science.2ch.net/test/read.cgi/math/1020310032/
2( ○ ´ ー ` ○ ):02/05/10 12:31
【過去のスレッド】
◆ わからない問題はここに書いてね1〜30 ◆
01 http://cheese.2ch.net/math/kako/967/967755172.html
02 http://cheese.2ch.net/math/kako/970/970795775.html
03 http://cheese.2ch.net/math/kako/974/974911042.html
04 http://cheese.2ch.net/math/kako/978/978209589.html
05 http://cheese.2ch.net/math/kako/981/981372834.html
06 http://cheese.2ch.net/math/kako/985/985594205.html
07 http://cheese.2ch.net/math/kako/988/988952592.html
08 http://cheese.2ch.net/math/kako/991/991223596.html
09 http://cheese.2ch.net/math/kako/993/993571403.html
10 http://cheese.2ch.net/math/kako/995/995448453.html
11 http://cheese.2ch.net/math/kako/997/997329928.html
12 http://cheese.2ch.net/math/kako/999/999689496.html
13 http://cheese.2ch.net/math/kako/1001/10013/1001342715.html
14 http://cheese.2ch.net/math/kako/1002/10028/1002893257.html
15 http://cheese.2ch.net/math/kako/1004/10041/1004171159.html
16 http://cheese.2ch.net/math/kako/1005/10057/1005735838.html
17 http://cheese.2ch.net/math/kako/1006/10068/1006859798.html
18 http://cheese.2ch.net/math/kako/1007/10078/1007834117.html
19 http://cheese.2ch.net/math/kako/1009/10091/1009102965.html
20 http://cheese.2ch.net/math/kako/1010/10107/1010708150.html
21 http://cheese.2ch.net/math/kako/1011/10116/1011689052.html
22 http://cheese.2ch.net/math/kako/1012/10125/1012535858.html
23 http://cheese.2ch.net/math/kako/1014/10146/1014673280.html
24 http://cheese.2ch.net/math/kako/1014/10146/1014673280.html
25 http://cheese.2ch.net/math/kako/1015/10158/1015866030.html
26 http://cheese.2ch.net/math/kako/1016/10165/1016541847.html
27 http://cheese.2ch.net/math/kako/1017/10175/1017511624.html
28 http://natto.2ch.net/math/kako/1018/10183/1018304190.html
29 http://natto.2ch.net/math/kako/1019/10193/1019394107.html
30 http://science.2ch.net/test/read.cgi/math/1020310032/
3( ○ ´ ー ` ○ ):02/05/10 12:32
【関連スレッド】
雑談はここに書け!【3】
http://natto.2ch.net/test/read.cgi/math/1010679340/
くだらねぇ問題はここへ書け ver.3.141592653589
http://science.2ch.net/test/read.cgi/math/1020559784/
4( ○ ´ ー ` ○ ):02/05/10 12:33
【掲示板での数学記号の書き方例】
■数の表記
●スカラー:a,b,c,...,z, A,B,C,...,Z, α,β,γ,...,ω, Α,Β,Γ,...,Ω, ... (← ギリシャ文字はその読み方で変換
可.)
●ベクトル:V=[V[1],V[2],...], |V>, V↑, vector(V) (← 混同しない場合はスカラーと同じ記号でいい.通
常は縦ベクトルとして扱う.)
●テンソル(上下付き1成分表示):T^[i,j,k...]_[p,q,r,...], T[i,j,k,...;p,q,r,...]
●行列(1成分表示):M[i,j], I[i,j]=δ_[i,j]
●行列(全成分表示):M=[[M[1,1],M[2,1],...],[M[1,2],M[2,2],...],...], I=[[1,0,0,...]',[0,1,0,...],...] (← 行(また
は列ごと)に表示する.)

■演算・符号の表記
●足し算:a+b
●引き算:a-b
●掛け算:a*b, ab (← 通常"*"を使い,"x"は使わない.)
●割り算・分数:a/b, a/(b+c), a/(bc) (← 通常"/"を使い,"÷"は使わない.)
●割り算分数2:(a+b)/(c+d),a+(b/c),(a/b)+c(←括弧を用い分子分母を他の項と区別できるように表
現する。)
●複号:a±b=a士b, a干b (← "±"は「きごう」で変換可.他に漢字の"士""干"なども利用できる.)
●内積・外積・3重積:a・b, axb, a・(bxc)=(axb)・c=det([a,b,c]), ax(bxc)

■関数・数列の表記
●関数:f(x), f[x]
●数列:a(n), a[n], a_n
●平方根:√(a+b)=(a+b)^(1/2) (← "√"は「るーと」で変換可.)
●指数・指数関数:a^b, x^(n+1), exp(x+y)=e^(x+y) (← "^"を使う."exp"はeの指数.)
●対数・対数関数:log_{a}(b), log(x/2)=log_{10}(x/2), ln(x/2)=log_{e}(x/2) (← 底を省略する場合,
"log"は常用対数,"ln"は自然対数.)
●三角比・三角関数:sin(a), cos(x+y), tan(x/2)
●行列式・トレース:|A|=det(A), tr(A)
●絶対値:|x|
●ガウス記号:[x] (← 関数の変数表示などと混同しないように注意.)
●共役複素数:z~
●転置行列・随伴行列:M', M† (← "†"は「きごう」で変換可.)
●階乗:n!=n*(n-1)*(n-2)*...*2*1, n!!=n*(n-2)*(n-4)*...
●順列・組合せ:P[n,k]=nPk, C[n.k]=nCk, Π[n,k]=nΠk, H[n,k]=nHk (← "Π"は「ぱい」で変換可.)

■微積分・極限の表記
●微分・偏微分:dy/dx=y', ∂y/∂x=y,x (← "∂"は「きごう」で変換可.)
●ベクトル微分:∇f=grad(f), ∇・A=div(A),∇xA=rot(A), (∇^2)f=Δf (← "∇"は「きごう」,"Δ"は
「でるた」で変換可.)
●積分:∫[0,1]f(x)dx=F(x)|_[x=0,1], ∫[y=0,x]f(x,y)dy, ∬_[D]f(x,y)dxdy, ?_[C]f(r)dl (← "∫"は「い
んてぐらる」,"∬?"は「きごう」で変換可.)
●数列和・数列積:Σ_[k=1,n]a(k), Π_[k=1,n]a(k) (← "Σ"は「しぐま」,"Π"は「ぱい」で変換可.)
●極限:lim_[x→∞]f(x) (← "∞"は「むげんだい」で変換可.)

■その他
●図形:"△"は「さんかく」,"∠"は「かく」,"⊥"は「すいちょく」,"≡"は「ごうどう」,"∽"は「きごう」で変
換可.
●論理・集合:"⇔⇒∀∃∧∨¬∈∋⊆⊇⊂⊃∪∩"は「きごう」で変換可.
●等号・不等号:"≠≒≦≧≪≫"は「きごう」で変換可.

※ ここで挙げた表記法は1例であり,標準的な表記法からそうでないものまで含まれているので,後者の
場合使う時にあらかじめことわっておいたほうがいい.
※ 関数等の変数表示や式の括弧は,括弧()だけでなく[]{}を適当に組み合わせると見やすい場合があ
る.
※ 上記のほとんどの数学記号や上記以外の数学記号は大体「きごう」で順次変換できる.
5( ○ ´ ー ` ○ ):02/05/10 12:33
【一般的な記号の使用例】
a:係数、数列 b:係数、重心
c:定数、積分定数 d:微分、次数、次元、距離、外微分、外積、公差
e:自然対数の底、単位元、分岐指数、基底、離心率 f:関数、多項式、基底
g:関数、多項式、群の元、種数、計量、重心 h:高さ、関数、多項式、群の元、類数、微小量
i:添え字、虚数単位、埋めこみ、内部積 j:添え字、埋めこみ、j-不変量、四元数体の基底
k:添え字、四元数体の基底、比例係数 l:添え字、直線、素数
m:添え字、次元、Lebesgue測度 n:添え字、次元、自然数
o:原点 p:素数、射影
q:素数、exp(2πiτ) r:半径、公比
s:パラメタ、弧長パラメタ t:パラメタ
u:ベクトル v:ベクトル
w:回転数 x:変数
y:変数 z:変数(特に複素数変数)

A:行列、環、加群、affine空間、面積
B:行列、開球、Borel集合、二項分布
C:複素数体、連続関数全体の集合、組み合わせ、曲線、積分定数、Cantorの3進集合、チェイン複

D:関数の定義域、微分作用素、判別式、閉球、領域、二面体群、Diniのderivative、全行列環
E:単位行列、楕円曲線、ベクトル束、単数群、辺の数
F:原始関数、体、写像、ホモトピー、面の数
G:群、位相群、Lie群
H:Hilbert空間、Hermite多項式、部分群、homology群、四元数体、上半平面、Sobolev空間、重複
組み合わせ
I:区間、単位行列、イデアル
J:Bessel関数、ヤコビアン、イデアル、Jacobson根基
K:体、K群、多項式環、単体複体、Gauss曲率
L:体、下三角行列、Laguerre多項式、L関数、Lipschitz連続関数全体の集合、関数空間L^p、線
型和全体
M:体、加群、全行列環、多様体
N:自然数全体の集合、ノルム、正規部分群、多様体
O:原点、開集合、整数環、直交群、軌道、エルミート演算子
P:条件、素イデアル、Legendre多項式、順列、1点、射影空間、確率測度
Q:有理数体、二次形式
R:半径、実数体、環、可換環、単数規準、曲率テンソル、Ricciテンソル
S: 級数の和、球面、部分環、特異チェイン複体、対称群、面積、共分散行列
T:トーラス、トレース、線形変換
U:上三角行列、unitary行列、unitary群、開集合、単数群
V:ベクトル空間、頂点の数、体積
W:Sobolev空間、線形部分空間
X:集合、位相空間、胞複体、CW複体、確率変数、ベクトル場
Y:集合、位相空間、ベクトル場、球面調和関数 Z:有理整数環、中心
6( ○ ´ ー ` ○ ):02/05/10 12:34
【一般的な記号の使用例】
α:定数、方程式の解 β:定数、方程式の解
γ:定数、Euler定数、曲線 δ:微小量、Diracのdelta関数、Kroneckerのdelta
ε:任意の正数、実二次体の基本単数、Levi-Civitaの記号
ζ:変数、zeta関数、1の冪根
η:変数 θ:角度
ι:埋めこみ κ:曲率
λ:定数、測度、固有値、Z_p拡大の不変量、モジュラー関数
μ:定数、測度、Z_p拡大の不変量、Mobiusの関数
ν:測度、付値、Z_p拡大の不変量
ξ:変数 ο:Landauの記号
π:円周率、射影、素元、基本群
ρ:rank、相関係数
σ:標準偏差、置換、σ関数、単体、σ代数
τ:置換、群の元、捩率 υ:
φ:空集合、写像、Eulerの関数
χ:Euler標数、特性関数、階段関数  ψ:写像
ω:character、1の3乗根、微分形式

Β:beta関数  Γ:gamma関数、SL(2、R)の離散部分群、Christoffelの記号
Δ:微小変化、対角線集合、対角線写像、weight12のcusp form、単位円板、ラプラシアン、行列式
Λ:作用域、添え字集合、対角行列 Π:積記号
Σ:和記号、素体、(共)分散行列 Ο:Landauの記号
Φ:写像 Ψ:写像
Ω:代数的平方、拡大体、領域
7( ○ ´ ー ` ○ ):02/05/10 12:35
【業務連絡】
■900を超えたら新スレに移行準備.
■旧スレ側 → 終了宣言,新スレへの誘導.
■新スレ側 → 開始宣言と目次,旧スレのリンク,掲示板での数学記号の書き方例,
  業務連絡・その他,旧スレ側の残り問題の移動.
■数学板の要望スレで数学板の注意書き(リンク先)の変更依頼.
■単独の質問スレは,このスレか「くだらんスレ」に誘導して下さい.
■誤って過去スレに新たに書き込まれた質問は,最新スレに誘導して下さい.
【数学板削除依頼スレ】
http://kaba.2ch.net/test/read.cgi/saku/986384122/ (レス削除)
http://kaba.2ch.net/test/read.cgi/saku/987829968/ (スレッド削除)
【ローカルルール等リンク先更新総合スレッド2】
http://kaba.2ch.net/test/read.cgi/accuse/1012720188/l50
★__________________________.
|              │
│ はにゃ?ん     |
| γ∞γ~       |
│人w/ 从从) )   │
│ ヽ | |┬ イ |〃  │
│ `wハ~ . ノ)    │
│  /  `「 .     │
| 数学板さくらスレ  |
|_________________________│

〃二二ヽ
| |77777〉
| | ゚д゚ノ|  サクラチャンノハタケイヨウデスワ
|⊂   つ
8( ○ ´ ー ` ○ ):02/05/10 12:36
━━━━━━━━━━━━━━━━━━━━━━━━━━━━━━━━━━━━━

                     移転完了しましたわ♪
              ◆ わからない問題はここに書いてね 31 ◆
         いよいよ始まりますわ それではみなさま心置きなくどうぞ

━━━━━━━━━━━━━━━━━━━━━━━━━━━━━━━━━━━━━
9( ○ ´ ー ` ○ ):02/05/10 12:39
雑談スレの場所まちがったべ( ● ´ ー ` ● )
正しくはここだべ( ● ´ ー ` ● )
http://science.2ch.net/test/read.cgi/math/1010679340/
前のスレッドで明確な答をだしてもらっていないので一応。

exp((|x|^2-ε^2)^(-1))/(|x|^2-ε^2)が|x|→ε-0で0に収束する事の証明を教えて下さい。
ロピタルの定理は使えませんでした。
迷惑だったら申し訳ないです、これっきりにしますので。
11132人目の素数さん:02/05/10 13:05
>>10
合成関数の微分そのもののように見えるのは気のせいなの?
>>10
y=-(|x|^2-ε^2)^(-1) と置けば、

lim[y→∞]-y/e^y

の形だからなんとでもなるんでないの?
13132人目の素数さん:02/05/10 13:20
>>11 気のせいだよ
>>10 t=1/(|x|^2-ε^2) とおくと(以下略)
少しは自分で考えなよ
1413:02/05/10 13:21
毛コーンしちゃたよ
15132人目の素数さん:02/05/10 14:01
素因数分解の一意性について(ある数を素因数分解した時、その組み合わせ(2×3など)は一通りしかない、とかだったと思います)、
証明して下さる方、いらっしゃったら教えてください。
何か、素数の性質(pがnmを割り切るとき、pはnかmを割り切る)を使って 証明しないといけないらしいんですが・・・
お願いします…
16132人目の素数さん:02/05/10 14:24
ベクトル空間で有限基底が取れるならば無限基底は存在しないということの
エレガントな証明ってありますか?
>>12
yを-(|x|^2-ε^2)^(-1)に置くのがコツだったんですね。
全く気がつきませんでした。
本当にありがとうございます。

>>113
すいません、以後気をつけます。
ただ、t=1/(|x|^2-ε^2)だけだと上手くいかないですよ。
ていうか、>>1 がオタクくさいというのはなんとかならないの?
>>18

な ら な い 。
20132人目の素数さん:02/05/10 16:11
任意の正方形のfとfの合成変換による像がもとの正方形であり、
(1、1)のfによる像が(2、0)であるとき、行列をもとめよ。

この問題がわかりません。
よろしくお願いします。
つーか>>1の名前をまずなんとかしろ
22小1:02/05/10 16:31
なんで1+1=2になるのですか?何が2になるの?
23132人目の素数さん:02/05/10 16:35
f(x)=(3x^2+5)/(3x^2+20)でx∈Rの時f(x)の取りうる値の範囲を求めよ
って問題なんですけどすっきり解く方法はありませんか?
24132人目の素数さん:02/05/10 16:38
ごめんなさい↑のやつf(x)=(3x^2-5)/(3x^2-20)でした
25132人目の素数さん:02/05/10 16:56
関数 y=2^x+2-4^xの最大値、最小値と、それらに対するxの値を求めよ。
ただし、x<=3とする
この問題がわかりません。
よろしければ詳しい計算もお願いします。
26132人目の素数さん:02/05/10 16:56
>24
f(x)=1+15/(3x^2-20)と変形するのはどうですか?
>>25
とりあえず 2^x=t とでもおいてみそ。
>>25
2^x = t とか置けば?
遅かったほうの俺が死んどく。
>>27,28
すいませんそれでも分かりません
4^x = (2^x)^2 でしょ?
32132人目の素数さん:02/05/10 17:08
t=2^xとおくと、y=t+2-t^2とおいて、あとはtについての2次式になるから、
2次関数の最大・最小問題となるけど、これでどうかな?
3332:02/05/10 17:10
訂正:   誤  y=t+2-t^2とおいて 
      正  y=t+2-t^2となるので
34132人目の素数さん:02/05/10 17:11
>26
そのあとの15/(3x^2-20)の範囲はやっぱり微分を使うしかないですか?
35132人目の素数さん:02/05/10 17:12
すいませんy=2^(x+2)-4^xでした
36132人目の素数さん:02/05/10 17:14
>34
微分は使わなくても解けますよ。
3x^2-20の最小値が分かれば、15/(3x^2-20)の範囲が決まりますから。
37132人目の素数さん:02/05/10 17:17
>35
2^(x+2)=2^x・2^2=4・2^x
>>35 ようするに君は答を全部書いてくれないとわからないということか?
3920:02/05/10 17:20
こっちもお願いします
40132人目の素数さん:02/05/10 17:21
>38
すいません、習ったばかりで全部かいてくれないと分かりません
よろしくお願いします。
41132人目の素数さん:02/05/10 17:25
任意の正方形のfとfの合成変換による像がもとの正方形であり、
(1、1)のfによる像が(2、0)であるとき、行列をもとめよ。

この問題がわかりません。
よろしくお願いします。
4215:02/05/10 17:34
こっちもお願いします・・・
43132人目の素数さん:02/05/10 17:40
>>15
任意の整数pが
p=an=am(nとmは互いに素)
と2通りに素因数分解できると仮定すると、、、
これでだめかな?
44132人目の素数さん:02/05/10 17:46
>43
それって、途中で最大公約数(AとBの最大公約数が1なら、sA+tB=1)を使う証明でしょうか?

僕は上のやり方でやって、教師にそれを見せたら、
素数の性質(pがnmを割り切るとき、pはnかmを割り切る)を使わないと駄目だ、とか言われたんです・・・
45132人目の素数さん:02/05/10 17:56
>44
論理的に合っていれば、どんな方法で解いてもよいはずです。
その教師がなぜだめだと言っているのかを確認してみましょう。
20は、マルチだ
>>44
43のやり方でも君が言っている素数の性質を使う事になると思うんだが。。。
48132人目の素数さん:02/05/10 18:21
>45
素数の性質を使ってみる、とかいうテーマ(?)だから・・・だそうです。
49132人目の素数さん:02/05/10 18:22
>>46
前スレにあるのは見たけど、丸痴なの?
問題の雰囲気は大数の宿題っぽいとおもった(w

>>20=39=41
ffが恒等写像だって事が証明出来ればオシマイ。
5042:02/05/10 18:33
どうでもいい訂正:44で、「言われました」というのは、
僕の前に教師にレポートを見せた人が、同じような(少ししか見れませんでしたが)証明をしてたので、
その人が言われたんです。
・・・どうでもいいですね。


>45
そうなんですか?

僕のやったのは、
n=a(1)a(2)・・・a(x)=b(1)b(2)・・・b(y)とおいて、
(()内の数字が多きいほどaやbは大きい)

a≠b(a<b)とすると互いに素、最大公約数は1。
なのでsa+tb=1
両辺にb(2)b(3)・・・b(y)をかけて、b(1)・・・b(y)=nなので
sa(1)b(2)b(3)・・・b(y)+tn=b(2)・・・b(y)となり、
両辺a(1)で割る事を考えて、左辺第一項は当然割り切れて、
第2項はtn=ta(1)・・・a(x)なのでこれも割り切れる。
なので右辺も割り切れるはず。
しかし、a<b、b(p)<b(p+1)なので右辺にa(1)は含まれて居ない
矛盾。よってa(1)=b(1)
以下(a(q)=b(q))同様。

・・・としたんですが・・・。
51132人目の素数さん:02/05/10 18:35
結論を書くの忘れてました。


以上より、nの素因数分解は一通りしか存在しない。(一意的である)
>しかし、a<b、b(p)<b(p+1)なので右辺にa(1)は含まれて居ない
>矛盾。

ここがよくわからない。間違っていると思う。
53132人目の素数さん:02/05/10 18:42
>25,40
t=2^xとおく。
y=2^(x+2)-4^x=2^x・2^2-(2^2)^x=4t-(2^x)^2
=4t-t^2=4-(t-2)^2
f(t)=4-(t-2)^2とおく。
x<=3より、0<t<=8。よって、f(t)はt=2のとき最大値4をとり、
t=8のとき、最小値-32をとる。
よって、yは、x=1のとき最大値4をとり、x=3のとき、最小値-32をとる。
>>50
n=a(1)a(2)・・・a(x)=b(1)b(2)・・・b(y)
の部分で両辺がa(1)で割り切れるんだから右辺の
b(i){i=1...y}のどれかはa(1)で割り切れる事を使えば良いんじゃないかな?
これは君が言ってた素数の性質だよね。
55132人目の素数さん:02/05/10 18:47
>36
なるほど、どうも有り難うございいます.

56132人目の素数さん:02/05/10 19:31
>52
訂正:a<b→a(1)<b(1)でした。
証明はやはりもっと丁寧に書くべきなんでしょうか・・・(←自分の説明が下手なだけ)

で、r>1とするとa(1)<b(1)<b(r)だから、
絶対に右辺にb(r)=a(1)となるようなrは存在しない、とかそういうつもりでした。

右辺にa(1)・・・・・・あ゙。

ごめんなさい、「a<b、b(p)<b(p+1)」に加えてもう1つの条件、
「aとbは互いに素」というのを忘れていました・・・。

で、右辺にa(1)と同じ値のものが存在していないということは、
右辺はa(1)では割り切れない。
(左辺は割り切れる為)これは矛盾。

・・・こんな感じで正しいんでしょうか?
57132人目の素数さん:02/05/10 19:34
>54
n=a(1)a(2)・・・a(x)=b(1)b(2)・・・b(y)
の両辺、a(1)で割り切れるんですか?
ごめんなさい、何か良く分からないです・・・。
58132人目の素数さん:02/05/10 19:37
階乗x!の連続関数ってありますか?
59132人目の素数さん:02/05/10 19:50
>>58
Γ関数。一部不連続だけどね・・・・
http://mathworld.wolfram.com/GammaFunction.html
>>56
素数には、次の性質があるよね。
[性質A] N_1*N_2*N_3*…*N_k が素数 P で割り切れるなら、
N_1〜N_k のうち少なくともひとつは P で割り切れる。

これを既知にするなら、50 の初めの所で、sa+tb=1 について
やっていることは回り道だよ。54 さんの様にやればいい。

もし、[性質A] を既知とはしない、というなら、

>右辺にa(1)と同じ値のものが存在していないということは、
>右辺はa(1)では割り切れない

というのは論理になっていない気がする。
素因数分解の一意性を仮定してしまっているかんじで、
循環論法になってるんじゃないのかな?

>>57
a(1)a(2)・・・a(x) は a(1) で割り切れるし、等号でつながっているんだから
b(1)b(2)・・・b(y) も a(1) で割り切れるよ。
61132人目の素数さん:02/05/10 20:10

>60
あ、なるほど・・・。
ありがとうございます。

ちょっとそれでやってみます。
また分からなくなったら(なりそうですが(汗))質問させていただきます。
6216:02/05/10 20:19
どなたかおわかりのかたはおりませんか?
そんなに自明じゃないのかもですね。
63132人目の素数さん:02/05/10 20:22
現金値引き10%、ポイント10%のお店があります。
定価1000円の商品の場合、100円値引きして900円になったあと
90ポイントがつく計算です。次回1000円の商品を買う場合、
同じく100円を引いて900円になったあと90ポイント分を引いて810円。
これに81ポイントがつくことになります。

これを現金値引きのみに換算した場合、何%引きになりますか。
6461:02/05/10 20:25
えーと・・・
今思い出したんですが(遅い)、
帰納法も使え、という条件でした。

この場合、
「n=1(2?)で一意的である」を示して、
「n未満で成立を仮定」
・・・とかを言えば良いんでしょうか?
65132人目の素数さん:02/05/10 20:28
>>17
y=-(|x|^2-ε^2)^(-1) と
t=1/(|x|^2-ε^2) の差は
符号だけだとわかって言ってんのかね
66132人目の素数さん:02/05/10 20:29
67質問です:02/05/10 20:35
箱1、箱2、箱3があったとする。
そして三つの赤いボール、四つの緑色のボール、そして五つの黄色いボールがあったとする。
三つの箱にボールを入れたとして、何種類の入れ方があるか。
ただし、
・各箱には最低一つでも入っていること
・同色のボールしか入っていない箱がないこと。

この問題をどうやって解いたらいいのかわかりません。
もしよろしければお助けください。お願いします。
>>65
いや、符号だけですけど、その符号がないとド・ロピタルの定理の形にならないんですよ。
掛け算形でもロピタルの定理って使えるんですか?
まぁなんにせよ、僕が馬鹿でした、すみません。
6961(15):02/05/10 20:49
うう・・・。
一応途中までやってみました。



素因数分解の一意性について(ある数を素因数分解した時、その組み合わせ(2×3など)は一通りしかない)


n=2の時については、
2=2^1×3^0×・・・であり、これは一意的。
n≧2の時、n未満の整数の素因数分解は一意的であるとする。

nの素因数分解が、
n=a(1)a(2)…a(x)=b(1)b(2)…b(y)・・・(@)と2通りあるとする。
ただし、a(p)≠b(p)(1≦p≦x、y、pは整数)、a(q)<a(q+1)、b(r)<b(r+1)
(1≦q、r≦x−1、qとrは整数)とする。

(@)は、(左辺がa(1)で割り切れるので)両辺がa(1)で割り切れる。
よって、素数の性質(pがmnを割り切るならば、pはmかnを割り切る)より、b(1)〜b(y)のいずれかはa(1)で割り切れる。

しかし、仮定より、a(1)≠b(1)<b(2)<b(3)・・・<b(y)となる為、
右辺には・・・・・・・。




1、帰納法の部分がこれで良いのか
2、右辺には・・・。の続き(このままいくと60さんのおっしゃっている循環論法になってしまう)

・・・どうすれば良いんでしょうか?
>>69
あのさ、一意性のみを示せばいいの?
それとも、素因数分解できる事も示さないといけないの?
>>64
「N=P_1*P_2*P_3*…*P_n=Q_1*Q_2*Q_3*…*Q_m と任意の仕方で
二通りに分解してみても、実は、同じ分解になっている」ことを証明する。

これは、「〜が一意である」という命題の証明ではよく使われる方法。

N についての帰納法でいい。(i=max(n,m)とおいて i での帰納法でもできる)。

Q_1〜Q_m の中に P_1 と同じ素数があることを示し、その素数で割る。
仮に、P_1=Q_k とすれば、それで両辺割って、

N'=P_2*P_3*…*P_n=Q_1*Q_2*…*Q_(k-1)*Q_(k+1)…*Q_m

となるが、N' がより小さい値になっているから、ここに帰納法の仮定が使える。
>>63
定価1000円のものを買うのに、全てポイントで払ったとすると、
900ポイント払って90ポイント返ってくるから結局810ポイント使ったことになる。
と言うことで19%引きだと思う。
ただし、そのお店で買い続けた場合の話。
7361(15):02/05/10 21:14
>70
いえ、一意性だけです。

>71
えーと・・・
69の「しかし〜」を消して、その代わりに


仮に、a(1)=b(k)とすると、
それで両辺(左辺:a(1)で、右辺:b(k)で)を割ると、(@)は
N=a(2)a(3)…a(x)=b(1)b(2)…b(k-1)b(k+1)…b(y)・・・(A)となる。(N=n/a(1))
このNは、当然nより小さいので、仮定よりNの素因数分解は一意的。
つまり、(A)のa(2)a(3)…a(x)とb(1)b(2)…b(y)は等しい。
そして、それにa(1)(仮定よりa(1)<a(2),b(1))をかけたnも素因数分解は一意的である。


を入れて、それで証明終わり、で良いんですか?
74132人目の素数さん:02/05/10 21:16
>>62
自明ではないが線形代数のどの教科書にも書いてあること
7563:02/05/10 21:19
>>72
全てポイントで払った場合は、ポイントは付かないんです。
ポイント値引き後の現金支払い分に対してポイントが付く計算です。
76132人目の素数さん:02/05/10 21:20
お願いします。教えてください。

次の連立方程式を解きなさい

x+y+z=1
3x+2y-2z=12
x^2+y^2+z^2=9
77132人目の素数さん:02/05/10 21:21
>>74
どうしてそういう答え方しかできないんかなぁ。

16 :132人目の素数さん :02/05/10 14:24
ベクトル空間で有限基底が取れるならば無限基底は存在しないということの
エレガントな証明ってありますか?
78132人目の素数さん:02/05/10 21:24
エレガントな方法を求めるからには、自分が知っている解法に
どんなものがあるのかを列挙すべきでは?
7961(15):02/05/10 21:26
何かバラバラに書いてしまって、見にくいので証明をまとめて書いておきます。
これで合ってる・・・んでしょうか?


n=2の時については、
2=2^1×3^0×・・・であり、これは一意的。
n≧2の時、n未満の整数の素因数分解は一意的であるとする。

nの素因数分解が、
n=a(1)a(2)…a(x)=b(1)b(2)…b(y)・・・(@)と2通りあるとする。
ただし、a(p)≠b(p)(1≦p≦x、y、pは整数)、a(q)<a(q+1)、b(r)<b(r+1)
(1≦q、r≦x−1、qとrは整数)とする。

(@)は、(左辺がa(1)で割り切れるので)両辺がa(1)で割り切れる。
よって、素数の性質(pがmnを割り切るならば、pはmかnを割り切る)より、b(1)〜b(y)のいずれかはa(1)で割り切れる。

仮に、a(1)=b(k)とすると、
それで両辺(左辺:a(1)で、右辺:b(k)で)を割ると、(@)は
N=a(2)a(3)…a(x)=b(1)b(2)…b(k-1)b(k+1)…b(y)・・・(A)となる。(N=n/a(1))
このNは、当然nより小さいので、仮定よりNの素因数分解は一意的。
つまり、(A)のa(2)a(3)…a(x)とb(1)b(2)…b(y)は等しい。
そして、それにa(1)(仮定よりa(1)<a(2),b(1))をかけたnも素因数分解は一意的である。



・・・そういえば、どこに「数学的帰納法より〜」とかって入れたら良いですか?(^^;)
80132人目の素数さん:02/05/10 21:29
>>77
次元の不変性の証明で行列式を使うのがあるけど2つの有限基底に対して
それらの元の数が一致するという証明だよね。

たしかに有限基底があるときには無限基底がないということを示して
おかなくてはならないね。
81132人目の素数さん:02/05/10 21:31
77は何をカラんでるんだ?
74の答えで不十分だと思うなら自分なりの答えを書けばいい
82132人目の素数さん:02/05/10 21:32
>>78
16 の設問を理解してないね。
83132人目の素数さん:02/05/10 21:34
>>81 逝ってよし。
84132人目の素数さん:02/05/10 21:34
からむのはタンだけにしょうぜ
85132人目の素数さん:02/05/10 21:39
16はいい設問なのにそれに対する回答が貧弱であることは否めないな。
8676:02/05/10 21:43
誰か教えて…
べクトルスペース上の任意の元が基底の線形結合であらわされるから
その有限基底にどんな元を付け加えてもそれは線形独立にはならない。
っつーんじゃ駄目かね?
>>63
かなり乱暴な計算かもしれないけど…。
ポイント10%というのは、1000円で1100円分のお買い物ができるシステム。
したがって、割引率は(1-1000/1100)=0.090909…=9.0909…%
よって、現金値引き10%、ポイント10%ならば、
割引率は1-(1-10%)*(1-9.0909…%)=18.1818…%

まちがってたら、ごめんよぉぅ。
89132人目の素数さん:02/05/10 21:48
>76
ちょっと今から計算してみます(間違ったこと教えたら大変なので(^^;)
90132人目の素数さん:02/05/10 21:54
>>87 トンデモですか?(ワラ
91132人目の素数さん:02/05/10 21:55
>>85
どこらへんがいい設問なの?
っていうか、ここは設問を書くスレなの?
92132人目の素数さん:02/05/10 21:56
>76
・・・(汗)
滅茶苦茶な答えになってしまった・・・


とりあえず、上2つの式からy、zをxの一次式で表して、それを3つ目のに代入すればOKのはずです。
・・・間違ってたらごめんなさい。
93132人目の素数さん:02/05/10 21:56
>>87
駄目。
16も馬鹿
87も馬鹿
っつーんじゃ駄目かね?
9563:02/05/10 21:58
>>88
ありがとうございます。19%より下なのは自分で計算できたのですが
そこからわからなかったので。これで納得できました。
96132人目の素数さん:02/05/10 21:58
>>94
O.K.
「叱らないと勉強しない」の対偶を教えてください。
「勉強すると叱る」は間違いですよね?
9876:02/05/10 21:59
>>92それをすると答えが虚数になってしまった…(泣
99132人目の素数さん:02/05/10 22:00
M_SHIRAIAHI のホームページでも見たのか? > 97
>>92 >>98
俺はその方法でx=2 or 38/21 と出たけどな。
もちろん合ってる自信はないが(w
>97
「勉強しているならば(既に)叱られている」
10299:02/05/10 22:04
-------風俗の総合商社・MTTどこでも-------

〇デリバリーヘルス〇デートクラブ〇女性専用ホストクラブ〇
〇ハードSM奴隷クラブ〇レズビアン倶楽部〇ホモ・オカマ倶楽部
〇変態痴女と遊ぶ会〇痴漢・覗き趣味の会〇変態同好会・各種!
●楽しく遊べます! 090-8002-8356番
-----------美男・美女会員など多数在籍中-----------
  http://www.mttdocomo.jp/
-----女性アルバイト随時募集・高収入(日払い)月100万円可能-----
-----レズビアン・スタッフ●ホモスタッフ●女性専用ホストスタッフ同募-----
http://www.mttdocomo.jp/


10387:02/05/10 22:04
やっぱ駄目か…
阿呆だな、がははは。
>94
いや、馬鹿は俺だけで十分だと思う。
直線y=ax+a^2+2a+3は定数aがどのような値でも,
一定の,y軸に平行な直線を軸とする,
放物線Cに接するという.このCの方程式を求めよ.
10576:02/05/10 22:05
>>100
途中式を書いてくれませんか?
10697:02/05/10 22:05
>>101
ありがとうございます
107104:02/05/10 22:07
中3なんで、中3にも分かるように
説明してくれると幸いです。
10887:02/05/10 22:08
問題に答える以前に設問の意味さえ分かっていないような気がする…
馬鹿な俺に誰か解説してくれないか?
109132人目の素数さん:02/05/10 22:08
>>91 設問を書くスレですが何か?
110兄貴:02/05/10 22:10
輪ゴムってあれかなぁ。一種のメビウスなの?
111132人目の素数さん:02/05/10 22:11
>>109
で16はどこらへんがいい設問なんだい?
112132人目の素数さん:02/05/10 22:11
盛り上がってまいりますた
113132人目の素数さん:02/05/10 22:12
ちみんとこの輪ゴムはねじれてんの > 110
114132人目の素数さん:02/05/10 22:12
>>76
{x = 38/21, y = 26/21,z = -43/21}
{ x = 2, z = -2, y = 1}
いい設問というより
どうでもいい設問…
昔の数学板は16みたいな電波がよく飛び交ってたものだが。。。
117132人目の素数さん:02/05/10 22:15
2ちゃんでいい設問するなんて!
118兄貴:02/05/10 22:16
>>113 断面が四角じゃなく丸いんだけど・・・。
119132人目の素数さん:02/05/10 22:20
Mathematica 使ってない? > 114
漏れの Mathematica と同じだった
12088:02/05/10 22:21
>63
このポイント還元というシステム、
割引率と明らかに違うので、ちょっと前に計算したことがありました。
ポイント10%:割引率=(1-1000/1100)= 9.0909…%
ポイント13%:割引率=(1-1000/1130)=11.5044…%
ポイント15%:割引率=(1-1000/1150)=13.0434…%
となるはずです。
121100:02/05/10 22:22
>>114 >>119
をを、やっぱりそうなるか!
漏れの手計算も、捨てたもんじゃないな。
122132人目の素数さん:02/05/10 22:23
nコの基底があるとして、任意の(n +1)コのベクトルは
一次従属になるに決まってるじゃんか。
どこがどう、「いい設問」なんだ。エレガントもクソも
あるかい。>>16は逝ってよし
12387:02/05/10 22:23
いや、煽ってばかりいないでほんとにお願い。
初めはどうでも良かったんだけど、自分がわからないとなるとやっぱり困る。
自分の阿呆さ加減を棚上げして言うわけじゃないけど、分からないわけじゃないんだろ?
別にエレガントじゃなくてもなくてもいいから、頼みますよ。
12487:02/05/10 22:25
>122
それだと、俺のと大差ないような気がするけど…
それであっているのか?
12576:02/05/10 22:26
>>114途中式も書いてくれませんか?
126132人目の素数さん:02/05/10 22:26
>>122 おまえアホ決定。
>>79
なんか、ちょっと変な感じ。

はじめに、n=a(1)a(2)…a(x)=b(1)b(2)…b(y) とおいた段階では、
この両辺が異なる分解とは仮定しないわけよ。
だから、「a(p)≠b(p)」とかいう条件はいらない。

それに、a(q)<a(q+1)、b(r)<b(r+1)とかいうのは
素因数が重複して入っているケースを忘れている(50=2*5*5とかね)。
この証明で、a(i),b(j)の大小について述べる必要はないよ。

適当に二通りの分解を取ってみても、じつは、

「『a(1),a(2),…,a(x)』と『b(1),b(2),…,b(y)』は、
順序を除いて同じものになっている」

ことを帰納法で証明するわけ。
それがいえれば、素因数分解が一意ということと同じでしょ。
>>122
ベクトル空間で、加算個の基底が取れるが、
同時に非加算個の基底も取れるものが存在する。
129132人目の素数さん:02/05/10 22:29
>>126
アフォはおまえじゃ。
文句があるなら、一次独立な(n + 1)コのベクトルをとってみろ。
回線切って(以下略
130132人目の素数さん:02/05/10 22:30
>>104
ax+a^2+2a+3
=a^2 + (x+2)a + 3
=[a + {(x+2)/2}]^2 - {(x+2)/2}^2 + 3
=(1/4)(x + 2a + 2)^2 - (1/4)x^2 - x + 2
だから y=ax+a^2+2a+3 と y=(-1/4)x^2 - x + 2
は(-2a - 2, -a^2 + 3)で接している。
Cの方程式は
y=(-1/4)x^2 - x + 2
13116:02/05/10 22:31
87さん、どうもです。

116さん、そんなに電波な質問でしたか?
>>125
おそらく君の計算が間違っているのだから、
それがどこかわからなくて困っているのなら、
君の方が計算過程を書くべきだ。
133132人目の素数さん:02/05/10 22:32
あふぉどうし仲良くしようぜ
13463:02/05/10 22:32
>>120
通販の店選びで迷ってたのでさっそく参考にさせてもらいます
135132人目の素数さん:02/05/10 22:34
>>128
それが一体、何の反証になっているというんだ??
有限個の基底がとれて、任意のベクトルがその基底の線形結合で
表される場合のハナシだろ?
関係ないじゃん。
>129

>nコの基底があるとして、任意の(n +1)コのベクトルは
>一次従属になる
という命題をどう証明するか、というのが
そもそもの問題でしょ。
どの線形代数の教科書にも載ってるはずだけど。

正しい命題だからといって「決まっている」っていうんじゃ
数学にならないわけで・・・
13787:02/05/10 22:38
>nコの基底があるとして、任意の(n +1)コのベクトルは
>一次従属になる
ようやく、自分の間違いに気がついたよ…すまん。
138132人目の素数さん:02/05/10 22:38
そりゃあ、>>136の言う通りだけど‥‥。
だから、エレガントも何もないっちゅうねん。
n x (n + 1)の行列の階数を、どうやったら(n + 1)以上にできるっちゅう
んじゃ‥‥。

「決まっている」っていう言い方は、確かに漏れが悪かった。
139132人目の素数さん:02/05/10 22:39
久々の上がり調子ですだ
140132人目の素数さん:02/05/10 22:43
わっしょいわっしょい。
141132人目の素数さん:02/05/10 22:44
祭りがあるのはここですか?
142132人目の素数さん:02/05/10 22:45
1999^213÷2000の余りってどうやって求めるん?
143132人目の素数さん:02/05/10 22:46

マルチ
>>104
点(X,Y)がその直線を通る ⇔
Y=aX+a^2+2a+3 を満たす実数aが存在する ⇔
Y≧-(1/4)(X+2)^2+3 。

従って、そのような放物線が存在するとすれば、
それは y = -(1/4)(x+2)^2+3 以外にはあり得ない。

これが本当に題意を満たす放物線かどうかは、
別途検証する必要がある。
145132人目の素数さん:02/05/10 23:15
平面上に2点A、Bと直線Lがある。L上に点Pをとり、内積s=(AP↑,BP↑)を考える。
L上でsを最小にする点PをQとおく
(1)この平面上でLを平行移動させると、Qは1つの直線上を動くことを示せ。
(2) (1)で求めた直線は、ABの中点を通りLに直交することを示せ。

この問題がわかりません。
誰か宜しくお願いします
146うきゃ@1年ぶり:02/05/10 23:34
>145
具体的に座標軸においてみよう
例えば,直線Lをy=eとし,A(0,a),B(b,c),P(d,e)としても一般性を失わない.
その上でsを求め,dを変数と見て平方完成してQの座標を求めると
Qのx座標はeによらないことがわかります

・・・そうすると(1)(2)同時に解けてしまったなぁ(--;;;
147132人目の素数さん:02/05/10 23:50
>>128
>ベクトル空間で、加算個の基底が取れるが、
>同時に非加算個の基底も取れるものが存在する。

いや、たしか濃度は一定だったはずだが。
148分からずや:02/05/10 23:57
ぬおー誰か助けてください。(マジです)

nは2以上の自然数とする。
k=1,2、・・・、nについて、整式P(x)を
x−kで割った余りがkとなった。p(x)を(x−1)(x−2)・・・(x−n)
で割った余りを求めよ。

数学T・Aだとおもいます。
149132人目の素数さん:02/05/10 23:59
次のものを希望します。
(1)Q⊂K⊂Fで、K/Q, F/Kがガロア拡大のとき、F/Qがガロア拡大とならない例。
(2)非分離拡大の例。
よろしくお願いします。是非。
>148
P(x)-x を x-k で割った余りが何になるか分かるか?
151分からずや:02/05/11 00:02
>>150
??わかりません、、
152うきゃ@1年ぶり:02/05/11 00:09
>148
P(x)を(x-k)で割ったときの商をQ(x)とすると
P(x)=(x-k)Q(x)+k (k=1,2,・・・,n) とおけるので
P(k)=k

P(x)を(x-1)(x-2)・・・(x-n)で割ったときの商をR(x),あまりをS(x)とすると
これについても等式がたてれて
R(1),R(2),・・・,R(n)の値が分かる.
R(x)は高々(n-1)次式なので,通る点をn個与えられれば一意に決まる・・・って自明じゃないな(--;;; どうしよ
153132人目の素数さん:02/05/11 00:12
>>152
150の後にそういうことを書くあなたを尊敬します。
お手数おかけしますが、どなたか助けていただけませんでしょうか?
統計でデータの対数や比率をとると外れ値が減少する例って
どんなものがあるんでしょうか?
155150:02/05/11 00:15
>153
おいおい、煽るなよ。
多分レス書くの夢中で
読んでなかっただけでしょ。
>>153
ワラタ

>>151
君、因数定理しってる?
157うきゃ@1年ぶり:02/05/11 00:16
>152
あー,やっぱ僕教科書の応用みたいな答え方しかできないよ.
>150 見て感動した
158分からずや:02/05/11 00:21
>>156
名を聞いた事はありますけど、説明となると・・・
>158
手遅れだ…
160150:02/05/11 00:27
>156
今回は因数定理いらないでしょ。
(因数定理でもいいけど)

>158
P(x) を x-k で割ったら k 余った。
ッてことは、
P(x)-x を x-k で割った余りは
k-x を x-k で割った余りと 一緒だよね?


161分からずや:02/05/11 00:28
>>159
思い出しましたけど、まだ分かってません・・・
162分からずや:02/05/11 00:34
>>192
>k-x を x-k で割った余りと 一緒
が分かりません。
スンマセン、、、
163150:02/05/11 00:39
>162
P(x) を x-k で割ったら k 余った。
⇔ (ある多項式Q(x)に対して)  P(x) = (x-k)Q(x) + k
なので、

P(x)-x = (x-k)Q(x) + k - x

P(x)-x を x-k で割ることを考えるのだが、
(x-k)Q(x)の部分は(x-k)でわりきれるから、
残りの部分は k-x
164分からずや:02/05/11 00:39
>>192
ごめんなさい、バカです。分かりました。
165分からずや:02/05/11 00:41
>>192>>163に訂正
あせってたもんで・・・
166分からずや:02/05/11 01:01
ぬおー続きがわかりません・・・
167132人目の素数さん:02/05/11 01:16
解の公式について知りたいと思ってこんなページを見つけたのですが
さっぱり理解できません。

ttp://www.sci.hokudai.ac.jp/science/H12_10/math/Math_2000_3.htm

この辺の事を詳しく書いてある本を教えて下さい。
168132人目の素数さん:02/05/11 01:19
>92,98,100
俺も、x=2,38/21となった。多分合ってる。
169168:02/05/11 01:29
おーい。76、みてるかー?
1番目の式と2番目の式を、xを定数として解くと、
y=(-5x+14)/4
z=(x-10)/4となる。
これを3番目の式に代入すると、xについての2次方程式になる。
その2次方程式を整理すると、21x^2-80x+76=0
これを解いて、x=2,38/21となる。
170132人目の素数さん:02/05/11 03:44
初めて質問します。よろしくお願いします。

以前から疑問があります。
実数関数の相互相関関数は

∫f(t)×g(t+τ)dτ

ですよね?
では、複素関数の場合はなぜ

∫f(t)×{g(t+τ)}*dτ (*は複素共役)

と言う感じに関数gの複素共役を積分することになるのでしょうか?
この場合のイメージが湧いてきません。。。。

直感的なイメージ、上記のように共役複素数を用いる理由をどなたか
教えていただけないでしょうか?よろしくお願いします。

それと、相関関数や複素行列における固有値などを詳細に解説している
数学の参考書はありますか?

以上よろしくお願いします。ちなみに私は通信工学系の学生です。
171132人目の素数さん:02/05/11 03:51
>>170

複素数の場合、内積は片方の成分を複素共役にとるからだよ。
なぜそーするかは、複素数の場合、絶対値はどうやって求めるか考えてみよ。
172132人目の素数さん:02/05/11 04:18
>>170

ありがとうございます。でも今ひとつイメージが湧きません。。。。

例えば、今二つの複素数(ここでは関数ではなく、単純に複素数を考えます)

α=a+bi, β=c+di (iは虚数)があるとします。
この複素数の絶対値は、例えばαの場合、

|α|=√(a^2+b^2)

ですね。これは分かります。それで、この2つの複素数の内積ですが、
各複素数の成分同士の和ですよね?

この場合、a・b=ac+bd ですか?それともa・b=ac-bdなのでしょうか?

手近な場所に参考書がありませんので、ご教授の程、よろしくお願いします。


173171ではないが:02/05/11 04:26
>172
>2つの複素数の内積ですが、
>各複素数の成分同士の和ですよね?

違う。
α・β
= α β~ (β~ はβの共役複素数)
= (a+bi)(c-di)
= (ac+bd) + (-ad+bc)i
だよ。

念のため、この定義で
α・α = |α|^2
になることを確認されたし。
174132人目の素数さん:02/05/11 04:29
>>172
>|α|=√(a^2+b^2)

いきなり、このようにかくとわからなくなるが、これって
|α|=√(α×α*)=√{(a+bi)×(a-bi)}=√(a^2+b^2)
だってわかる?
175173:02/05/11 04:30
こんな時間に結婚するとは・・・
>174 挙式はハワイで。
176174:02/05/11 04:36
ねむいよー
177174:02/05/11 04:40
それより、内積は片方の成分をCCにするけど、外積はどーするか知ってる?
>173
178オラ来る:02/05/11 04:52
tan(x/2)=tのとき、次の等式を証明しなさい。
(1) sinx=2t/1+t^2
(2) cosx=1-t^2/1+t^2
(3) tan^2(x/2)

この問題、全然わからないです。
半角の公式とか使うんですか?
右辺にタンジェントを放り込んだら訳わかんなくなっちゃいました。
スミマセンが教えてください。
179オラ来る:02/05/11 04:53
(3)間違えました。

tanx=2t/1-t^2

っていう問題です。
180132人目の素数さん:02/05/11 05:00
>>173、174

本当にありがとうございます。こんなくだらない質問に答えてくださり、
感謝します。

いままで自分があまり複素数を理解していなかったことがわかりました。
でも、もう少し疑問があります。

α=a+biはベクトルと考えることもできますから、その内積は各成分同士の
積の和、として、どうして考えてはいけないのでしょうか?


|α|=√(α×α*)=√{(a+bi)×(a-bi)}=√(a^2+b^2)

となるのは理解できますので、複素の場合、その内積は複素共役をかけるべき、
と、頭では理解できましたが、実数ベクトルとの整合性(私の頭の中での話ですが。。。。)
が取れていないように、どうしても感じるのです。

つまり、どうして 

(a,bi)・(a,bi) と考えてはだめなのでしょうか?

定義の問題でしょうか?
理解が足らないのでしょうか。。。。。
それとも、「複素の世界はそう定義されている」と理解すべきでしょうか?

よろしくお願いします。
>実数ベクトルとの整合性
とれてるでしょ、一応
182174:02/05/11 05:03
sin(x)
=2sin(x/2)cos(x/2)/(cos(x/2)^2+sin(x/2)^2)
=2tan(x/2)/(1+tan(x/2)^2)
=2t/(1+t^2)

cos(x/2)
=(cos(x/2)^2-sin(x/2)^2)/(cos(x/2)^2+sin(x/2)^2)
=(1-tan(x/2)^2)/(1+tan(x/2)^2))
=(1-t^2)/(1+t^2)

tan(x)
=2tan(x/2)/(1-tan(x/2)^2)
=2t/(1-t^2)
183174:02/05/11 05:05
>>182
>cos(x/2)
>=・・・

cos(x)
=・・・
>177
外積は何もしないでいいんじゃないの?
185174:02/05/11 05:07
>>182

最後のtanは
tan(x/2)=sin(x/2)/cos(x/2)と計算してもええよ
186132人目の素数さん:02/05/11 05:09
>>181

(a,bi)・(a,bi) と考えてはだめなのでしょうか?
といいますのは、単純に、やりかただけ真似て、

(a,bi)・(a,bi)=a^2-b^2

になってしい、a^2+b^2にならないな、ということです。

何か単純なことをだらだら書いてしまい、申し訳ないです。。。。

187オラ来る:02/05/11 05:09
>174さん
さっそくの解説ありがとうございます。
今から頭を整理して理解してみます。
取り急ぎお礼まで。チュッ。
>>180
そのように定義しても別にいいけど、例えばその内積だと、(z,z)<0と
なる場合もあって、(z,z)=0<=>z=0とさえ言えなくなる。
道具としては、余り便利なものではない。
普通の内積 (z1,z2):=z1(z2)~ {(z)~は共役の意味}
とすると、道具としてはずっと強力になる。
189174:02/05/11 05:12
>>187
>チュッ

いやん。はずかし・・・
>180
もちろん定義の問題なのだけど、
その定義だと、内積の重要な性質である
線形性がウマく成立しないんです。

実ベクトルでは
(ta)・(sb) = st(a・b) (a,bは実ベクトル, s,t は実数)
という性質が成立してますね。

複素ベクトル同士の内積を
(a[1],…,a[n])・(b[1],…,b[n]) = a[1](b[1]~)+…+a[n](b[n]~)
で定めれば、
(ta)・(sb) = s(t~)(a・b) (a,bは複素ベクトル, s,t は複素数)
という、上に似た性質を実現できるわけです。
しかも同時に、(a,a)≧0 が任意の a に対して言えて、
(a,a)=0 と a=0 が同値になる、といいことずくめなわけです。
191132人目の素数さん:02/05/11 05:17
>>181さん

ありがとうございます。確かに言われるとおりですね。
うーん、私の頭が固いのかも知れません。

もうちょっと勉強が必要かも知れません。
しかし、相関関数の話から基本的な複素の内積にまで話が及ぶとは
思いませんでした。
皆さん、ありがとうございました。またちょくちょくご質問させていただきます!
192オラ来る:02/05/11 05:21
>174さん
sinとcosは倍角のちっちゃい版を使うのはなぁんとなくわかりましたけど、
分母にsinの2乗+cosの2乗がでてきちゃうのは、経験でそんな
技がでてくるんですか?こんな問題いきなりみても絶対思いつかないです。
なんでそんな発想がでてくるのかな?
193132人目の素数さん:02/05/11 05:25
>>188(さっき181さんって書いてしまいました)さん、190さん

ありがとうございます。

>>190さん

さっそく簡単な問題を自分で考えて確認してみます。
本当にありがとうございます。

最後に質問です。
複素の内積、外積について、190さんの言われているような議論を
紹介している参考書はありますか?

よろしければ教えていただけないでしょうか?
天下り式に「このように定義する」的な解説なあまり好きではありません
ので。。。。。
194174:02/05/11 05:27
えへへ♪
やっぱり経験の差だよ(うそ)

本当は昔だれかがそうやって変形しているのをみたからだよん。
sin(x),cos(x),tan(x)をtan(x/2)で表す式は応用範囲の多い式だから、
いつでも簡単に求められるように、この変形法を覚えておくのも吉だよ。
195190:02/05/11 05:27
>複素の内積、外積について、190さんの言われているような議論を
>紹介している参考書はありますか?
残念ながら、しりません。
自分で試行錯誤して、他の定義はうまくいかないことを
確認するのが良いと思います。
>192 >194
結果をしらないのならば、右辺を整理して、倍角公式の形にして
左辺を導く方が自然でしょう。そんなに難しくないですよ。
やり方分かってるなら、>182みたいに好き放題やれば
いいんですけどね。
197174:02/05/11 05:41
>>193

内積については、ふつーの線形代数の本に定義とちょっとした説明は書いてあるけど、
外積については、どの本をみてもどれも実ベクトルだけに限定して書かれていて、
なぜか複素ベクトルについては定義すら議論してないんだよね。

ただ、電磁気学の電磁場の計算の中などで、一方の成分をCCにして
複素ベクトルの外積を定義しているんだけど、この場合は、その計算結果の
実数部分だけを取り出すといった、条件付きでやってるんです。
なんたってこのままふつーに使うと、α×α=0にならんし・・・・

ってことで、だれかしらないかなぁ?
198オラ来る:02/05/11 05:43
>>196さん
右辺を整理仕切れずに煮詰まって夜を明かしちゃいました。。。
右辺から導いてみたいです、正直。

えっともう一個きいてもいいですか?
199オラ来る:02/05/11 05:46
(1) e^logx=x
(2) e^logb=b^a
これらの等式が成り立つことを示しなさい。

やっぱり左辺から右辺を導くんですか?
対数の公式っていうか、決まり事使っちゃダメなんですよねやっぱり。
200オラ来る:02/05/11 05:50
>>194の174さん

174さんはすごいですねホント。
前に見たことある変形をサラッと取り入れちゃうんだから。
自分にはそんな経験がないんで、高校の教科書をひっくり返して
あきらめていました。
201174:02/05/11 05:57
>>199

(2)の左辺は、e^log(b^a) または e^(alog(b)) のどっちかのまちがいだよね。
(1)(2)も変形だけなら両辺をそれぞれ対数をとってみれば一発で証明終わりなんだけど・・・

log_{a}(b)の値って、「aを何乗したらbになるか」って考えたときの
「何乗」の数のことじゃん。(例えばlog_{2}(8)の値は、2を3乗すれば8だから、3)
ってことは、(1)の場合、log(x)の値ってeを何乗かしてxになる数なんだから、
その数をeの指数にしたら当然xになるよねぇ。(頭こんがらがってる?)
202オラ来る:02/05/11 06:05
>>201の174さん
ホント丁寧な解説ありがとうです。
こんがらがったけどちゃんとわかりました!
そして(2)の問題はご指摘の通り書き間違えました。。。
問題はe^alogb = b^a
でした。
朝早くしつれいしました〜。
またお世話になりに来ると思いますので、その時も
よろしくお願いします。
ルートiって、
複素数であらわせますか?
>203
i=exp(iπ/2)
exp(iθ)の2乗がiになるとすると

θ=π/4 or 5π/4
205132人目の素数さん:02/05/11 08:50
>203
±(cos45°+isin45°)
普通は+のほうをとるだろうとおもう
206132人目の素数さん:02/05/11 11:57
久しぶりにフーリエ変換の問題を解こうと思ったら
どうしても上手くいきません。
誰か教えてください。

F(ω) = ∫[-∞,∞] sin(2t)*e^(iωt)dt

この式をそのまま計算すればいいはずです。
詳しい解法を教えてくれるとありがたいです。
207132人目の素数さん:02/05/11 12:30
>>128
>ベクトル空間で、加算個の基底が取れるが、
>同時に非加算個の基底も取れるものが存在する。

そんなベクトル空間あるの?
208ねーよ:02/05/11 13:25

209132人目の素数さん:02/05/11 14:02
>>206
sin2tを指数関数で表わしてみたらどうかなあ。
>>207 >>208
206を見ていて思ったんだが、次の2つの関数族

{e^(iθx) | θ∈実数}
{e^(iθx) | θ∈有理数}

は、どちらも関数空間L^2かなんかの
基底になるんじゃないか?
基底とは、独立だけでなく、きちんと生成する、つまり有限個の線形和ですべてを
尽くすことが出来るということです。
>>211
それは"線形空間"の基底
213132人目の素数さん:02/05/11 16:05
2/1k(k+1)+(k+1)がどうして
2/1(k+1)(k+2)になるんですか?
全然わからん
214132人目の素数さん:02/05/11 16:06
>>212
その話をしてるとおもうが
215132人目の素数さん:02/05/11 16:09
(k−9)(k−1)>0  がどうして
k<1,9<k  になるのですか?
216132人目の素数さん:02/05/11 16:09
>>212
L2は線型空間じゃなかったのか。
>>211
有限個の線型結合だけしか考えなきゃ無限次元なんか存在しなくなるだろ。
217132人目の素数さん:02/05/11 16:14
>有限個の線型結合だけしか考えなきゃ無限次元なんか存在しなくなる

これはちょっと…なんのこっちゃ?
218216:02/05/11 16:16
>>217
我ながら禿げ同(w
219132人目の素数さん:02/05/11 16:17
>>215 グラフを書くとわかる
220132人目の素数さん:02/05/11 16:18
>>215
ならない。
>>213

>>3の規則守って正しく書いてくれ。2/1=2
22261(15):02/05/11 16:24
>213
2/1kって・・・二分のk(k/2)の間違いですよね?

だとすると・・・
(k+1)でまとめてみて下さい。
そうすると、

(k+1)(k/2+1)

で、1/2を外に出して、

1/2(k+1)(k+2)

になります。
sin(mx)という形の関数のいくつか有限個を取って線形和を作る
f(x)=Σ[i=1..k]αk sin(m_i x) kは自然数
こういった形の関数の全体は、一つ一つは有限個のsin波で表されているけど
線形空間を為し、しかも無限次元です。L^2はこういった関数の列の極限も
仲間に入れて作った線形空間でこれももちろん無限次元です。性器直交邸と
基底は別物と考えるべきです。
224223:02/05/11 16:54
m_i→m_kです。訂正
225223:02/05/11 16:58
f(x)=Σ[i=1..k]αi sin(m_i x) kは自然数 m_iは自然数でなくても
いいやすべて異りさえすれば
.....
に直してください。(つながりが悪くて...鬱氏)


226132人目の素数さん:02/05/11 18:31
「Kroneckerの青春の夢」ってなんですか?
227132人目の素数さん:02/05/11 19:19
長さ12cmのひもでおうぎ形をつくるとき、次の問いに答えよ。
(1)おうぎ形の半径をr,面積をSとして,Sをrで表せ。
(2)Sが最大になるような半径と中心角を求めよ。

(1)

2r+l=12
l=12-2r
l=rθだから
θ=l/r=(12-2r)/r
S=(1/2)(r^2)θだから、
S=(1/2)(r^2)*(12-2r)/r=r(6-r)=-r^2+6r

(2)

S'=-2r+6=-2(r-3)
S'=0となるrの値は3で、
その前後でS'の符号は正から負に変わるので、
関数S=-r^2+6rはr=3のとき極大で、そのグラフは上に凸だから、
r=3のときSは最大になる。
よってl=12-2r=6
l=rθ
θ=2

半径3cm.中心角2

しかしながらこの問題は三角関数の問題であり
まだ微分法を習っていない人向けの問題なので、
わたしの解き方は模範解答ではないのですが、
微分法を使わない模範解答が分かりません。
微分法を使わずにどうやって解けばいいでしょうか?
よろしくお願いします。
228132人目の素数さん:02/05/11 19:21
誰か解いてくれ。ベクトル解析の問題なんだが、

次の公式を証明せよ。
1,divr〔(gradφ)×Aベクトル〕=−(gradφ)・rotAベクトル
2,grad〔(gradφ)×Aベクトル〕=〔(gradφ)・∇)Aベクトル
                      +(Aベクトル・∇)(gardφ)
229132人目の素数さん:02/05/11 19:24
>>228定義にぶち込んで終わり。
230132人目の素数さん:02/05/11 19:26
じゃあ、定義を証明してくれんか?
231132人目の素数さん:02/05/11 19:29
な  に  ?
232132人目の素数さん:02/05/11 19:29
どうみても収束しそうな気がしないんだが > 206
233132人目の素数さん:02/05/11 19:35
>>227
う〜ん、真剣に書いているのだろうか?
2次関数で微分使ってるし、弧度法使ってるよ
う〜ん
234132人目の素数さん:02/05/11 19:37
>>233
わたしは真剣です。
微分を使えば解けるけれども使わない解き方が分からないので
それをご教示願いたいのです。
弧度法は三角関数で習う内容なので、使っていいです。
235132人目の素数さん:02/05/11 19:39
>>230
定義を証明??????????????????????????
236与作は木を切る〜へいへいほー:02/05/11 19:41
平方完成すれば?
あと、今のカリキュラムでは数IIの三角関数では弧度法は扱かわない
237132人目の素数さん:02/05/11 19:41
>233
ひまなだけではあれだけ書けんだろう>227
文系の人が家庭教師でもやるとこんな感じ。余計な詮索だった。
238132人目の素数さん:02/05/11 19:43
>227
微分を使わなければ、2次関数の問題でしょ。他に何があるの。
ラジアンが分かっていて(1)ができればどうということのない問題。
(ラジアンしらなくても比でなんとかなる)
239132人目の素数さん:02/05/11 19:54
228です。さっきのなし。問題うろ覚え。
次の公式を証明せよ。
1,∇・[(∇φ)×Aベクトル]=−(∇φ)・(∇×Aベクトル) (スカラー)
2,∇・[(∇φ)・Aベクトル]=[(∇φ)・∇]・Aベクトル
                +(Aベクトル・∇)(∇φ) (ベクトル)
 やったと思うんだけど、これでもやっぱり定義に入れろってか?
240ベホマズン ◆DEENepXc :02/05/11 19:59
>>前スレのこけこっこタン
T京R科大はキターけど、今はT場大生としてマターリやっています。
詩・・・作ってみました。




僕はひとつの小さな丘を越えた

たった一つの小さな小さな丘を

いつもとは違う世界に

戸惑いながらも

確実に一歩一歩、、、

僕は夢に向かって

暖かな祝福の中

和やかな微笑みの中

笑ったり、泣いたり、、、


そして僕はまた新しい自分を見つけに

旅立っていく
241:02/05/11 20:00
>>227
(1)
中心角をθとおく
r+r+2πr*(θ/360)=12
θ={180*(12-2r)}/πr
S=π*r^2*θ=π*r^2*[{180*(12-2r)}/πr]/360=6r-r^2

(2)
S=6-r^2=-(r-3)^2+9
よって、r=3のとき最大値9
r=3よりこの長さは6
2πr*(θ/360)=6
θ=360/π

ごちゃごちゃしてるけど...
242132人目の素数さん:02/05/11 20:00
>>236>>238
昔の教科書を使っているもので・・・
二次関数のグラフは平方完成によって書けるということは
知りませんでした。教えて下さり有難うございました。
243241:02/05/11 20:03
ちょいっと訂正
(2)
S=6-r^2=-(r-3)^2+9
よって、r=3のとき最大値9
r=3より弧の長さは6
2πr*(θ/360)=6
θ=360/π(°)
244132人目の素数さん:02/05/11 20:09
>>241
それが今の高校数学の模範解答なんですね。
ありがとうございました。
245132人目の素数さん:02/05/11 20:26
確率の問題です。
どなたかお願いします。

3×3(9マス)の中に、1〜8までの任意の数を最低2個、最高8個
記入して参加する変則型ビンゴゲームがある。
(同じ数は重複しない。真ん中はFREE。)

1〜8の中から3つの数字が選ばれ、ビンゴの当選が決められる。
(縦、横、斜めどのラインでもよい。)

2個記入した場合から8個記入した場合まで、
それぞれビンゴになる確率を求めよ。
(ただし、数字を記入する位置は最も確率の高い場所であるとする)

俺にはわかりませんでした…。
問題文は人から聞いたものを自分で文章にしたものなので、
変なトコとかあったら言ってください。
よろしくお願いします。
>>240
T場大、受かったんだあ!おめでとう。。
詩、ありがとう。保存しておきます。
たまには受験板きてね。。
247132人目の素数さん:02/05/11 20:54
二直線が交わると四つの角ができます。
四つのうち二つは同じ角度になって、
残りの二つも同じ角度になりますが、
それはなぜでしょうか?
また、それは何という定理でしょうか?
248質問:02/05/11 21:28
『f(x)=x^3-3ax^2+3bx+1が0≦x≦1において単調増加するとき、
点(a,b)の存在する範囲を求め、図示せよ。』

という問題があるんですが、
これを解いていくと、
1≦a-√(a^2-b)または
a+√(a^2-b)≦0という風に出るじゃないですか。

このとき、√(a^2-b)は実数であるために、
a^2-b≧0じゃないといけないと思うのですが、
a^2≧bより領域はa^2<bの範囲は含まれないと思うのですが、
解答では含まれています。
どうしてなんでしょうか?
249132人目の素数さん:02/05/11 21:28
対頂角は等しいの定理
250132人目の素数さん:02/05/11 21:32
>>248
「これを解いていくと、 1≦a-√(a^2-b)または
a+√(a^2-b)≦0という風に出るじゃないですか」
がおかしい
251248:02/05/11 21:39
>>250
f(x)が0≦x≦1において単調増加ということは、
f(x)の極大値であるa-√(a^2-b)が1以下、
もしくは極小地であるa+√(a^2-b)が0以上であればいい、
ということではないのですか?
252132人目の素数さん:02/05/11 21:39
>247
ある点を通って直線の作る角は180度だから。
非ユークリッドだと成り立たないかもしれな愛。無知なのに知ったかぶり。
253248:02/05/11 21:41
>>251
極小地→極小値でした。
254一数学者:02/05/11 21:41
Rij-1/2gijR=−kTij

十元連立方程式
非線形微分方程式
255132人目の素数さん:02/05/11 21:45
>>251
極値を持たなければ全然OKっしょ
あと、日本語が変だぞ
256132人目の素数さん:02/05/11 21:51
>>249
有難うございます。

ニ直線の交点にできる四つの角のうち
∠Aと∠C、∠Bと∠Dを対頂角とすると、

∠A+∠B=180°
∠C+∠B=180°
よって∠A=∠C
同じようにして∠B=∠D

以上で対頂角は常に等しいことが示された。
257248:02/05/11 21:52
>>255
極値を持たない、ということは、
f'(x)=0における判別式Dが、
D≦0になったときは、a,bはどんな値でも取れるということで
いいのですか?
>>251は、判別式が正の場合の話をしてたってことなんでしょうか?

質問ばかりですみません。
よろしくお願いします。
あるクラスで席替えをします.生徒は全員で40人.
誰一人として元の席にならないような席替えのパターンは
全部で何通りありますか.どうやって数えれば良いかよくわかりません.
ヒントだけでもよろしくおねがいします.
259132人目の素数さん:02/05/11 22:18
>258
こういうのに全然自信が無いんですが、一応・・・

同じ席に座れないんだから、最初の人は40−1の39通り。
次の人は39−1=38通り。・・・

で、39×38×37×・・・×2×1

・・・かな?間違ってる気がしますが(駄目じゃん)
>259
最初の人が2番目の人の席に座っている場合
2番目の人は40−1通りの選択が可能
261132人目の素数さん:02/05/11 22:29
mathematicaで、
a[x_List,y_List]:=x[[y]]
こういう関数あったとして、

b[x_List]:=
がx[[y]]={1,2,...,n}になるようなyを表示させる関数にしたいんだけど
どうすればいいどすか?
262132人目の素数さん:02/05/11 22:41
>>258
最初の人は39通りの選択が可能

1人目の席を決める(39通り)
→1人目が座った席だった人を2人目とする
→2人目の席を決める(39通り)
→2人目が座った席だった人を3人目とする
→3人目の席を決める(38通り)

→38人目の席を決める(3通り)
→39人目と40人目は席が決まっている(1通り)

39x39x38x37x…x3=39!x39/2
263258:02/05/11 22:51
2人目の人が選んだ席が,1人目の人の元の席だったりしたら
どうするのでしょう?
>>258
これは、かく乱順列ってやつ。
正解は、次の通りだけど、説明がちょっと面倒。

40!*Σ[k=0,40](-1)^k/k!

だれかうまい説明考えてみてね。
265245:02/05/11 23:02
よろしければこちらもお願いします…。
>>245
266132人目の素数さん:02/05/11 23:11
>>245
1〜8の中から3つの数字が選ばれ、ビンゴの当選が決められる。
(縦、横、斜めどのラインでもよい。)

これの意味がようわからん。
267245:02/05/11 23:16
>>266
例えば、「1,3,8」とか、「2,4,6」とか、
ナンバーズ3みたいな感じです。
(同じ数字は選ばれません)
268132人目の素数さん:02/05/11 23:23
こっちも頼む〜。
≫239
269132人目の素数さん:02/05/11 23:24
>>239
滝本は、スーパーマン。

サリンという極めて毒性の高い神経ガスを三度も吸引させられながら、軽症で
済んでいる。これ、まさに奇跡。

サリンの副作用で、瞳孔が収縮する。タキモト大先生の場合は、その縮瞳が、
な、なんと自然に治癒してしまったそうだ。これは、医学の常識を超えた、
極めて特異な事象であり、医学的には説明できない。奇跡。

サリン襲撃が嘘であったとすれば、そんなもん不思議でもなんでもない。

オウム事件でタキモトの果たした役割は、>>239程度の人物には理解不能だろう。
タキモトは「マインドコントロール説」を垂れ流し、オウムの背後に隠れていた
連中への追及を断ち切るのが、与えられた使命だった。タキモトは、オウムには
北朝鮮も統一協会も創価学会も何ら関係なかったと言っている。その「オウムに
カンケイない」3つの組織がタキモトのスポンサーだよ。
だれか>>261
271132人目の素数さん:02/05/11 23:30
>>270
滝本は、スーパーマン。

サリンという極めて毒性の高い神経ガスを三度も吸引させられながら、軽症で
済んでいる。これ、まさに奇跡。

サリンの副作用で、瞳孔が収縮する。タキモト大先生の場合は、その縮瞳が、
な、なんと自然に治癒してしまったそうだ。これは、医学の常識を超えた、
極めて特異な事象であり、医学的には説明できない。奇跡。

サリン襲撃が嘘であったとすれば、そんなもん不思議でもなんでもない。

オウム事件でタキモトの果たした役割は、>>261程度の人物には理解不能だろう。
タキモトは「マインドコントロール説」を垂れ流し、オウムの背後に隠れていた
連中への追及を断ち切るのが、与えられた使命だった。タキモトは、オウムには
北朝鮮も統一協会も創価学会も何ら関係なかったと言っている。その「オウムに
カンケイない」3つの組織がタキモトのスポンサーだよ。
272うきゃ@1年ぶり:02/05/11 23:46
>かくらん順列
とりあえず考えてみました
で,a_1=0, a_2=1, a_(n+2)=(n+1)a_(n+1) +(n+1)a_n
という漸化式までは辿り着いたんですが・・・.
a_6 まで計算しましたが>>264と一致します

これ,結果を予測しての帰納法以外に解き方ないですか?
>>264を予想するのはきついっす
273うきゃ@1年ぶり:02/05/11 23:49
>245
2個記入したとき,はビンゴの確率わかる?
これが分かればその先はスムーズに行けると思うけど・・・.
>>264
40人の順列は40!通り
そのうち1番の人が動かない順列は39!通り。
同様に2〜40番の人が動かない順列もそれぞれ39!通り。
ってことで、40!-39!×40って計算すると0になっちゃう。
それは2人以上が動かない順列をダブって引きすぎたから。
そこで2人が動かない順列を足すということで、
40!-39!×40+38!×C[40,2]
ってやると今度は多すぎ。
3人以上が動かない順列をダブって足したから
以下同様にして…
こうやって最後まで考えた式を整理すると>>264の式になる。
275245:02/05/12 00:11
>>273
6/56...ですか...?
276うきゃ@1年ぶり:02/05/12 00:18
>275
あってるよー.約分して3/28ですね
って,それが分かるなら最後までいけるんでは?

3個書いたときも同じく3/28になります.
書いた3つのうち,どう考えても1つは
選ばれても列がそろうのに意味をなさない場所にいるからね.

4つのときは十字型に並べて,
縦にそろう確率=3/28,横にそろう確率=3/28
これらは排反だから足し合わせればおっけー.

・・・って続けて
277245:02/05/12 00:51
>うきゃ@1年ぶりさん
レスありがとうございます。

4つの場合→3/14
5つの場合→1/8
6つの場合→23/56
7つの場合→39/56
8つの場合→13/14

で、いいのでしょうか…?
正直、数学は中学レベル以下なもので…。(´Д`)
278245:02/05/12 01:01
あ、全然違うくさい…。
もっかい考えます。
279名無しさん:02/05/12 01:04
|a+b|≦|a|+|b|を証明するのに、参考書に(別解で)、

 一般に、-|a|≦a≦|a|,-|b|≦b≦|b|が成り立ち、辺々を加えると、

   -|a|-|b|≦a+b≦|a|+|b| (@)

∴   |a+b|≦|a|+|b|    (A)

と、なっているんですが@からAへはどうやって飛躍したんでしょうか?
>>279
|x|≦y ⇔ -y≦x≦y
281うきゃ@1年ぶり:02/05/12 01:14
>277
5つから違ってる.計算式書いてくれればどこが違うのかわかるんdなけど・・・.
5つの場合は,
●●●
○F○
●○● (●:選ばれた,F:フリー)

Fを通る2列は今まで通り6/56,Fを通らない上の列は1/56
よって13/56.

後は同じようにやってみて.

しかしそろそろ自分の答えに自信がなくなってきたぞ
282279:02/05/12 01:14
>>280
 ワカタ(・∀・)YO!ありがとね!
283245:02/05/12 01:23
>うきゃ@1年ぶりさん
すんません、アホな上にあせってたんで
わけのわからん計算していました。
自分でも何をしたのかわかりません。(;´△`)

5つの場合→13/56
6つの場合→5/14
7つの場合→3/8
8つの場合→1/2

でしょうか??
また違ってたらごめんなさい。
284うきゃ@1年ぶり:02/05/12 02:02
>283
おっけーですよー,100点満点です.
単に僕と答えが一緒ってだけですけど.
285245:02/05/12 02:26
>うきゃ@1年ぶりさん
ありがとうございました!
非常に助かりました。

低脳な質問で長々とスレを汚しました。
みなさんどうもすいません。
286258:02/05/12 07:19
撹乱順列よくわかりました.
本当にありがとうございました.
287132人目の素数さん :02/05/12 10:05
高さ0mで、打ち出す角度が45度で飛び出したボールが150mのホームランになるには、初速度Voは、何km必要か。
ただし、重力加速度は、g=980cm/sec^2である。

この問題、どの式を使ったらいいか教えてください。
288132人目の素数さん:02/05/12 10:42
>>287
どう見ても物理だが。。。
とりあえず斜方投射の3式使っとけ
289132人目の素数さん:02/05/12 10:49
(u×v)'=u'×v+u×v'(u,vは3つの成分で表せるベクトル)
の証明がわかりません、お願いします。
>289
成分で計算しろ。
291一数学者:02/05/12 11:26
 Rij-1/2gijR=-kTij

十元連立方程式でなおかつ非線形微分方程式です。
誰か解いて 
292132人目の素数さん:02/05/12 11:33
>>290
できないのです。。。
お願いします左辺を教えてください。
293132人目の素数さん:02/05/12 11:38
>287
空気抵抗無視してすると軌道は放物線
水平方向には等速で移動する
鉛直方向には重力が作用している

等速の場合はvt=X
力が働いている時はF=ma dv/dt=a
dv/dt=aの両辺をtで積分するとv=at+C ds/dt=at+C
dtで積分するとs=at^2+Ct+C1

計算すると約137km/hになりましたが
「AならばBが成り立つことを証明せよ。」という問題で普通は
A
⇒X
⇒Y
⇒Z
⇒B

のようになるとおもうのですが、
B
⇔X
⇔Y
⇔Z
⇔A

のように証明しても良いのでしょうか?Bと書いた時点で証明すべき内容を
先にかいているからバツになるかなぁと思ったのですが。

同じような例でよくa^2 + ab + b^2 ≧0を証明せよというので
いきなりa^2 + ab + b^2 ≧0から始めてしまって
a^2 + ab + b^2 ≧0⇔
⇔(a + ab/2)^2 + 3/4b^2 ≧0
のように書いてしまって間違いになること同じことかなと思ったのですが。
295132人目の素数さん:02/05/12 11:47

同値変形は間違いではない
それができて、何故反対に書かないのか?
こいつはひねくれ者だ!
ということで実社会では減点対象
数学的には別に問題無いと思うが。
297132人目の素数さん:02/05/12 11:58
>293
どうも、ありがとう。
298132人目の素数さん:02/05/12 12:10
>287
293ですがきずいているとはおもうが一部まちがってました
ds/dt=at+Cを積分するとs=(1/2)at^2+Ct+C1
299132人目の素数さん:02/05/12 12:11
>294
同値の両矢印を書いているのかどうか、それが書いてなければ同値を
分かって変形しているかどうか伝わらないので不可。
で、矢印を書くぐらいなら、反対から書いていったり、=で変形してい
くほうが簡単(>294の例の場合)。
300132人目の素数さん:02/05/12 12:12
>>296
いってることがよくわからんが
同値変形はやりやすい方から書けば良い
論文の場合、いつも上から天下り的に書くが
これは同業者に仕掛けを見せないためだろう
>>281
相対論の解説本よんで喜んでいる厨房か?
Rij、gij、R、k、Tij がそれぞれ何なのか
まず定義を書いてみな。
302296:02/05/12 12:38
>>300
そうですか、よくわかりませんか。
A>Bを証明せよとあって
A>Bと書いてからこれを同値変形するのはバツですよね。
ちゃんとA-Bが0より大きいことを示してからA-B>0と書けますよね。
これはいいですか。

それなら、
「AならばBが成り立つことを証明せよ。」という問題で普通は
A
⇒X
⇒Y
⇒Z
⇒B

のようになるとおもうのですが、
B
⇔X
⇔Y
⇔Z
⇔A

のように証明すれば先ほどの問題と同じように不都合が生じるかと思ったのですが、
言ってることがわかってもらえたでしょうか?
303132人目の素数さん:02/05/12 12:39
そんなことより>>289の左辺の微分は両方の成分について微分するのですか?
304132人目の素数さん:02/05/12 12:44
>>302
「A>Bと書いてからこれを同値変形」しても同値記号を
用いれば全然問題なし
>292
(u×v)の成分表示は?
>303
ハァ?
307132人目の素数さん:02/05/12 13:48
>303
(u×v)計算してから微分するんだから、両方も無いでっしゆ。
308132人目の素数さん:02/05/12 13:58
三角関数で、点Pが、長さ1の線分ABを直径とする
半円周上w動くとき、3AP+4BPの最大値を求めよ。
/P
/
A/________________________B  

ちなみに、角BAPがΘです。

当方は、sinΘ=AP,cos(90-Θ)=sinΘ=BP
これを代入して、3sinΘ+4sinΘ=7sinΘで、
0<=Θ<90より、まで解けたのですが、
この値が出せません。教えて下さい。
>>308
>cos(90-Θ)=sinΘ=BP

勘違いしてる。BP=cosΘ になるはず。
310132人目の素数さん:02/05/12 14:13
高校数学なんですけど
いまいち交代式と対称式ってのがよくわかりません
例を挙げて丁寧に教えてほしいです
お願いします
>310
対称式…任意の2つの文字を入れ替えても式全体が変わらない式
x+y→xとyを入れ替えると→y+xなので元の式と同じ式
xyも同じ

x+y+z→xとyを入れ替えると→y+x+zなので元の式と同じ式
yとzを入れ替えても、zとxを入れ替えても同じ式
xy+yz+zxとかxyzとか(x+y+z)^2なども対称式


交代式…任意の2つの文字を入れ替えたときに、元の式の(-1)倍になる式

x-y→xとyを入れ替えると→y-xこれは元の式の-1倍

(x-y)(y-z)(z-x)
→xとyを入れ替えると→(y-x)(x-z)(z-y)これも元の式の-1倍
yとzを入れ替えたときも、zとxを入れ替えた時も同じく-1倍になる

因みに、(交代式)x(交代式)のようなかけ算をすると
文字を入れ替えたときに元の式の(-1)倍x(-1)倍=1倍なので
対称式になる。

対称式でも交代式でもないもの
例えば
xy+x→xとyを入れ替えると→yx+y これは元の式の定数倍ですらない
312132人目の素数さん:02/05/12 14:27
>308
k(θ)=3cosθ+4sinθ
k'(θ)=-3sinθ+4cosθ=0
cosθは0でないので-3sinθ/cosθ+4=0
θ=Tan-1(4/3)
313132人目の素数さん:02/05/12 14:34
>>305
u2v3i+u3v1j+u1v2k-(v2u3i+u1v3j+u2v1k)
vについてとuについて微分するんですね、もうわかりましたからいいです。
314132人目の素数さん:02/05/12 14:37
>>311
えっと交代式は
(x+y)(x+z)ならこのときの交代式は
(y+x)(y+z)ですか?
315314:02/05/12 14:38
ごめんなさい
補足読んでませんでした
>>311の説明でよくわかりました
どうもありがとうございます
316132人目の素数さん:02/05/12 15:00
YOKOHAMAという語の8文字を全て並べてできる順列について
(1)順列の総数を求めよ
(2)AAという並びとOOという並びをともに含む順列はいくつあるか
(3)AOという並び、またはOAという並びを少なくとも一方を含む順列はいくつあるか

という問題なのですが、(1)(2)はわかったのですが[解:(1)10080 (2)720]、
(3)はどうすれば解けるのかわかりません。答は7440なのですが...
よろしくおねがいします。
10080-4!*(5C4*4!/2!/2!+2*5C3*3!/2!+5C2*2)=7440
318132人目の素数さん:02/05/12 15:23
(1)成功する確率は6%
(2)25回挑戦できる
(3)4回以上成功する確率は何%か?

という問題です、よろしくお願いします
319132人目の素数さん:02/05/12 15:29
環Q[t+1/t]とQ[t]は環として同型なんでしょうか?
それと、こういう場合はありますか?
「環R、R’について R⊃R’(R≠R’)でかつ、R
とR’は環として同型 」
320132人目の素数さん:02/05/12 15:30
ベクトルの問題です。
A(s),B(s):ベクトルとする。

(A(s)・B(s))'=(A(s)'・B(s))+(A(s)'・B(s))

なるのはどうしてですか??
内積微分がよくわからなくて。
321316:02/05/12 15:31
>>317
余事象を考えて、
○□○□○□○□○
□にはYKHMのいずれかが入る。4!
(i)○に[A][A][O][O]がはいるとき
  5つの○のうち、文字が入る4つを選び、入るもの4つの順列。
  5C4*4!/2!/2!
(ii)○に[AA][O][O]または[A][A][OO]がはいるとき
  5つの○のうち、文字が入る3つを選び、入るもの3つの順列。
  2*5C3*3!/2!
(iii)○に[AA][OO]がはいるとき
  5つの○のうち、文字が入る2つを選び、入るもの2つの順列。
  5C2*2
ということでいいのでしょうか。
納得できました。ありがとうございました。
322132人目の素数さん:02/05/12 15:33
>>320
もろ概出
323132人目の素数さん:02/05/12 15:40
半径rの円板を3つ用いて、
一辺1の正三角形を覆う。
これが可能であるようなrの最小値を求めよ。

という問題なんですが。
おそらく答えは(√3)/3 だと思うのですが
(三角形の頂点をA,B,C、重心をGとして
 AG、BG、CGを直径とする円板をとる)
これが最小だということを
どのように説明するといいでしょうか?
324323:02/05/12 15:42
>>323(自己レス)

>おそらく答えは(√3)/3 だと思うのですが
答えは(√3)/6 だと思うのですが、
の間違いです。スミマセン。
(直径と半径を勘違いしてました)
325132人目の素数さん:02/05/12 15:55
322>すみませんでした。
最後に質問ですが、既出のもののを検索する場合どうすればよいのですか?
326308:02/05/12 15:56
k(θ)=3cosθ+4sinθ
k'(θ)=-3sinθ+4cosθ=0←ここで、なんで0が出てくるのですか?
cosθは0でないので-3sinθ/cosθ+4=0
θ=Tan-1(4/3)
↑これから、値は出せますか?

教えて下さい。
327132人目の素数さん:02/05/12 16:20
>>325
Ctrl+Fで検索したい言葉を入れればよし
328132人目の素数さん:02/05/12 16:30
>308
θを使わなければいけないの?
AP=x,BP=y とおくと
x^2+y^2=1 (x>0,y>0)
このとき3x+4y の最大値を求めよ。
という問題で
3x+4y=k とおくとy=(k−3x)/4
これを x^2+y^2=1 に代入して整理して、判別式の計算

k(θ)=3cosθ+4sinθ でやるならば
合成して√(3^2+4^2)sin(θ+α)=5sin(θ+α) 最大値は5
ってやるのが簡単 角の吟味は残るけど
329 :02/05/12 16:37
3点O(0,0,0)、B(1、−2,0)、C(2,0,1)がある。
Oから線分ABに下ろした垂線の足Hの座標を求めよ

3点A(1,1,0)、B(1,4,2)、C(2,3,1)を通る平面と
2点D(1、−1,0)、E(2,0,ー1)を通る直線の好転の座標を求めよ

正四面体OABCについて点Oから△ABCに下ろした垂線の足Hとするとき、ベクトルOH=pベクトルOA+qベクトルOCとなる定数p,q,rの値を求めよ。
また、正四面体の1編の長さを1とするとき、OHの長さを求めよ。


ベクトルを使って教えてください
330329:02/05/12 16:38
3点A(1,1,0)、B(1,4,2)、C(2,3,1)を通る平面と
2点D(1、−1,0)、E(2,0,ー1)を通る直線の交点の座標を求めよ

331 :02/05/12 16:47
四面体OABCにおいて辺OA,BCの中点をそれぞれM,Nとし、辺AB,OCを1対3に内分する。このとき、直線MNとPQが1点で交わることを証明せよ

四面体OABCにおいてベクトルOP=tベクトルOA+mベクトルOB+nベクトルOCとする。
このとき、点Pが△ABCの内部またはその集にあるときl,m,nの満たす条件はl≧1、m≧1n≧2、l+m+n=1であることを示せ


ベクトルを使って教えてください
>>329>>330>>331
マルチポストには誰も教えてくれねぇよ。
>>323
それより小さい r をとって背理法。
A, B, C, G の4点に着目すればいい。
どの2点間の距離も 2r より大きいわけだから、
ひとつの円板に2つの点が入ることはない。
ということは、覆うためには4つの円板がいる。
>>319
前半は同型。
後半は、下の783-784 が参考になるんじゃない?

http://cheese.2ch.net/math/kako/1009/10091/1009102965.html
335308:02/05/12 17:13
<<328
ありがとうございますした。

角度は求めるのは不可能ですか?
336132人目の素数さん:02/05/12 17:13
>308
k(θ)は上に凸だ。
だからk'(θ)=0の点で最大値

θ=Tan-1(4/3)で直角三角形で3 4 とくれば次は5だ。
だから比例で各辺の長さは3:4:5=3/5:4/5:1
3AP+4BP=3*3/5+4*4/5=5

>328
なんとなく微分の問題のような木がしたもんで。
337132人目の素数さん:02/05/12 17:36
明日自分が当たるのですが、全くわかりません(汗
誰か親切な方、教えていただけると嬉しいです。m(__)m


x軸上を原点から出発し、さいころを投げて偶数の目が出ると右へ1、
奇数の目が出ると左へ1進む点Pがある。さいころを5回投げるとき、
点Pの最後の位置のx座標をXとして、次の問いに答えよ。

(1)Xのとり得る値を求めよ。
(2)Xの確率分布を求めよ。
(3)Xの平均と標準偏差を求めよ。

(1)と(2)の解き方がわかれば(3)は出せそうなんですが・・・
ドキュソな質問かと思いますが助けてください(涙
338316:02/05/12 17:47
>>337
(1)
X=-5,-3,-1,1,3,5

(2)
-5:1/32 -3:5/32 -1:5/16 1:5/16 3:5/32 5:1/32
339302:02/05/12 18:28
>>304
そうなんですね。今まで勘違いしていました。間違いを直せて良かったです。
どうもありがとう!
340329:02/05/12 18:32
ごめんなさい。
もう2度としないので教えてください
わからなくてこまってます
お願いします
341132人目の素数さん:02/05/12 18:35
>335
tanθ=4/3だから三角比の表を見るか、関数電卓を使うかでしょうか。
342132人目の素数さん:02/05/12 18:39
2進法で178って出せる?
343132人目の素数さん:02/05/12 18:41
lim[x→∞]{1+(1/x)}^x=eを認めてlim[x→∞]{1+(1/x)}^x=eを証明せよ。
ヒント:x=-zと置くと{1+(1/x)}^x={1+(1/(z-1))}^z,lim[z→∞]{1+(1/z-1)}=1
ヒントの意味はわかるのですがどのように使うのかわかりません
非常に見にくい式かと思いますがお願いします。
>343
前半と後半が同じになってるぞ
書き間違い?
345132人目の素数さん:02/05/12 18:53
いえ、このように問題がかいてあるのです…
346132人目の素数さん:02/05/12 18:55
すいません!!
lim[x→∞]{1+(1/x)}^x=eを認めてlim[x→−∞]{1+(1/x)}^x=eを証明せよ。
マイナスを忘れてました、、申し訳ありません。。
>>342
だせる
348318:02/05/12 19:11
>>318
お願いしていいでしょうか…
>>343
「x=-zと置くと{1+(1/x)}^x={1+(1/(z-1))}^z 」って所まではわかるわけ? 
>>318
(4回以上成功する確率)=1−(22回失敗)−(23回失敗)−(24回失敗)−(25回失敗)
351323:02/05/12 19:34
>>333
ありがとうございました。
わからない問題がないんですが何を書けばいいでしょうか?
>352
質問に答えてやれ
354132人目の素数さん:02/05/12 19:51
>>349
はい、そこはわかりました。そこからがどうすればいいのか
見当がつきません。
>>354
{1+(1/(z-1))}^z={1+(1/(z-1))}^(z-1)*{1+(1/(z-1))}

これでわかるんじゃないかな?
356132人目の素数さん:02/05/12 20:09
たのむ3x^3-10x^2+3x+10を因数分解してくれ。
357132人目の素数さん:02/05/12 20:14
X^4+Y^4
を因数分解したいんですがどうすればいいでしょうか?
358132人目の素数さん:02/05/12 20:31
>>355
その式をz→∞に飛ばせばeになるのですか?
359323:02/05/12 20:37
>>358
z-1 = t とおいてt→∞としてみそ。
360132人目の素数さん:02/05/12 20:40
>>359
なるほど、よくわかりました、本当にありがとうございました。
361  :02/05/12 20:42
3点O(0,0,0)、B(1、−2,0)、C(2,0,1)がある。
Oから線分ABに下ろした垂線の足Hの座標を求めよ

3点A(1,1,0)、B(1,4,2)、C(2,3,1)を通る平面と
2点D(1、−1,0)、E(2,0,ー1)を通る直線の好転の座標を求めよ

正四面体OABCについて点Oから△ABCに下ろした垂線の足Hとするとき、ベクトルOH=pベクトルOA+qベクトルOCとなる定数p,q,rの値を求めよ。
また、正四面体の1編の長さを1とするとき、OHの長さを求めよ。

四面体OABCにおいて辺OA,BCの中点をそれぞれM,Nとし、辺AB,OCを1対3に内分する。このとき、直線MNとPQが1点で交わることを証明せよ

四面体OABCにおいてベクトルOP=tベクトルOA+mベクトルOB+nベクトルOCとする。
このとき、点Pが△ABCの内部またはその集にあるときl,m,nの満たす条件はl≧1、m≧1n≧2、l+m+n=1であることを示せ



ベクトルを使って教えてください。お願いします
362132人目の素数さん:02/05/12 20:43
でも勝手にt→∞にしてよろしいのでしょうか?
363132人目の素数さん:02/05/12 20:43
>>361
マルチ・リターンズ
>329-332
>361
マルチどころが同一スレだよ。おい。
365361:02/05/12 20:47
>>363,364
さっきのことは反省していて、もうそんなこと絶対しないのでどうか、おねがいします。
何度やってもできなかったので。
366318:02/05/12 20:51
>>350
25回失敗は0.94の25乗=0.21と分かりますが、他が…
367365:02/05/12 20:53
3点O(0,0,0)、B(1、−2,0)、C(2,0,1)がある。
Oから線分ABに下ろした垂線の足Hの座標を求めよ

3点A(1,1,0)、B(1,4,2)、C(2,3,1)を通る平面と
2点D(1、−1,0)、E(2,0,ー1)を通る直線の好転の座標を求めよ

正四面体OABCについて点Oから△ABCに下ろした垂線の足Hとするとき、ベクトルOH=pベクトルOA+qベクトルOCとなる定数p,q,rの値を求めよ。
また、正四面体の1編の長さを1とするとき、OHの長さを求めよ。

四面体OABCにおいて辺OA,BCの中点をそれぞれM,Nとし、辺AB,OCを1対3に内分する。このとき、直線MNとPQが1点で交わることを証明せよ

四面体OABCにおいてベクトルOP=tベクトルOA+mベクトルOB+nベクトルOCとする。
このとき、点Pが△ABCの内部またはその集にあるときl,m,nの満たす条件はl≧1、m≧1n≧2、l+m+n=1であることを示せ



ベクトルを使って教えてください。お願いします
368132人目の素数さん:02/05/12 20:54
>>366
25回のうち、1回が成功、24回が失敗
→25c1*(6/100)^1*(94/100)^24
369スレを汚すな:02/05/12 20:55
>>367
さっきのことは反省していて、もうそんなこと絶対しないのでどうか、おねがいします。
何度やってもできなかったので。

さっきのことは反省していて、もうそんなこと絶対しないのでどうか、おねがいします。
何度やってもできなかったので。

さっきのことは反省していて、もうそんなこと絶対しないのでどうか、おねがいします。
何度やってもできなかったので。

さっきのことは反省していて、もうそんなこと絶対しないのでどうか、おねがいします。
何度やってもできなかったので。

さっきのことは反省していて、もうそんなこと絶対しないのでどうか、おねがいします。
何度やってもできなかったので。

ベクトルを使って反省してください。お願いします。
370マジレス:02/05/12 20:55
>367
そんなことするから誰も教えてくれねーんだよ。
>365 読んで教えてあげようと思ったヤツだって
>367 の時点で止めるに決まってるだろ?

愉快犯じゃないんだったら、ちゃんとしろよ。
371132人目の素数さん:02/05/12 20:58
△:X→X×X;x→(x,x)を対称線写像という
A⊂Xの時
△(A)=△A (△Aは対称線集合)
を示せ。
おねがいします
>371
示せって、それが対称線集合(対角線集合と呼びたいが・・・)の定義じゃないの?
そうじゃなかったら、取りあえず、対称線集合の定義教えて。
373371:02/05/12 21:01
三行目の()の中は
×対称線集合→○対角線集合
でした
374132人目の素数さん:02/05/12 21:03
くだらねぇのスレにも書きましたが、
どなたか分かる方がいましたらお願いします。
次の証明問題を解きたいのです。

Gkはガウス関数とする。
Gk(x)= 1/(2πk) exp(-x^2/k^2)

Gkと関数f(x)との畳み込み積分を次の様に表す。
Gk(x)○f(x)=∫[-∞,∞] Gk(x') f(x-x') dx'

このとき、

Ga○Gb○f=G(a+b)○f

となることを証明せよ。同時に、

Ga○ΔGb○f=ΔG(a+b)○f

が成り立つことも証明せよ。
(Δはラプラシアン)

という問題を解きたいのですが、うまく出来ません。
どなたかご回答願います。
(x+y+z)^3は何ですか?手元の本には載ってないのでこの公式を教えてください。
どこにも載っていませんでした。
>>375
何でもかんでも公式になってると思うな。
>375
(a+b)^3 (★) の計算の仕方は分かるよな?
だったら、
(x+y+z)^3 = ((x+y)+z)^3 を(★) を使って計算したあと、
括弧をはずす。
378132人目の素数さん:02/05/12 21:09
A+B+C=180°のとき
sin2A+sin2B+sin2C=4sinAsinBsinCを証明せよ。

わからないのでどなたかお願いします。
379132人目の素数さん:02/05/12 21:10
あまりにも初歩的で申し訳ないのですが
2:3に外分するってどうやってやるのですか?
外分の意味と解き方を教えてください。
380132人目の素数さん:02/05/12 21:13
以下の問題、お願いします。

自然数から自然数への関数fで

f(f(n)) = n + 1987 (nは自然数)

をみたすものは存在しないことを示せ。

どこから手をつけていいかさっぱりわかりません。
高校生にもわかるように説明していただけませんか。
381132人目の素数さん:02/05/12 21:18
以下の問題お願いします。

f:X→Y を写像とするとき、以下を証明せよ。

(1) f'(f(X))⊃ X

(2) f(f'(Y))⊂ Y
「'」は「-1(インバース)」の意です。

よろしくお願いします。
>>374
Ga○Gb=G(a+b) は、たんなる積分の計算問題。
これが出来れば、他はほとんど自明だけど。

ガウス関数でなくても、一般に、
F(x)○G(x)=∫[-∞,∞] F(t)*G(x-t)dt
で畳み込みを定義すると、

(1) F(x)○G(x)=G(x)○F(x)
(2) {F(x)○G(x)}○H(x)=F(x)○{G(x)○H(x)}

になることは知っている?
(1) はどうでもいいけど、(2) を知らないと、
Ga○Gb○f の意味が不明だよ。
>380
手が付けられるようにヒントだけ。
示すべきことは次の二つ。

・ m ≡ n (mod 1987) ⇔ f(m) ≡ f(n) (mod 1987)
・ 任意の自然数 n について、 f(n) と n は mod 1987 で異なる。
384132人目の素数さん:02/05/12 21:26
>>382
ご回答ありがとう御座います。
ヒントを頼りにがんばってみます!
385132人目の素数さん:02/05/12 21:29
4*4のマスに1〜16までの数字を入れ、縦、横、斜めの和が
等しくなるようにしなさい。
この問題の答え誰か教えてください。
386132人目の素数さん:02/05/12 21:31
誰か>>357の問題お願いします
387ゆう:02/05/12 21:33
中学校3年生で習う平方根の問題なんですがどうもわからないんです。一応自分は中3なんですけど・・
√54−√12÷√2この問題のとき方が説明できる人いたらよろしくお願いします。
388132人目の素数さん:02/05/12 21:34
X + Y=LOVE です
>>357
x^4+y^4
=x^4+2*x^2*y^2+y^4-2*(x*y)^2
=(x^2+y^2)^2-(√2*x*y)^2
=(x^2+y^2+√2*x*y)(x^2+y^2-√2*x*y)

>>385
http://www.hitoyoshi.net/tokumasa/mahoujin.html
>386
係数体をはっきりしてくれないとねぇ。
複素係数なら、
x^4+y^4
= (x-ω y)(x-ω^3 y)(x-ω^5 y)(x-ω^7 y)
( ω= (1+i)/√2 )

実数係数なら
x^4+y^4
= ( (x-ω y)(x-ω^7 y) ) ( (x-ω^3 y)(x-ω^5 y) )
= (x^2 - √2 xy + y^2) (x^2 + √2 xy + y^2)

有理数係数なら、既約。
391361:02/05/12 21:40
>>370
367は自分じゃないんだけど・・・
392ゆう:02/05/12 21:42
すいません、361さんと自分は別人です。
393380:02/05/12 21:42
>>383

すいません

m ≡ n (mod 1987)

これの意味って、mとnは合同???
1987で割った余りが同じってことですか???
>>387
√54=3√6

√12÷√2=√6
>393
これの意味って、mとnは合同???
そういうこと

1987で割った余りが同じってことですか???
そういうこと
396385:02/05/12 21:45
結局、学校の問題ばっかりで頭を使う問題を解ける奴はいないのか・・・。
397132人目の素数さん:02/05/12 21:45
あの質問なんですけど
出発点から東へ1Km,南西に10Km,北に5Kmにある終了地点まで移動しました。
出発点から終了地点までの変位ベクトルの大きさと向きを求めたいのですが、
教えてください!!

>396
もう答えでてるぞ
399132人目の素数さん:02/05/12 21:47
>>389-390
thanks!
400132人目の素数さん:02/05/12 21:48
お願いします。
f:X→Y g:Y→Z に対して以下を示せ。

・gofが全射ならばgは単射である

・gofが全射でgが単射ならばfは全射
401132人目の素数さん:02/05/12 21:49
4x4の魔方陣の話なんか誰も見向きもしないよ > 396
gofが全射ならばgは単射である
そんなバカな
>>391
そうなの?
教えてやるからもうレスやめろ。
でも、ベクトルでのやり方は説明できん(よくわからん)
誰か、やってやれ
404132人目の素数さん:02/05/12 21:50
>>387
√54−√12÷√2

√54=3√6。
√12=2√3。

 √12/√2=2√3/√2 ※これを有理化する
=2√3*√2/√2*√2
=2√6/2

これで、あとは四則計算ですね。(^^
405132人目の素数さん:02/05/12 21:50
>>397
答えは出発点によるな
406132人目の素数さん:02/05/12 21:52
>>400
golf は単車でなく四輪車ですよ
407132人目の素数さん:02/05/12 21:52
>>402
失礼しました。
単射→全射

よろしくお願いします
>>407
gof が全射 ⇔ ∀z∈Z ∃x∈X (gof)(x)=z
⇔ ∀z∈Z ∃x∈X g(f(x))=z

ここでy=f(x)とおけば、
∀z∈Z ∃y∈Y g(y)=z■
409132人目の素数さん:02/05/12 22:12
>>408
ありがとうございました。
410132人目の素数さん:02/05/12 22:26
誰か>>381の問題お願いします。
411132人目の素数さん:02/05/12 22:29
>361,>403
いっぺんにこれだけ問題書かれてもやる気起きないよ。やればできそうと思えば
余計に。
しかもところどころ問題写し間違ってるし。他の人に任せます。
412ゆう:02/05/12 22:33
ありがとうございました132人目の素数さん。お手数をおかけしました。
413132人目の素数さん:02/05/12 22:35
帰納的定義の漸化式について教えて下さい
a[n+1]=pa[n]+q (q≠1) 、α=pα+qとすると
辺々を引いて
a[n+1]-α=p(a[n]-α)

とあるのですが、α=pα+qはなんなのでしょうか。
突然出てきて辺々を引いてといわれても納得いきません。
何故このように計算するとa[n]が求まるのかも理解できません。

漸化式から回帰公式というのを見つけて調べたのですが
積分が出てきて、理解できませんでした。
微分積分を知らないリア工ですが教えて下さい
414132人目の素数さん:02/05/12 22:40
x^2-y^2+2y-1を因数分解すると
(x+y-1)(x-y+1)
になるそうなのですがなぜこうなるのかわかりません
もしよろしければどなたか教えてください。
415132人目の素数さん:02/05/12 22:43
ここんとこ、回答するのが嫌になるような問題が多いね
416132人目の素数さん:02/05/12 22:46
>>414
x^2-(y^2-2y+1)
=x^2-(y-1)^2
=(x+(y-1))(x-(y-1))
=(x+y-1)(x-y+1)
今仕事上必要になって考えていることです。
考え方のヒントだけでも頂ければ幸いです。

2行2列の行列による一次変換について。
任意の一次変換Aについて回転Bの成分を取り除いた変換Cを求めよ。
つまりもっと厳密に書くと
A=BC
C=DE(なる行列Eは存在しない)
ただしB,Dは回転行列
A=[[a,b][c,d]],B=[[e,f][-f,e]](ee+ff=1)
以上を満たすCをa,b,c,d,e,fを使って表せ。
>>381
Xは定義域全体になっているようだが、それでいいのか?
定義域の部分集合じゃないのか?まあいいや。

「Xをfの定義域とすると、」

(1) f'(f(X)) = { z | f(z)∈f(X)} = { z | z∈X} = X

(2) f(f'(Y)) = f ({ z | f(z)∈Y}) = f(X) ⊂ Y

419417:02/05/12 22:56
無知な私には単に計算するしか求める方法がないようです。
計算すれば求まりそうなので、質問を取り下げます。
失礼しました。
>>417
仕事というのであれば、基本的に無料という訳にはいかないと
思います。
421417:02/05/12 23:01
>>420 そんな殺生な(笑い
>>413
a[n+1]=pa[n]+q (*)
a[n+1]-α=p(a[n]-α) (**)

(*)はそのままでは解けないが、(**)の形なら解ける。
従って、何とかして(*)を(**)に変形したい。

そこでとりあえず(**)のように置いてみて、
それを展開整理して(*)と係数を比較してみる。
するとαはα=pα+qを満たしていることがわかる。
423132人目の素数さん:02/05/12 23:09
>413
その数列が「収束するなら」 極限値をαとすると
n→∞のとき a(n)→α,また a(n+1)→α
と考えるとどうしてその式に気が付いたかが少しは分かるだろうか
424375:02/05/12 23:13
>>377
ありがとうございます。
公式はやはりないんですね。ところで、(x+y+z)^3の公式がないということは
あまり使わないと言うことですよね?ということは、(x+y+z)^3の展開を考えてる
時点で間違った解法に向かっていると言うことでしょうか?377さんはこれを
展開して問題が解けた経験はありますか?
425132人目の素数さん:02/05/12 23:15
>381
こういうのって f'(f(x))=x を使って良いんだろうか。だったらもう
証明するまで無く明らかと思うんだが。
(1)Xの任意の要素x∈X に対してその原像xが存在してf'(f(x))=x
だからf’(f(X))∋x,f’(f(X))⊃X
(2)任意のy∈Y に対してf(f'(y))=y∈Y よってf(f'(Y))⊂Y

何かとんでもない勘違いをしているんだろうか
426132人目の素数さん:02/05/12 23:16
>425
すでに解答済みみたい。失礼しました。
361がまたレスしてきそう・・・
ちなみにオレは厨房なんで全然わからん。
野郎が荒らす前に教えといてやんないとこのままどんどんスレ消費してくぞ
まいったなこりゃ。
429380:02/05/12 23:18
>383
ヒントありがとうございます。
まだ、このヒントをどう扱ったらよいかわかりませんが
もうちょっと、頑張ってみます。

modなんて高校で出てこないぞ。。。
EXCELの関数やってなかったらわからんとこでした。
シンプルそうなのにむちゃくちゃ難しい問題だ。。。
>>424
>公式はやはりないんですね。

ないなら自分で作ればいいだけ。

>公式がないということは、間違った解法に向かっている?

君は、(1+2+3)^2 は公式がないから求められないというのか?
431413:02/05/12 23:35
>>422
>>423
ありがとうございます。
完全ではありませんが、何となく分かりました。
>424
(x+y+z)^3の展開公式はあるぞ。
当然ながら。
433132人目の素数さん:02/05/12 23:51
>>424
そんな公式、覚えてる暇があったら計算しる
434132人目の素数さん:02/05/13 00:00
f:R→R’を環準同型としてMをRの空でない部分集合とするとき
M’=f(M)ならばf^-1(M’)=M+kerfであることを
示すにはどうしたらよいのでしょうか?
435132人目の素数さん:02/05/13 00:08
集合Aと集合Bが等しい、即ちA=Bを示すには
A⊂B かつ B⊂A を示せば良い。

つーか集合の教科書にA=Bの定義が書いてあるべ。
教科書読めや。
聞きたいことがここじゃなかったら群、環の所をちゃんと復習しる。
436新人です宜しく:02/05/13 00:27
星印を書いてください。一筆書きで書ける、真ん中に五角形のある、
よくあるタイプの星印です。
さて、この星印には、三角形が5個あります。ふたつ以上の図形を
合わせて大きな三角形があるっていうのは無しです。
この星印に、直線を2本引いて、三角形を合計10個にしてください。
437132人目の素数さん:02/05/13 00:34
角XOYの内角に任意の点aを設ける。
三角形abc野周りの長さが最短になるように、線XO上に点b線YP上に
点cをかき入れよ。

中一の姪から宿題が分からないと電話がかかってきました。
線対称の応用だと思うのですが、線対称は次の単元でまだ習って
いないと言い張るのです。
平行線と垂線だけで解ける問題なのでしょうか?
>436
とりあえず、星形とか考えずに7本の直線で10個の三角形を作ってみて
その図をじっと眺めると、星がどこかにある…
>>430君は、(1+2+3)^2 は公式がないから求められないというのか?
そういう話ではないんです。計算にまかせて(x+y+z)^3を展開しても解ける
問題はほとんど無いのではないかと。どつぼにはまるんじゃないかと。
430さんは具体的な数字でなくて(x+y+z)^3のような文字式の式を展開した
ことはありますか?展開して解ける問題に出会ったことがありますか?

>>432
よろしければそれ教えてください。
>437
Pって何?
441132人目の素数さん:02/05/13 00:41
実数を3乗すればその3つの解はすべて実数になるのですか?
例えばx(実数)があってxを3乗したx^3の解はありさまはどうなりますか?
442132人目の素数さん:02/05/13 00:43
>440
ごめんなさい
Oです
443132人目の素数さん:02/05/13 00:44
>>441日本語大丈夫ですか?
444132人目の素数さん :02/05/13 00:45
よろしくお願いします。

f:X→Y を写像、Xi⊂X、Yi⊂Yとするとき以下を証明せよ。

(1)f(X−X1)⊃f(X)−f(X1)

(2)f'(Y−Y1)=X−f'(Y1)  
   「'」は「-1(インバース)」の意です。

お願いします。
>>439
公式がないからって展開できないわけでもなかろう。
展開公式の有無で解ける解けないが変わる問題もないだろう。
>>444
有料となります。
>>439
いってることはわかるが、答えようがないよ。
めったに出てこない式だが、絶対に出てこないともいいきれん。
公式の定義:
1)誰もが公式と認めるものは公式である。
2)公式を使って出てくる式は、実はすべて公式である。
3)これ以外を公式と呼んではならない。
>439
有名な公式なんだけど…
言いたいことがよくわからない…
(x+y+z)^3程度の展開をして解けない
問題ってどんなの?
>448
その公式の定義によれば
(x+y+z)^3の展開式は確かに公式ですが?
451うきゃ@1年ぶり:02/05/13 00:57
>439
例えば,小学生くらいの人が君にこんな質問をしてきたらどうする?
「学校で3桁×3桁の掛け算しかならわないんだけど,
それより大きい数の掛け算ってやらないの?
使うことが滅多にないから習わないんですか?」
※小学校で習うのは3桁×3桁の掛け算まで・・・だったと思う

答え:何桁でも掛け算は必要.それ以上は3桁の応用でやれ

つまり3乗でも4乗でも展開はやる.2乗の展開の応用でやれ,っていうこと
もっと先に進めばn乗の展開とかもやります
定理:すべての公式を記憶する手段は存在しない。
定理:その式が公式であるかどうかを判定する為には、その公式を
(一時的にでも)記憶することが必要である。
>437
例えばさ、bとcのいずれか一方を固定しておいたら
もう一方はどんな点をとればいいと思う?
455132人目の素数さん:02/05/13 01:24
>454
bを固定するとして、線OYを中心として線対称の位置にaダッシュ
で線OYと線aダッシュbの交わる点がcだと思うのですが。
456132人目の素数さん:02/05/13 01:40
どなたかお願いします。
元の問題が英語なので、多少和訳に不自然なところがあるかもしれませんが…
Aは行列、xはベクトルです。

A=A(z)と仮定すると(zはスカラー)、∂(x'Ax)/∂z は何を表すか。
また、xのそれぞれの要素がzの関数だとすると、∂(x'Ax)/∂z は何を表すか。

まったくお手上げなのですが…
最初のA=A(z)ってところからどういうことかよく分かりません。
英語が読めないだけとみた。
458132人目の素数さん:02/05/13 01:48
数論なんかでよく出てくる「法」はなんと読むのですか?
「ほう」?
>>457
では原文を。

Suppose that A = A(z), where z is a scalar. What is ∂(x'Ax)/∂z?
Now,suppose that each element of x is also a function of z.
Once again, what is ∂(x'Ax)/∂z? 
>>459
前後の文脈がないと少しきつい。
本・章のタイトルは?
>>458
ほうだよ。
462456:02/05/13 01:55
本はGreenの…何だっけな。
章のタイトルは
Matrix Algebra
です。
x'って何?>456
>>461
ほうですか。
もう少し具体的に言うと、
a≡b (mod n)とかの法のことです。
465132人目の素数さん:02/05/13 02:00
>464
法と書いてダルマと読みます
466456:02/05/13 02:01
>>463
列ベクトルxを転置した、行ベクトル。
(x'Ax)はxの2次形式で、係数行列A、変数xの要素を全てzの関数だと思って
微分しただけじゃん?
本の名前、文脈から察するに計算せよ
と翻訳するのが妥当であると思われます。

参考:(x+1)^2って何? (x+y)^2-(x-y)^2って何?
469456:02/05/13 02:12
>>467
そこまで進む前に、最初の
Suppose that A = A(z), where z is a scalar.
ってのがどういうことなんだかピンとこないんです。
スカラー、っていう概念もよく分からない。
470456:02/05/13 02:14
>>468
なるほど。難しく考えすぎだったのかな。
訳(というか意味)だけ。
Suppose that A = A(z), where z is a scalar. What is ∂(x'Ax)/∂z?
x が 定ベクトルで A が z に依存する行列の時
∂(x'Ax)/∂z はいくつか?

Now,suppose that each element of x is also a function of z.
Once again, what is ∂(x'Ax)/∂z? 
A は z に依存する行列であり、
x も z に依存するとき、
∂(x'Ax)/∂z はいくつか?
472456:02/05/13 02:18
>>471
要するに、Aの要素はzの関数ってことですか?
で次のやつはxの要素もzの関数と。

これでいいのかな。
473471:02/05/13 02:20
>472
そゆこと
474456:02/05/13 02:21
サンクス。
どうにか糸口がつかめました。
475132人目の素数さん:02/05/13 02:21
>437
未知数2個
条件1個でどうやって解くのですか?
>>443
おっしゃってる意味がよくわからないのですが・・・。

どなたかお願いします。

実数xを3乗したx^3の3つの解は全て実数なのですか?
また、そうだとすればなぜそうなるのですか?
+123と-684を16ビットの固定少数点形式で2進数の表記と16進数の表記で表せ

誰かおしえてくださーいm(。_。;))m
>>476
質問の意味がわからないですね。
たとえば、x=2 としましょう。x^3=2^3=8 ですよね。
「x^3の3つの解は全て実数なのですか」という質問は、
そのまま読めば、「8の3つの解はすべて実数ですか」
になって、わけわかりません。

x^3=8 の解のことを言いたいんですか?
479476:02/05/13 05:49
>>478
すいません、やはり日本語おかしかったですね。

>x^3=8 の解のことを言いたいんですか?

その通りです。よろしくお願いします。
>>479
要するに、a を実数とするとき、x^3=a^3 の解ですね。
x^3-a^3=0 を因数分解すれば、(x-a)(x^2+ax+a^2)=0 です。
x-a=0 からは、x=a と元の数がでてきますが、
x^2+ax+a^2=0 を2次方程式の解の公式でとくと、

x=1/2*{-1±√(-3)}a

となって、虚数になります。
つまり、実数解は1つだけで、他の2つは虚数ですよ。
481479:02/05/13 06:19
>>480
すごくわかりやすかったです!!
思い違いをしておりました。きちんと考えれば実数解は1つだけで、
他の2つは虚数になることがわかりますね。

ありがとうございます!感謝です!!
>>445>>447>>449>>451
(x+y+z)^3についてなんですがこんな問題がありました。

x,y,z,aについて
x+y+z=a・・・@
x^3 + y^3 + z^3 =a^3・・・A
の2式が成立するときx,y,zの少なくとも一つはaに等しいことを証明せよ。

@Aの右辺に注目して@の左辺を3乗した式がAの左辺と等しいと思ったのですが、
解答見ると全然別の解法でした。

この教訓から「(x+y+z)^3を計算しようなどとせず別のうまい手を考えなければ
ならない⇒(x+y+z)^3を見てもまず展開しないと思った方がいい」という結論に
なってのですが、どうですか?こう考える方が実践的だと思うのですが。

>つまり3乗でも4乗でも展開はやる.2乗の展開の応用でやれ,っていうこと
もっと先に進めばn乗の展開とかもやります

わたしも以前は同じように考えてたんですが、むしろこういう考えが逆に
問題を解けなくしていないでしょうか?



483132人目の素数さん:02/05/13 06:50
>@Aの右辺に注目して@の左辺を3乗した式がAの左辺と等しいと思ったのですが、
>解答見ると全然別の解法でした。
で、君はそのやり方で解けたの?解けなかったの?
別の解法を見てどう思った?
「解けたからどうでもいいや」と思った?「うまいな」と思った?
自分のやり方で解いてないから自分の解き方はダメ?
模範解答が最高の解き方なの?

>この教訓から「(x+y+z)^3を計算しようなどとせず別のうまい手を考えなければ
>ならない⇒(x+y+z)^3を見てもまず展開しないと思った方がいい」という結論に
>なってのですが、どうですか?こう考える方が実践的だと思うのですが。
で、君はそれを実践できるの?出来ないなら自分のやり方でやった方がよくないかい?

私の質問に答える必要はないよ。
自分で考えて自分なりの答えを見つけたらいい。
大事なのは自分の頭で考える事。
>482
その解答がどういう解法なのかは知らないけど
展開しても十分解ける

x+y+z=aより
x,y,zの内少なくとも一つがaならば残りの2つの和は0
x=a⇔y+z=0

a^3=(x+y+z)^3
= x^3 + y^3 + z^3 + 3 x^2 (y+ z) +3 x (y+z)^2 + 3 yz(y+z)
= a^3 + 3 x^2 (y+z) +3 x (y+z)^2 + 3 yz(y+z)
= a^3 + 3 (x+y)(y+z)(z+x)

故に
(x+y)(y+z)(z+x)=0

よって、x,y,zの内少なくとも一つはaに等しい

という解法ができる
展開を途中で止めているけど、これは最後までバラしても
(x+y)(y+z)(z+x)が因子になることはすぐ分かるので
特に難しい事も無いはずだが…?

※分からなければ、xでまとめて因数分解してyでまとめて因数分解すればよい

解法なんてのはいろいろあるのだから、解答と自分の方針が
違っているからといって、自分の方針が間違っているとは限らないよ
485132人目の素数さん:02/05/13 07:52
この手の問題で通常の発想は

x,y,zの少なくとも一つはaに等しい
⇔ (x-a)(y-a)(z-a)=0

だろうなあ
やみくもに式を弄繰り回してもだめで、見通しを立てないとね
486うきゃ@1年ぶり:02/05/13 09:40
>482
あくまでその問題の場合は,でしょう
展開しなきゃ解けない問題だってあらーな.
(x+y+z)^3は習った公式の応用で展開できるって言ってるだけで
いつもそうしろとは言ってませんよー

たとえ(x+y)^2だって展開公式は習うけど
この形が出たらいつも展開するってわけじゃないでしょ
487まおまお:02/05/13 10:34
>>434
定義通り、ボチボチやればよろし。

・M + Ker f ⊂ f^(-1) (M') を示す

m ∈ M, k ∈ Ker f とする。
f(m + k)
= f(m) + f(k)
= f(m)
= m' ∈ M' とおくと、
m + k ∈ f^(-1) (m') ⊂ f^(-1) (M')

・ f^(-1) (M') ⊂ M + Ker f を示す

r ∈ f^(-1) (M'), f(r) = m' とする。
f(m) = m' となる m ∈ M が存在する。r - m = k と置くと、
f(k)
= f(r - m)
= f(r) - f(m)
= m' - m'
= 0' (R' の加法群の単位元)
よって k ∈ Ker f なので
r = m + k ∈ M + Ker f
>>487
模範解答を書くのは結構だけど、脊髄反射的に問題解いてませんか?
489まおまお:02/05/13 11:48
>脊髄反射的に問題解いてませんか?

そうかも(^^;
大脳皮質の発達が不十分なもんで、それを相補すべく、脊髄の
ニューロンが発達してるんだ。

>模範解答を書くのは結構だけど

私は大して頭良くないんで、ショボいミスも多い。
模範的かどうかなんて、知らんよ。
そ〜ゆーことじゃなくって、「質問」と名が付いてりゃ、実は相手に問題
出されてるってことに気づかず、解いて発表して喜んでんじゃないかと...
この場合、教師と生徒の関係が逆転しちゃってるね。
>私は大して頭良くないんで
実はそれなりに良いとは思ってる。ニュロンが発達してるからだよね〜
492132人目の素数さん:02/05/13 12:23
集合X,Yの要素がそれぞれm個、n個であるとする
m≦nのときXからYへの単射の個数を求めよ

おねがいします
493132人目の素数さん:02/05/13 12:29
>>490-491
自分よりできる者は、数学以外の点で難癖をつけて叩くわけだ。
みっともねー
>>492
n個の中から、行き先となるm個を取り出して並べ替える

∴nPm
>490,491
嫌味を言っているようにしか見えん。みっともないから止しなさい。
>489
こういうのは気にせず、無視するがよろし。
いや、でも>>490は、結構深い。ここは2chだし、色々な可能性はあるよね。

こら、>>434、何とか言え。
>こら、>>434、何とか言え
こんな昼間から2ちゃんを見てはいないと思われ。
498132人目の素数さん:02/05/13 13:28
ま、安直に答を書きすぎるという意見があるという事で
499132人目の素数さん:02/05/13 13:39
>>498
三国無双のことか
500132人目の素数さん:02/05/13 13:48
わーい500
>437
解けるけれど
やっぱり点 a の、 OX, OY に関して
対称な点を作る必要アリと思われ。
正四面体OABCについて点Oから△ABCに下ろした垂線の足Hとするとき
正四面体の1辺の長さを1とするとき、OHの長さを求めよ。

四面体OABCにおいて辺OA,BCの中点をそれぞれM,Nとし、辺AB,OCを1対3に内分する。このとき、直線MNとPQが1点で交わることを証明せよ


お願いします
503132人目の素数さん:02/05/13 16:06
>>499
今は別のこて半だけどね
>>494
「行き先」じゃねーだろ(゚д゚)ゴルァ
数学はできても日本語はダメなのか?
506494ぢゃないよ:02/05/13 16:51
言いたい事位わかるだろ? > 504
504 によると 494 ができたら数学ができるらしい
507132人目の素数さん:02/05/13 17:04
e^πi=-1
になると聞いたのですが、どうしてそうなるのか、
中学生、高校生にわかるように教えて下さい。
あと、x^iの定義を教えて下さい。
お願いします。
508132人目の素数さん:02/05/13 17:53
交換式・対称式がなんでx-yとかで表せるか証明してください
509福田和也:02/05/13 17:57
アーベル群の基本的な性質、何でもいいから教えて!後、位相空間の、
分離公理って何が言いたいのかいまいちわからんこれも頼む。
馬鹿な質問して申し訳ないですけど無理数全体の集合は完備じゃないですよね。。。?

それともう一つ質問があります。
一般の測度では一点でも測度が0にならないという話ですが、
その例を教えて下さいませんかね?お願いします。
なんか自分では上手い例が見つからないのです。
>>509
凄いね。あんた猛者だわ。

>>510
無理数からなる数列で、有理数に収束するものがあるだろ?
測度の件は、知らんよ。
無意味に病理的な例に過ぎないんじゃなくて??
512副田和他:02/05/13 18:10
ベキ零群の基本的な性質、何でもいいから教えて!後、位相空間の、
可算公理って何が言いたいのかいまいちわからんこれも頼む。
>>511
どうも、完備の方は多分僕が変な読み方をして間違えたんでしょうね。
測度の方ですが、確かに病気的な例なのかも知れませんけど、後学の為にもそこをなんとか。。。
一応一つは反例知っとかないとすっきりしないんですよ。
つか、なんかに例が載ってますかね?ざっと見たかぎりでは見つからないんですけど。
もし、載ってるようだったら済みません、もっかい探してみます。
514482:02/05/13 18:14
>>484
そんな解法もあったんですね。驚きです。ちなみに本の解答は
x,y,zを解とする3次方程式を立ててその解の中にaが含まれているから
少なくとも一つはaと等しいっていう方法です。そのために(x+y+z)を2乗
しています。

>>485
そうなんですか。でもこの問題ではそれは通用しないですよね。
515132人目の素数さん:02/05/13 18:15
※問題※
おうぎ形の中心角を求めなさい。中心角をAとする。
2π×6×360分のA=2π×3

厨房ですうう。おしえてください
516132人目の素数さん:02/05/13 18:24
2π×6×360分のA=2π×3
2πで割る
6×360分のA=3
60倍する
6×6分のA=180
A=180

A.180度。
517132人目の素数さん:02/05/13 18:28
>>515
問題の背景が気になるな。
どういう経緯でそんな問題が出るんだ?
──アーベル群の基本的な性質──

可換である
519132人目の素数さん:02/05/13 19:00
──ベキ零群の基本的な性質──

特にベキ零という訳でもない
絶対値が一未満の整数はべき零群である。
521132人目の素数さん:02/05/13 19:37
>>514
通用するから書いてるんだけどね
君には無理みたいだけど
>>520
日本語としてヘンだ。
「整数」がべき零群になるのか?
>>522
整数→整数全体の集合でOKかな?
条件を満たすのは0だけだったから、ついつい。
つーか、素人です。すんません。
シッタカシテマシタ。
>>520
どの程度つきあっていいもんか、ちょっと探ってみたくもあり
まず・・・複素数の範囲で代数的整数も考えるのかい?
525132人目の素数さん:02/05/13 21:54
nを自然数、aを実数とする。すべての整数mにたいして、

m^2-(a-1)m+an^2/(2n+1)>0 

が成り立つようなaの値をnを用いて表せ。
宜しくお願いします。















526132人目の素数さん:02/05/13 21:59
↑その空白に書くんですか?
527132人目の素数さん:02/05/13 22:04
それは言わないで・・
528507:02/05/13 22:23
e^πi・・・。
x^i・・・。
ホント、お願いします。m(_)m
>528
中学生向けには無理なんだけど・・・
530507:02/05/13 22:33
じゃぁ、もう難しくてもいいや、
ワカランならワカランなりに努力します、
教えて下さい!
531132人目の素数さん:02/05/13 22:35
厳密には高校生にも無理だろう > 528
532132人目の素数さん:02/05/13 22:36
>>507
定義だと思え
533528:02/05/13 22:37
>531
まあ、そうだけどさ。感じが分かればいいだろ。
534132人目の素数さん:02/05/13 22:37
微分積分全く分からない状態なんですけど、
良い参考書あれば教えてください。
535132人目の素数さん:02/05/13 22:42
>528
急に態度でかくなったね
536528:02/05/13 22:51
>507
ええと、>532 のいうとおり定義みたいなものなんだけど、
まあ、どんな感じなのか説明しる。

ある二つの関数が”等しい”ということはどういうことかというと、
任意の点での値と、その無限個の微分係数が
一致すればよさそうだろ。
で、 e^x の微分は何回しても e^x だから、
なんとなく
e^x = 1 + x + x^2/2! + ... + x^n /n! + ...   (1)
みたいな気がするだろ。
(右辺を微分すると自分自身に戻る)
孝行だと f (x) 〜 f(a) + f'(a) (x-a) + f''(x) (x-a)^2 / 2 + ...
みたいなの習うだろそれだ。

で、 cos x と sin x の場合は、
(cos x)' = - sin x, (sin x)' = cos x だから
cos x = 1 - x^2 / 2 + x^4 /4! - x^6/6! + x^8/8! - ... (2)
sin x = x - x^3 /3! + x^5 / 5! - x^7 /7! + x^9/9! - ... (3)
とすればよさそうだろ。(手を動かして微分してみれ)
今、(1) で x = i t (ただし t は実数) とおいて実部と虚部をまとめると、

e^(i t) = 1 + i t + (i t)^2/2! + (i t)^3/3 ! + ... + (i t)^n/n! + ....
= (1 - t^2/2! + t^4 /4! -.....) + i ( t - t^3/3! + t^5/5! - ....)
= cos t + i sin t
となる。
これに t = π を代入すれば
e^(i π) = cos π + i sin π = -1
となるだろ。

x^i = e ^ (i log x) = ((1) で x の代わりに i log x としたもの)
だ。

ああ、疲れた。
537529=533=536:02/05/13 22:52
>535
スマソ。うっかり間違えた。
>531 読んで自分が 528 だと思ってしまった。(w
無限回微分できる関数f(x) を、
f(x) = a_0 + a_1*x^1 + a_2*x^2 + a_3*x^3 + …
とかおいて、f(0),f'(0)とか計算してa_nを求めていくと
e^x sinx conx の展開式は一応わかるよ。

一応、ね。
log(-1) = ?
>539
i (2 n +1)π
logxはx>0だけじゃないん?
542132人目の素数さん:02/05/13 23:04

その質問は「√x は x≧0 だけじゃないん?」に似ている
543380:02/05/13 23:05
>>380 です
せっかくヒントもらったんですが
やっぱりさっぱりわかりません。。。
どなたかお願いします。
544132人目の素数さん:02/05/13 23:14
>541
複素数への拡張
e^(iθ)=cosθ+isinθ (>536さんの努力を見よ)
log(cosθ+isinθ )=i(θ+2nπ)
ちょっと書き方変か。でも感じは分かってもらえるかな。
545514:02/05/13 23:20
>>521
その解法を教えてください。
546132人目の素数さん:02/05/13 23:25
>380,>543
それはyahooの掲示板、数学板に書いて見た。こちらのものを
向こうに書くのは反則気味だが、漏れも昼間は2ちゃんに接続できない
環境にある。(LANの制約)
自信は無いが、一度見てくれ。
547132人目の素数さん:02/05/13 23:29
x→a のとき f(x)→A,g(x)→Bならば

(1)kf(x)→kA (kは定数)
(2)f(x)g(x)→AB
(3)f(x)/g(x)→A/B (B≠0)
を証明せよ。という問題で、
ε-δ式とやらをつかって証明するみたいなんですが全然わかりません!
どなたかご教授ください。

548132人目の素数さん:02/05/13 23:37
ガウス関数の周波数特性について教えて頂けないでしょうか?
検索しても見つかりませんでした。
非周期関数なので、フーリエ変換できないと思うのですが…。
よろしくお願い致します。
549132人目の素数さん:02/05/13 23:39
>>548
フーリエ級数とフーリエ変換の違いを調べてごらん、
ついでにラプラス変換とフーリエ変換の違いも。
550HHH:02/05/13 23:41
5次方程式の解法教えてください
x^5の係数は1でいいので
551132人目の素数さん:02/05/13 23:47
>547
「x→a のとき f(x)→A」 これをまず ε-δ で表現して見てください
552541:02/05/13 23:48
>>542
>>544
なるへそ。ありがとう!
553132人目の素数さん:02/05/13 23:49
>550
グレーフェの根2乗の方法で解くべし。
554380:02/05/13 23:58
>546
ありがとうございます。
剰余類とか奇数個の置換云々ちょっとわからないところがありますが
後は調べてやってみます。
ほんとにありがとうございました。
数学は奥が深い。。。でも、なんか楽しいです。
555132人目の素数さん:02/05/13 23:59
>>551

任意のε>0に対して、δ(ε)>0が存在する時、
lim[x→a]f(x)=Aは
|x-a|<δ(ε)→|f(x)-A|<ε

これのことですか?
556132人目の素数さん:02/05/14 00:03
>>549
なるほど。ヒントを頼りに分かりました!
有難う御座いました!!
557132人目の素数さん:02/05/14 00:05
ひとつ気になってることがあります。
円と直線ってどちらの方が点の数が多いんですか?
有名らしいですが私にはわかりません。
558132人目の素数さん:02/05/14 00:06
>>557
イミガワカリマセン
>>557
自然数全ての数の個数と偶数全ての数の個数は一緒ですが、何か?
560132人目の素数さん:02/05/14 00:08
○(円)と−(直線)を作る点の数のことです。
なにやら中学生程度の作図で解けるときいたのですが・・・
561132人目の素数さん:02/05/14 00:09
>>560
直線を書いて、それに接するような円を書く。
んで、直線から最も離れた円周上の点から、直線上のある一点を線分で結び
その線分と円との交点を考える。

すると・・・

一対一対応の話は知ってるよね。
>>555
わけがわからない。
自分でわかって書いてるのか?
563うきゃ@1年ぶり:02/05/14 00:12
>560
言いたいことはたぶんこれかな?
円周上のある一点に光源をおいて,円の下にある直線に影を作るように作図すると
円周上の点と直線の点が一対一に対応づけられるっての.
564うきゃ@1年ぶり:02/05/14 00:12
先越された上に>561の方が分かりやすいですね.向こうを参照
って僕ってこればっかりだな
565555=547:02/05/14 00:14
>>562
ぜんぜんわからん
ただ写しただけ
566132人目の素数さん:02/05/14 00:14
>561
円のほうが1個だけ多い
>561
>563
その説明だと円のほうが一点だけ多いという
結論になってしまう罠。
568560:02/05/14 00:19
みなさん素早い回答感謝です。
>567
ってことは両方とも同じ数ってことなんですか?
>>565
まず定義の意味をきっちり理解することから始めるんだな。
「x→a のとき f(x)→A」の定義を正確に書くと、

∀ε>0 ∃δ>0 ∀x [ |x-a|<δ⇒ |f(x)-A|<ε]

言い換えれば、

「どんなに小さな正の数εに対しても、うまくδをとれば、
|x-a|<δである全てのxに対し |f(x)-A|<εとすることができる」

もっと砕いて言えば、

誰かに「A-ε<f(x)<A+εとなるようなxの範囲を求めよ」と
言われたとしよう。この時、君はうまくδを選べば
「a-δ<x<a+δなら大丈夫」と答えられる、ということだ。
相手の要求するεがどんなに小さくても。
570555=547:02/05/14 00:45
>>569
なんとなくわかりました。
これでどう証明すればいいのですか?
571 :02/05/14 00:51
>>521
その解法を教えてください。
もったいぶらずにお願いします。
>>570
たとえば(1)なら、

∀ε>0 ∃δ>0 ∀x [ |x-a|<δ⇒ |f(x)-A|<ε]

から

∀ε'>0 ∃δ'>0 ∀x [ |x-a|<δ'⇒ |kf(x)-kA|<ε' ]

を導けばよい。
573132人目の素数さん:02/05/14 01:14
>>572

横槍失礼します。
εδを根気良く説明している最中に申し訳ありません。

>>547さん、εδが分かっていないなら、早めに何か買って
証明を書くトレーニングを行なったほうが良いですよ。

証明がたくさん載っている、レポートを書くのに使えそうな書籍として
「微積分と集合 そのまま使える答えの書き方」
http://www.amazon.co.jp/exec/obidos/ASIN/4061539574/qid=1021306334/sr=1-5/ref=sr_1_0_5/250-4158863-6223403
を挙げておきます。
この本の前書きを参考にして証明を書く練習を積んでください。

それでは。

失礼しました。
574555=547:02/05/14 01:16
|kf(x)-kA| = |k||f(x)-A| < |k|ε
???
だめです。やっぱり理解できません
575132人目の素数さん:02/05/14 01:30
申し訳ありませんがよろしくお願いします。

∫x/(x^4+1)dx
576132人目の素数さん:02/05/14 01:46
>>575
部分分数
>>575
∫x/(x^4+1)dx = -1/2 * Arctan (1/x^2)
578555=547:02/05/14 01:55
>>573
そうですね。本屋を漁ってきます。

明日レポート提出ですけどあきらめます
>>572サンありがとう
579132人目の素数さん:02/05/14 01:55
>>576
分母が(x^4+1)ですが、虚数を使うのですか??どう部分にして良いか
知識がありません。

>>577
実は全く手がつけられず困っているのですが、最初に何をしたら
答えがでるでしょうか。「t=tanxと置く」とかですか?置いても
進まないことには変わりありませんが・・・。
わざわざ逆数で書くか(ワラ
0の近傍で扱いが面倒くさそ。
581132人目の素数さん:02/05/14 01:57
cos(α+β)cos(α−β)=cos^2α−sin^2β

これってどうやって証明するんですか?
>>579
とりあえずx^4+1は実係数では既約ではない。
583577:02/05/14 02:13
>>579 xdx= 1/2 d(x^2)
∫x/(x^4+1)dx = 1/2 Arctan(x^2)

逆数で書いてしまった理由はあるんですが..とても恥ずかしくて言えません.
>>581
加法定理→sin^2α+cos^2α=1を使う。
585132人目の素数さん:02/05/14 02:16
>583
逝ってくれ
586132人目の素数さん:02/05/14 02:40
隊長!
>>583
∫1/(x^2+1)dx
の積分方法を知らないと思われます。

x=tany
とでもおいて、
dy/dxをxで表すことからはじめさせた方がよいと思われます。
>>579
x^4+1 = x^4 + 2x^2 + 1 - 2x^2
「この3次方程式の任意の解がをaとすれば、|a|は1であり、・・・」

問題文に上のような表現があったのですが、意味がよくわかりません。
「任意の〜」は「すべての〜」という意味と常に等しいと考えて良いのでしょうか。
また、上の文が何を意味しているのかわかりやすく解説してください。
お願いします。
|a|=1の解しか無いってことじゃないの?
590132人目の素数さん:02/05/14 03:24
漠然とした質問なんですけど、数学的帰納法って何で
成り立つんですか?
>>589
すいません、もっとわかりやすくお願いします。
あと、すいませんが全ての質問にお答えしてください。
592575:02/05/14 05:15
レスをくださった皆さんありがとうございました。>>577さんには
ご迷惑をお掛けしたようで。感謝しております。
593132人目の素数さん:02/05/14 05:26
>>590
自然数の性質そのものだから。
594132人目の素数さん:02/05/14 07:05
>>583
95%信頼区間で Mathematica を使ったと思われ
595132人目の素数さん :02/05/14 07:52
「実部がpで虚部がqである複素数αがある」

この表現はαが必ず虚数だということを示しているわけではないですよね?
実数の可能性もありますよね?誰かお願いします。
>595
複素数⊃実数
597132人目の素数さん:02/05/14 10:08
>595
虚部が0ならばαは実数。
598132人目の素数さん:02/05/14 10:21
波形が2つあります。一つは原波形、もう一つはノイズがのった波形。
原波形に対してノイズが乗った波形がどの程度類似しているかを
示す尺度は何かありますか?
>588
「任意の」と、「全ての」が常に同じかどうか・・・
微妙にニュアンスが違うので、常に同じと言い切れる自信はないですが、
少なくともこの問題の場合は、同じと思われます。問題文を言い換えると、
「この3次方程式の全ての解は、絶対値が1となる・・・」
ここでの絶対値は、おそらく複素数の絶対値でしょう。|a+bi|=√(a^2+b^2)



>>598
差の絶対値を積分してみたらどうでしょう。
601もうだめぽ。:02/05/14 10:43
time(年)  0.5  1.0  1.5  2.0
Cashflow    5   5   5   5
Spot Rate   5.0% 5.5% 6.0% 6.5%

半年後の5の現在価値は5*1/(1+0.05/2)で4.878なんですが、
それ以降の現在価値の答えが合いません。
私としては1年後は5*1/(1+0.05/2)(1+0.055/2)で4.747だと思うのですが、
5*1/(1+0.055/2)(1+0.055/2)で4.736が正解です。
5.0%から5.5%への金利の変動は考慮されないのですか?
どなたか親切な方、間違いを指摘してください。
602 :02/05/14 11:38
1÷3×3=
分数で 1/3×3=1
少数で 0.3…×3=0.9…
あれ?答えが合わないぞ?
603132人目の素数さん:02/05/14 11:54
>>598機工板で一応書いたのでそちらで。ってマルチは駄目だよ。
>>600それでは駄目なのです。(理由は忘れた。)

>>601問題の趣旨がよく分からんが、、、
Rateは定期預金みたいに長く預けると金利が変わるってことじゃないの?
半年の定期預金は5.0%
1年の定期預金は5.5%
っていう問題ではないのでしょうか?
604132人目の素数さん :02/05/14 20:17
>>596>>597
どうもありがとう!!この場合はq≠0という条件が書いてないから、実数の場合も
あるんですね。Uh〜ややこしいです。
605132人目の素数さん:02/05/14 20:18
>>599
サンキューです。助かりました。問題の意味を取り違えて間違っちゃいましたが、
もう大丈夫だと思います。ありがとう。
>>602
そうだね。
608132人目の素数さん:02/05/14 21:06
時速40キロと60キロの平均は50キロ
だから、100キロの道を往復するのには4時間かかる。
でも、行き60キロ、帰り40キロで行くと
100/60+100/40=約4.2
だれか納得のいく説明を。
609132人目の素数さん:02/05/14 21:07
平均が48キロ
610608:02/05/14 21:15
>>609
何故ですか?
200/(100/60+100/40)と出したんでしょうか。これが平均なんですかね。
>>608
おまえの知能レベルによって説明は違ってくる。
612132人目の素数さん:02/05/14 21:16
そうよーん
613608:02/05/14 21:17
出来るだけ簡単にお願いします。
614132人目の素数さん:02/05/14 21:20
>>610
であってるよ
200をXで計算すればもっと良かった
答えは同じ
615132人目の素数さん:02/05/14 21:22
>608
同じ時間ずつ走れば平均時速50kmかも知れないが、同じ距離ずつ走っても
平均50にはならない。
616132人目の素数さん:02/05/14 21:22
2X/(X/60+X/40)
617608:02/05/14 21:27
わかりました。単に(60+40)/2とやってしまった自分が悪かったです。
でも、普通こうしちゃいますよね?私だけですか。
618132人目の素数さん:02/05/14 21:28
615のせちゅめい、でちゅ

Y時間づつ60km/h,40km/hではちったとしまちゅ
ちょうすると

距離は60Y+40Yでちゅ
時間は2Yでちゅから
平均時速は50km/hとなりまちゅ
>617

相加平均を逆数でとったものを調和平均といいますが、
1/R = ((1/R1)+(1/R2))/2

気づいてるかもしれないけど、並列につないである
電気抵抗の抵抗値の計算でやった筈。

もちろん、正数の平均であれば
相加平均≧相乗平均≧調和平均
620132人目の素数さん:02/05/14 22:18
相加平均と調和平均の違いだよ
621132人目の素数さん:02/05/14 23:10
実数a.bにたいし
X(n)={1/(n^b)}[{1/(n^a)}+{1/(n+1)^a}+...+{1/(2n-1)^a}
とおく
n→∞のときX(n)が収束するためのa.bの条件とそのときの極限値を求めよ

これが全然わかりません。
よろしくお願いします
622132人目の素数さん:02/05/14 23:17

区分求積の話っぽいね
暗算だけど a+b>1 位?
623621:02/05/14 23:26
>>622
a+b≧1,極限値:a=1.b=0のときlog2
aは1でなくa+b=1のとき{2^(1-a)-1}/1-a
a+b>1のとき0
となってます
>>621
a≧0 のとき、
1/(2n)^a ≦(かっこ内の各項)≦1/n^a
となるから、

(1/2^a) (1/n^(a+b-1)) ≦ X(n) ≦ 1/n^(a+b-1)
(a<0 のときは不等号の向きが逆になる。)

ゆえにX(n)が収束するための必要十分条件は、a+b-1≧0

前半はこんなんでどうでしょ。
625624:02/05/15 00:06
やっぱ、なんか変だな。
>>521
その解法を教えてください。
もったいぶらずにお願いします。
>>621
ヒント。
次の式は、n→∞で∫[1,2]1/x^a dx に収束する。
1/n*Σ[k=0,n-1]1/{1+(k/n)}^a
>>627
おれもよくわからんが・・
629(´・ω・`):02/05/15 00:59
最も効率の良いn進数って何でしたっけ?
>>629
「効率のよい」ってのが何を意味するか知らんがとりあえず3進数辺りか?
631132人目の素数さん:02/05/15 01:26
aを「特別でない」数とするときa^n(aのn乗)の先頭の数字(1〜9)の分布
を考えよ。また「特別な数」とはどのような数のことか。
(たとえばa=10のときはa^nの先頭の数字は常に1)

632132人目の素数さん:02/05/15 01:26
>>629
2だと思う。
633132人目の素数さん:02/05/15 01:30
>>629
e(2.7)進数?
634132人目の素数さん:02/05/15 01:33
>>633
なにそれ?
635132人目の素数さん:02/05/15 01:36
情報伝達を最も効率良く出来る理想は自然対数の底「e」
e=2.7......
ただし、どうすればいいんだ
636635じゃないが:02/05/15 06:19
>>635に関して熱く語りたい方は

http://science.2ch.net/test/read.cgi/math/1014541767/

こちらへどうぞ。
637132人目の素数さん:02/05/15 13:01
>>631を読んで思ったんだけど、
日常生活で見かける自然数は、先頭の桁が1であるものが
最も多い(2〜9に比べ)と思わない?

まあ大抵の数字は連番で打たれるものだから、
当然かもしれないけど。

ランダムに自然数Nを指定し、1〜N の中から
等確率で1つを抽出したとき、それの先頭桁が
1である確率はどれくらいなんだろう。
>>637 1/9でないの?
639637:02/05/15 13:35
>>638
たとえば、手近に転がっているまんが本の巻数の統計を
取ってみて欲しい。サンプルは多いほどいい。

恐らく、サンプル数が100を超えれば、先頭の桁が
1であるものの割合が明らかに1/9より大きくなると予想される。
640じじい外科医:02/05/15 13:53
x=cosxはどうやって解くのでしょうか?30年近くまえ、高校生のころからの疑問
お教えください。
641132人目の素数さん:02/05/15 13:57
1〜N までの自然数のうち、先頭桁が1であるものの個数を a(N) とすると
lim(N→∞){a(N)/N} は収束しないんぢゃないの
log使って簡単に評価できるだろアホ
log で一様に分布してるかって問題もあるがな。
644132人目の素数さん:02/05/15 15:31
春だね〜
>>640
解を簡単な記号の組み合わせで表すことはできないはず。
つまり、近似値しか求めようがないです。
646じじい外科医:02/05/15 16:38
むずかしいんですね。ありがとうございました。
おかげさまで、30年のわだかまりが解消しました。
647132人目の素数さん:02/05/15 16:53
>>641
求める確率をPとすると

P=lim(N→∞){a(N)/N}
=lim(N→∞){(10^0+10^1+10^2+……+10^(N-2)+10^(N-1))/10^N}
=0.1111111……

では?
648132人目の素数さん:02/05/15 17:14

丁度 10^N のときはそれでいいかもしれないが...
649637:02/05/15 17:20
1〜Nの要素を持つ「ひとつの集合」でN→∞とすれば、
おそらく1/9になってしまう。

そこで、m個の集合A_k(k=1, 2, …, m)を考える。
各A_kは、1〜N_kまでの自然数を要素に持つとする。

ただしN_kはランダムに定める。(ここが最大の問題)

A_kに含まれる数のうち、先頭が1であるものの個数を
a(k)とするとき、(Σ(k=1 to m)a(k))/(Σ(k=1 to m)N_k)は
m→∞でどうなるか、というようなことを考えたいのだが、
N_kをどう定めたらよいものか…
>>637
それもう、数学の問題じゃないんじゃないの?
漫画本の巻数なら、「漫画本の巻数の分布」が与えられないと
解きようがないと思うんだけど。
>>631
[x] をガウス記号、{x}=x-[x] を小数部分を表す記号、
log は常用対数とすれば、先頭の数字が k であるためには、

k*10^[log(a^n)]≦a^n<(k+1)*10^[log(a^n)]
⇔log(k)≦{n*log(a)}<log(k+1)

一般にαが無理数なら、{nα} は、区間(0,1) 上に一様分布
するという「定理」があるから、先頭が k である確率は、
log(k+1)-log(k)
になる。例外は log(a) が有理数のとき。
「定理」を既知としないなら、実質的に「定理」を証明する
必要があると思う。その場合は、難問じゃないかな。
652132人目の素数さん:02/05/15 17:40
>>648
確かにそうなんだよね。
実感として、サンプルの区切り方?では1/9よりも大きくなってしまう場合があるのは確かです。

>>649
そういう意味だったら。。。
→サンプル抽出としての乱数方面?

母集団のサンプル数の変化と本件の確率の変化。。。についてだったら。。。
→シュミレーション、もちろんPC

本文を読むと、ランダムに定めることの数学的根拠が欲しいらしい様子ですが。。。
同値?な問題に帰着してみては?
正しいか分かりませんが。。。
⇔『無限個のボールの入った箱から一個ずつ取り出して、順番に並べていって、出来上がったであろうこの玉の順序を無秩序にいくつかの集合に分配すると……』
653数学ニガテっ子:02/05/15 18:09
添削お願いします!
【問】aを定数とする、2次方程式X^2+(a+1)X+a^2=0の解を判別しなさい。
 
 【解答】 X^2+(a+1)X+a^2=0
      この2次方程式を判別式Dとする
      D=(a+1)^2−4・1・a^2
       =a^2+2a+1−4a^2
       =−3a^2+2a+1
       =(−3a−1)(a−1)
     
        D>0 a<1/3,1<a 異なる2つの実数解をもつ
        D=0 a=±1/3,1  実数の重解
        D<0 1/3<a<1   異なる2つの虚数解をもつ
      
          

 
>>635場合分けが間違ってる。
655132人目の素数さん:02/05/15 19:32
有理数が循環する事を証明せよ。

う〜ん・・・さっぱり方針が立ちません。
循環する小数が分数の形に持っていける事は感覚的に分かってるけど、
関係無さそうだし・・・
656数学ニガテっ子:02/05/15 19:39
654さん>場合分けのところを教えてくださーい!
657132人目の素数さん:02/05/15 19:46
>639
まんが本はサンプルとして適当では無いと思われ。大体90巻も続くシリーズがあるとも
思えない。
658132人目の素数さん:02/05/15 19:59
>>657
例:こちら葛飾区亀有公園前派出所
  ゴルゴ13(作者かわっとるが)
  
659132人目の素数さん:02/05/15 20:09
>658
たしかにそうだった。
しかし・・・900巻続くだろうか?
そう思えば、5ぐらいまでの数字が多くなるのは当然じゃないか。
っところでゴルゴって何巻まで続いているんだ?
660132人目の素数さん:02/05/15 20:11
>>653
勝手に想像すると、受験生がココの板の住民の実力を試してるんでは?とカキコするテスト。w
661132人目の素数さん:02/05/15 20:21
>655
有理数が循環するとはどういうことだ。何が証明したいんだ。
662132人目の素数さん:02/05/15 20:24
>>655
ボクは、数学家の出身ではないが。。。
例えば、1/3=0.3333……
のような、ひとつの整数の循環性を考えると。。。

10÷3=3余り1
のように、整数mが、m=n×k+jで表現される時、m=10jの時に循環性が発生する事は予想できます。
つまり、数学的には
整数mに関して
mがnを法としてmodejである時、m=10jの時⇒m/nは循環小数の性質を示すのでは?

しかし、2数以上の整数の循環性に関しては知識がありません。。。(´д`;)
しかし、その時でも、基本的な循環小数の循環性はこれに帰着されるのでは?と勝手に想像しています。
しかし、わたしは、数学科出身ではないので、別のヒトの意見を求めます。。。w
663132人目の素数さん:02/05/15 20:25
>653
赤ペンが無いので添削できません。
1つだけ
因数分解では、できるだけマイナスはカッコの外へ出す。
664132人目の素数さん:02/05/15 20:26
高校1年生レベルの問題について聞いても良いですか?
>664
聞くのは自由ですが、書式が分かりにくかったり
数式が読みづらかったり
前日に日本代表が惨敗してたりすると
無視されたり煽られたりする危険があるので
あらかじめご了承ください。
666132人目の素数さん:02/05/15 20:31
a,b,c,dが正の数のとき、次の不等式を証明せよ。
(ab+cd)(ac+bd)≧4abcd

途中まではできたんですが、授業中寝てしまって、進みません。
> 途中まではできたんですが、授業中寝てしまって、進みません。
その「途中まで」を書けやゴルァ。
668132人目の素数さん:02/05/15 20:34
>>667
自乗の付け方がわからないです。
669132人目の素数さん:02/05/15 20:35
>666
コーヒータイムを2回とる。
>>668
^2
671132人目の素数さん:02/05/15 20:39
(ab+cd)(ac+bd)≧4abcd を証明するには
a~bc+ab~d+ac~d+bcd~≧4abcdを証明するば良い。(~は自乗って意味です)
a>0,b>0,c>0,d>0だから相加平均、相乗平均の関係により

ここまでやりました。
672132人目の素数さん:02/05/15 20:39
>>670
サンクス
>>671 a^2 と d^2 に関する相加相乗の式書いてみて 何をすべきか考えると吉
>671
とんでもないところで途切れてるな・・・
って ひょっとして、授業中寝てたので
ここから先はノートにうつしてないってだけの話?
675132人目の素数さん:02/05/15 20:52
>>671
相加相乗なんて使わなくても,
a~bc+ab~d+ac~d+bcd~-4abcd
=(a~bc-2abcd+bcd~)+(ab~d-2abcd+ac~d)
=bc(a~-2ad+d~)+ad(b~-2bc+c~)
とすれば,もう分かるでしょ
676132人目の素数さん:02/05/15 20:55
>>673
やってみます。
>>674
そういう訳じゃないです。
>>675
左辺-右辺でやるのか。一応、相加相乗平均の部分の問題だったので。
677637=649:02/05/15 20:59
結局次のように解決してみたのだが、どうだろう。
N_kをランダムに選ぶのが難しいため、始めから10^m種類に固定してしまう。

10^m個の集合A_k(k=1, 2, …, 10^m)を考える。
各A_kは、1〜kまでの自然数を要素に持つとする。

A_kに含まれる数のうち、先頭が1であるものの個数をa(k)とすると、
それらの合計 S1 = Σ(k=1 to 10^m)a(k) = (19x^2 + 11x + 168)/198 ただしx=10^m
(↑これの計算が激しく面倒)

一方、全ての要素数の合計 S = Σ(k=1 to 10^m) k = x(x+1)/2
従って、lim[m→∞] S1/S = 19/99■

というわけで、連番で打たれている番号のみを対象にする場合は、
なんと1/5近い割合で先頭が1であることがわかった。
678674:02/05/15 21:04
>676
あそこで相加相乗を使うことが分かり、しかも
どう使ってイイかは分からないという状況が
想像できないんだが・・・

ともかく、
a^2bcとbcd^2, ab^2dとac^2d にそれぞれ相加相乗 (>673)
a^2bc, ab^2d, ac^2d, bcd^2 の4つ全体に相加相乗
のどっちでもOK.
もちろん、>675でも解ける.
679132人目の素数さん:02/05/15 21:14
>>666
この解法はダメ?

a,b,c,dが正の数のとき、次の不等式を証明せよ。
(ab+cd)(ac+bd)≧4abcd …@

式@を証明するには、
{(ab+cd)(ac+bd)}/4≧abcd …A を証明すればよい。

ここで、式Aの左辺を次のように変形する。
 {(ab+cd)/2}{(ac+bd)/2} …B

式Bの各項について、相加・相乗平均を用いると、次の不等式が成立する。
 (ab+cd)/2≧√abcd …C
 (ac+bd)/2≧√abcd …D
ただし、等号成立はa=b=c=dのときである。

ここで、式C、Dの各左辺右辺どうしをかけると
 {(ab+cd)/2}{(ac+bd)/2}≧(√abcd)(√abcd)
⇔{(ab+cd)(ac+bd)}/4≧abcd …A

これで、式Aが証明できた。
また、等号成立はa=b=c=dのときである。

以上。
680132人目の素数さん:02/05/15 21:15
>677
どうだろうと言われても、蚤(k)をどうやって計算しているか示してもらわないと
確認のしようが無い。
計算間違いしているほうに、100ムーン
そういうものがだせると思ってないけどね。
681676:02/05/15 21:16
解けました。みなさんありがとうございました。
しかし、このままじゃ数学のテストがヤバそう。
682132人目の素数さん:02/05/15 21:17
>655
x,yを正の整数として、数列a(n)を
x÷y の余りをa(0)
10x÷yの余りをa(1)
100x÷yの余りをa(2)
・・・・・・
(10^n)x ÷y の余りをa(n)
によって定義する。、
各a(n)は、0以上n-1以下の整数になるよね。
もし、a(n)=0となるnが存在すれば、x÷yは小数点第n位までで十分表せる。
a(n)=0となるnが存在しないと仮定しよう。
ここで、a(0),a(1), ...., a(n-1)と、n個の整数を考えると、
「鳩の巣原理」により、a(j)=a(k) となるj≠k が0,1,...,n-1の中に存在する。
このようなj,kのうち、L=|j−k|≠0 が最小となるものをとる。
すると、x÷yは、循環節の長さがLの小数として表せる。
683682:02/05/15 21:18
あ、間違ってる。
でも、まあ、こういう感じでギロンをすすめればいいと思われ。
メンドウなので、直すことはしない。
684132人目の素数さん:02/05/15 21:18
隣接行列を2乗すると、二つ離れたモノとの関係を示す行列になるそうなのですが、
その理由がよくわかりません・・・

どのように考えたら良いのでしょうか。
よろしく御教授ください。
685132人目の素数さん:02/05/15 21:20
区間0<x<2πにおいて次の関数の増減を調べよ
y=tanx−x
教えてください。
微分しろ
687132人目の素数さん:02/05/15 21:36
>655
ひょっとして
割り切れない有理数を小数で表すと循環小数になる
と言う意味か?それは自明だぞ。割算してみればいい。
例えば1/7なら余りに使える数は6個(0も入れれば7個)だから7回割算を続ければ
7回以内に必ず同じ数が表れて、そこから繰り返しに入る。
688132人目の素数さん:02/05/15 21:43
>684
素人なので隣接行列というのが良く分からん。
x2↑=Ax↑ というようなことか?
x3↑=Ax2↑=A(Ax↑)=A^2x↑ だわね。(行列の結合法則)
689132人目の素数さん:02/05/15 21:46
>>687
本質的には、それですね。
でも、非循環小数は?と考えてしまったが。。。有理数という条件があるから、イイんですね。
質問者ではありませんが、勉強になりました。サンクスです!
690655:02/05/15 21:46
>>682
各a(n)が0以上y−1以下の整数になるって事ですか?
あと、y−1<nとなるnを取って、鳩の巣原理に持っていく。

691132人目の素数さん:02/05/15 21:47
「九点円は同一円周上にある」ことをベクトルを使って示せ。

という問題なんですけど、お願いします。
692637=649:02/05/15 21:48
>>680
a(k)は単独では出てこないけど、Σa(k) なら出せる。
Σa(k)の中には、

1が10^m個
10が(10^m)-9個
11が(10^m)-10個
  :
19が(10^m)-18個
100が(10^m)-99個
101が(10^m)-100個
  :
199が(10^m)-198個
  :

というふうに並べて、合計する。

とりあえず式が合っているかどうか、
m=1, 2, 3 あたりで試してみていただきたい。
693132人目の素数さん:02/05/15 22:00
>>691
『九点円』って何?>語彙不足?
>>691
フォイエルバッハじゃねえの?急転圓。
695682:02/05/15 22:02
>690
そーいうこと。
ウルサクいうと、本当に長さLの循環節になることも示したほうがいいのかもしれないけど、
あまりにもタイクツかつアホラシイ証明になる。
まあ、証明に慣れていないヒトにとってはいい練習になるかもね。
>693
煽っているのか
本当に九点円の定理をしらないのか
判断に苦しむ。
697132人目の素数さん:02/05/15 22:07
>>696
知らないです。
理系なんですが、数学科出身ではないです。。。(´д`;)
ちなみにSEG出身でもないです。
698132人目の素数さん:02/05/15 22:12
>>697
http://www2.tokai.or.jp/yosshy/theorem/9ten-en.htm
あぁ・・・なんて優しいんだろう俺って
699132人目の素数さん:02/05/15 22:13
>>696
これですね。
インターネットって便利ですね。この時代に生まれてきてよかったです!w

http://www.google.co.jp/search?q=cache:pN0tjBkQfYEC:www.ies.co.jp/math/cabri/sakuzu/geobasic.html+%8B%E3%93_%89~%82%CC%92%E8%97%9D&hl=ja
700696:02/05/15 22:14
>698
なんで691よりも693が
優遇されてるんだろう。(w
701132人目の素数さん:02/05/15 22:15
>>698
サンクスです!
数学板の人って親切ですね。
ありがとうございます。
これで益々、数学が好きになれそうです。34歳ですが。。。(´д`;)
>>701
その一方で誰も>>691には答えないという罠。
数学板というのはそういうところです。
703655:02/05/15 22:19
>>682
すいません、もう1つ聞きたいんですが、
xを10倍ずつしていく事はどうしてそう思ったんですか?

「余りに限りがあるから循環する」って事がいまいちよく分かってません^^;
ユークリッドだとは思うんですが・・・
>691
自明。
705682:02/05/15 22:41
>703
10倍ずつしていくというのは、筆算の発想。
実際に手を動かして割り切れない割り算をやってみるとすぐにわかる。
余りの後ろに0をつけて筆算を続行していくよね?

あまりに限りがあるというのは、、
たとえば、7で割るときは、余りは0,1,2、3、4、5、6 のいずれかになります。
余りが0のときは割り切れたことになるのでこれを除外して考えると、
余りとしてとりうる数は高々6とおりです。
筆算をすすめていって小数点以下の余りを7個得たとき、この中に同じ数が必ずあります。
ウソだと思うなら、1〜6の整数をなんでもいいから7個書いてみてください。
そうすると、必ず同じ数字を見つけることができるでせう。

筆算をしていて同じ余りを得たら、その後は同じ計算の繰り返しになることは、
経験上明らかでしょ?
経験がないのなら、実際に計算してみてください。
例えば、1÷7を手を動かして計算してみてください。
706132人目の素数さん:02/05/15 22:48
lim√(x^4-x^2)/x
x→∞
( ´_ゝ`)フーン
708655:02/05/15 22:58
>>682
納得しました。ありがとうございました。(^o^)

709132人目の素数さん:02/05/15 23:03
>706
で、どうしろと?
710132人目の素数さん:02/05/15 23:04
<710
解いて欲しいってさ
711132人目の素数さん:02/05/15 23:07
無言で宿題を放り投げるようにしてやらせようとしてる感じ(w
712132人目の素数さん:02/05/15 23:33
>677
そんな計算を始める前にA1とA2が同じ程度に存在することが言えないとダメなんじゃ
ないの。その計算だと、A1やA2が1個ずつという前提だよね。
まんが本の例えでいえば、1巻だけで終わりというのが無数にあるだろうから、
そりゃ1が多くなるわね。そういう意味では感覚は間違っていないかもね。
713 :02/05/16 01:10
群数列
|1|1、2|1,2,3|1,2,3,4|・・・

|1|2,1|3,2,1|4,3,2,1|・・・
それぞれの第n項を求める式は?

714132人目の素数さん:02/05/16 01:48
>>713ちょっと待ってろ
716132人目の素数さん:02/05/16 01:53
>>715ちょっと待ってろ
717132人目の素数さん:02/05/16 10:18
>>653
まだ見てるかな?
D=(−3a−1)(a−1)
 =−(3a+1)(a−1)

a^2の符号が重要なので慣れないうちは符号を前に出す。

a<−1/3,1<a D<0 異なる2つの実数解をもつ
a=−1/3,1   D=0 実数の重解
−1/3<a<1   D>0 異なる2つの虚数解をもつ

補足
a=0,-1/3,1,2ぐらいは代入してみて検算しましょう。
718717:02/05/16 10:19
うわ、訂正。
a<−1/3,1<a D<0 異なる2つの虚数解をもつ
a=−1/3,1   D=0 実数の重解
−1/3<a<1   D>0 異なる2つの実数解をもつ
719132人目の素数さん:02/05/16 11:02
スレ違いと言われるかも知れませんがお聞きしたいことがあります。
なお、当方数学板初心者且つ、文系の人間です。
グラハム数スレに出てきたのですが、「スキューズ数」「超スキューズ数」
と言うのはどんなものなのでしょうか?検索しても具体的には出てきません。
できれば例えも含めて分かりやすくご教授いただければ僥倖かと存じます。
720132人目の素数さん:02/05/16 12:01
173 :参加するカモさん :02/03/12 09:46
みんなメルマガ読んでる?
あれ読むとひろゆきの性格よくわかるよ
すっごい陰湿。
根暗。
正直、引くよ。2chの管理人って、こういう奴なのかって。

176:参加するカモさん :02/03/12 11:18
ひろゆきの裁判の報告の仕方はよくわかったけど、たまには相手側の言い分も聞いてみたいね
まったく違う状況だったりしてw

177:参加するカモさん :02/03/12 11:42
ひろゆきさんっては自分の都合の良いことしか報告しないし、都合が悪くなると
報告しなくなっちゃうんだよね、、
それにしてもひろゆきさんてば見事に陰湿な文章書くなあ。。
721132人目の素数さん:02/05/16 12:26
>>637

えーと、大昔に読んだ本なので題名もなにも覚えてはいないのだけど、
たしか市町村の人口の最初の一数字も1が一番多いはす。モノの量とか
で対数とった方がキレイになる分布は、よほどのことがない限り、1が
一番多くなるはず。
722132人目の素数さん:02/05/16 12:29
コピペに反応するのもアホらしいが…
>>720
そんなに嫌なら2chに来なきゃいいのに(w
化学屋です。
どなたか体積計算してくれませんか?

木でできた円柱をイメージしてください。
それを横にして見たときに、その片側の端面に
自分の見ている方向から向こう側へ
半円状の彫刻刀で削った場合の、削った
部分の体積を計算したいのですが、
なにか公式などあるのでしょうか。
>>723
葬儀屋です。
死んでください。
725132人目の素数さん:02/05/16 13:37
>>723イメージが湧きません。(w
円柱と円柱が垂直に重なっているとき、その部分の体積ですか?
723です
図を書き込めたらいいのですが、方法がわからないので
このまま文章にします。

円柱を円の面があるほうを横にすると、長方形になりますよね。
そうした時に、左右どちらでもいいのですが右側としましょう。
長方形の右側のたての線(円周)の中点を中心として左側に
半円を書くように向こう側へ彫っていくイメージです。
(サッカー場をセンターサークルで折り返したイメージです)

円方向からみると、円の中心線に平行な2本の線が見える
ような格好になるかと思います。

うまく説明できませんが、どうでしょうか?
>>726
元の円柱の半径をa、くりぬくそれをbとすると(a>b)、
くり抜かれる部分の体積は

∫[x=-b to b] 2√((a^2-x^2)(b^2-x^2)) dx

になる。多分ね。
しかしこの積分が容易じゃなさそう。
728システムおばかさん:02/05/16 15:52
まーったくわからない問題がありまして。かきこんでみることにしました。
Φ(x・y)=Φ(x)*Φ(y)ならばΦ(x)=Π[i=1,n]xi
証明できないんです。
>>726 >>725の半分です。>>727であってると思うけど。そこから、、わからん。
>>728
>Φ(x・y)=Φ(x)*Φ(y)ならば
って、Φ(x)=1 とかでもいいのか?
>Φ(x)=Π[i=1,n]xi
の xi ってなに?x かける i?
>728
xiのiはおそらくxの添え字だと思うが、条件が少なすぎて、答えられない。
もう少し詳しく頼む。

(Φ(x)=Π[i=1,n]xiはxの関数のはずだが、右辺にxがないのも?です。)

732未熟者です ◆w.2maCMo :02/05/16 16:16
あまりに厨な質問ですみません。
√2+1 の整数部分をa、小数部分をbとした時の
a+2/b の求め方を教えていただきたいのです。
お願いします。
>>732
a=2
b=-1+√2
で計算すりゃいいんでネーノ?
734未熟者です ◆w.2maCMo :02/05/16 16:23
>>733
ありがとうございます。
やってみます。
735システムおばかさん:02/05/16 17:07
失礼しました。
Φの括弧の中のxはベクトルで、右辺は小文字のxです。iは添え字です。
で、これはシステムの話で、ベクトルx,yは1か0で構成されているベクトルです。
こんなもんでいいでしょうか。
>>735
情報が足りなすぎ。

台集合と係数体は何?
ベクトル演算(加法・実数倍)はどう入ってるの?
演算・(内積?)の定義は?
Φの定義域と値域は?
737132人目の素数さん:02/05/16 17:29
まったくわかりませんのです。
オシロスコープやってます。そこでCH1とCH2の波形を比較したい場合2現象表示ではTが1周期すなわち2π、tが位相差φに相当するとφ=360°×t/Tになる。
そして、リサジュー図形(XーY表示用いての位相の比較)でXY入力を零にしてとると位相差φはφ=アークサインb/aで表される。これらより
φ=360°×t/T=アークサインb/aの数式の証明を教えてください。
導出できる数学書もあったらいいのですが。お願いします。
>>737何がわからん?そのままじゃん。
739祭り:02/05/16 18:35
2ちゃんねらーの力でオールスター選手を誕生させましょう!
“不正投票無し”の人海戦術で2ちゃんねるの底力を見せましょう!地道に『1日5票、携帯も合わせて10票』で投票願います。
『2002サンヨーオールスターゲーム』
http://allstar.sanyo.co.jp/
2ちゃんねる党推薦候補は、
広島東洋カープ・キャッチャー・背番号63・鈴衛祐規さん


http://sports.2ch.net/test/read.cgi/base/1021303023/l50
740名無し:02/05/16 18:55
|x-3|≧2 これ。基本問題なんだけど、何故かこれだけ解けません。
わかる方、教えて下さい。
x-3≦-2、とかの方法でやっていいんでしょうか?
>>740
両辺正なので2乗してよい。

または-2≦x-3≦2
742?1/4?3?μ:02/05/16 19:13
>>741 有難うございます。
絶対値||を外す時に、二通り場合を考えないといけないらしいんで、
-2≦x-3≦2 では間違いだといわれました。
743名無しさん:02/05/16 19:15
問題1
φ(t)は0≦t<∞で定義され、ある正の定数Tが存在して、t>Tでφ(t)=0
である。(0≦t≦Tでのφ(t)のグラフは各自で決めよ。)このとき新しい
関数f(t)をf(t)=φ(t)、f(t+T)=f(t)、0≦t<∞と定義したとき、f(t)の
グラフはどうなるか?
問題2
lim[x→∞]{x+(1/x)}=eを用いて、im[x→-∞]{x+(1/x)}=eを証明せよ。
ヒント;x=-zとおく
744741:02/05/16 19:21
>>742
悪い、不等号の向きを勘違いしていた。
741の下の行は忘れてくれ。
745132人目の素数さん:02/05/16 19:23
>740
x−3<=−2,2<=x−3
746132人目の素数さん:02/05/16 19:41
Σ(k^(n-1) /n!)×a(k)=
1(n=0)
0(n=0以外の偶数)
Σはk=0から∞まで。
を満たす数列a(k)を教えてください。
747yokotee:02/05/16 19:46
初めましてyokoteeと申します。
友達から出された問題がよく分からないので、教えてください。
(もしかしたら答えが間違っているかもしれませんが。。。)

X2(Xの二乗)−4x+1=2x(1−4x) です。

情報提供お願いします。
748スッドレ予想師。:02/05/16 19:56
>>743
こんな風に解きました。。。

問題1
関数f(t)は題意より、周期Tの周期関数。
しかし、t>Tで、f(t+T)=f(t)=φ(t)=0
したがって、f(t)=0、ただし0≦t<∞

。。。では?ダメ?

749132人目の素数さん:02/05/16 20:15
>>747 方程式を解けってことだよね?

右辺を展開して左辺に移項。 できる式が9x^2−6x+1=0
これを左辺因数分解すると解けるよー。
750スッドレ予想師。:02/05/16 20:26
>>747
X2(Xの二乗)は x^2 と書きます。。。
それでは、

 x^2-4x+1=2x(1-4x)
⇔x^2-4x+1=2x-8x^2
⇔9x^2-6x+1=0 …… これを、解けば?
751スッドレ予想師。:02/05/16 20:27
>>749
スミマソン。既出でしたね。w
752スッドレ予想師。:02/05/16 20:31
>>750
ちなみに、
 9x^2-6x+1=0
⇔(3x-1)^2 。。。ですね。

このパターンの、因数分解が出来ないと予想。> >>747の高校生に100ウォン。
753132人目の素数さん:02/05/16 21:03
複素数zに対して、ω=(z-i)/(z+i)とおく。
点zが複素数平面の実軸上を動くとき、点ωはどのような図形を描くか。

わかりません。
754スレッド予想師。:02/05/16 21:07
厨房です。教えてください。
1 mを自然数とする。√(m^2+1)を小数で表したとき、
  少数第一位が1になるという。mを全て求めよ。
2 nを自然数とする。自然数286、412、727、937を
  nで割ったところ、余りが全て等しくなるという。
  nを全て求めよ。
755132人目の素数さん:02/05/16 21:08
2は引き算して約数求めればいいような気がする
7562項定理??:02/05/16 21:12
(1 + x + x^2 + x^3 + …… + x^n)^4 のx^nの係数を求めよ。
答えは(n+3)Cn (←コンビネーション)だそうなんですが...
解き方がわかりません。(>_<)
757うきゃ@1年ぶり:02/05/16 21:14
>753
与えられた式を,z=(ωの式)と変形する.
ここで,αの共役な複素数をα'とすると
zが実数
⇔z=z'
⇔(ωの式)=(ωの式)'
⇔・・・

以下整理すればおっけーです
758うきゃ@1年ぶり:02/05/16 21:16
>753
757の追記
z=(ωの式)にする時点で,分母が0になってはいけないことから,
ω=1のときとそうでないときに場合分けが必要だよー
ω=1のときは元の式にいれるとi=-iとなり矛盾するのでダメ.
これが答えに影響してきます
759質問です:02/05/16 21:23
次の数列の極限値を求めよ
@{1/√n} A{(sinθ)^n} B{(1/2)^n-(1/3)^n} C{(√(2n+1)/n)/((4n^3+1)/n^3)}
次の数列の収束、発散を調べよ
@{n^3/(n^2+5)} A{5^(1/n)} B{sinNπ+cosNπ}
760132人目の素数さん:02/05/16 21:23
>758
ωω´=1とかなりますか?
761うきゃ@1年ぶり:02/05/16 21:24
>760
あってますよー
そこから|ω|=1なるのは分かりますか?
762:02/05/16 21:27
この問題を解いて下さい。

12×3の床に1×2のタイルを「余すところなく」敷き詰める方法は
何パターンあるか?
(例)2×3の床に敷き詰めるパターンは3つある。

高校レベルの問題だと思うのですが、どういう計算式を使えば
いいかよく分かりません。
私なりに計算したら、2099パターンという答えが出たのですが、
確証は持てません。
763うきゃ@1年ぶり:02/05/16 21:27
>754
とりあえず(1)だけ
√(m^2+1)はm+1を越えないので(証明略)
√(m^2+1)=m+αとおくと
m+αの小数第一位が1なことから0.1≦α<0.2
ここに,α=-m+√(m^2+1)を代入して・・・
764うきゃ@1年ぶり:02/05/16 21:35
>756
4乗だから,(1+x+・・・+x^n)が4つあると考えて
例えば,1つ目の()からx^3を,2つ目からx^2を,
3つ目からxを,4つ目からx^5を選べば,
x^11ってのが出てくる

上手く選んでx^nとする方法は何通りあるか,と考えると
これは,n個のボールを4人に分ける(1個ももらえない人がいてもよい)問題になるので,
○をn個,棒(区切り)を3本並べる=(n+3)C3 通りとなる

・・・2項展開使って解けっていう指定あるの?(--;;;

765うきゃ@1年ぶり:02/05/16 21:38
>759
丸文字は使わない方がいいですよー見えない人がいるらしいです
で,(1)から分からない?lim[n→∞](1/√n)を求めるだけだよ
766うきゃ@1年ぶり:02/05/16 21:41
>762
(例)でなぜ3パターンになるかわからない...本当に3パターン?
僕の問題解釈がおかしいのかな?
767132人目の素数さん:02/05/16 21:45
『 /sqrt(n) 』
とは何を表している記号でしょうか?
768132人目の素数さん :02/05/16 21:54
外部から759の質問に便乗なんですがlim[n→∞](1/√n)の解き方はどうすればいいのでしょうか?根本的に私の場合はそこがわからないんです
769うきゃ@1年ぶり:02/05/16 21:55
>767
sqrt(n) = √n
ちなみに sqr(n) = n^2
>767
/sqrt(n) は、/√n のことと思われ。
つまり、x/sqrt(n) = x ÷ √n

プログラム言語を使った書き方。
771うきゃ@1年ぶり:02/05/16 21:58
>768
分母は+∞に発散,分子は1に収束
だからlim[n→∞](1/√n)=0
イメージ的には,1をバカでかい数で割るとほぼ0になるって感じ
772767:02/05/16 22:01
>769
助かりました。ありがとうです。
773767:02/05/16 22:09
>770さんもありがとう。
774うきゃ@1年ぶり:02/05/16 22:49
>762
ごめん,>766は気にしないと言うことで
急に問題がたまったからあせってたのだー
つーわけで計算中
連続カキコご迷惑をおかけしております>ROM含めALL
775132人目の素数さん:02/05/16 22:52
質問です。
{a(1±Δa/a)}^α{b(1±Δb/b)}^β
はどうやって分解するのですか?
どうしてもすっきりした形になりません。
776132人目の素数さん:02/05/16 22:52
a1≡b1,a2≡b2(mod m)ならば、
a1・a2≡b1・b2(mod m)となることを証明せよ。
>>776
a1・a2≡b1・a2≡b1・b2 (mod m)
778 :02/05/16 23:03
R=px R:収入、p:価格、x:生産量
両辺の変化率をとると、
儚/R=冪/p+凅/x

となるそうなのですが、証明してください。
779132人目の素数さん:02/05/16 23:06
儚=x冪+p凅

7801年生:02/05/16 23:09
r=xi+yj+zk
とするとき、閉区間Cについて
∫r・dr=0
となることを示してほしいんですが・・
781 :02/05/16 23:09
なるほど。速い。
ありがとうございました。
782うきゃ@1年ぶり:02/05/16 23:10
>762
2131通りになりました

縦3横nの長方形にしきつめるパターン数をa(n)
そこから最左列の下と真ん中の2つをとった形に並べるパターン数をb(n)とする

a(n+2)は,

i)
○○・・・ ○○・・・ ○●・・・
●●・・・ ●×・・・ ○●・・・
××・・・ ●×・・・ ××・・・ と並べる方法:3a(n)通りと

ii)
○○・・・・
●××・・・
●☆☆・・・ と並べる方法(上下反転あわせて)2b(n)通りあるので

よって,a(n+2) = 3a(n) + 2b(n)

同じように考えると b(n+2) = a(n) + b(n) となります(やってみよう)
あと,a(2)=3,b(2)=1から,こつこつとa(12)まで求めましょう
漸化式を解く方法は・・・ちょっとわかりません(--;;;
問.曲線y=x^3-4x^2+3xに対して、点z(p.0)から
異なる3本の接線がひけるとき、pは□の範囲である。

の問題を次のように考えました。
3本の接線を持たないときのx軸上の点z(p.0)の位置は、
3時間数yのグラフは、↑↓↑となるが、その真ん中の↓の部分を
通る直線(Lとする)とx軸との交点から左側の部分である。
Lは変曲点の中心の(4/3、20/27)を通り、その傾きは
y'=3x^2-8x+3にx=4/3を代入して、-7/3である。
これより、Lが求まり、Lとx軸の交点より右側の部分が求める範囲。
ここで、Lと垂直な線L'とすると、同様の考え方をして、
L'より左側が求める範囲。

このような方針でやりました。
解答がないので、添削してください。
784132人目の素数さん:02/05/16 23:28
次の極限値を求めよ。
 lim[n→∞]農[k=1,n]1/(k^3) 
学校の宿題なのですが、まったくわかりません…
お願いします
>>784
宿題でζ(3)かよ。
すごい学校だな。
786132人目の素数さん:02/05/16 23:45
>>785
ζ(3)ってなんですか?
なんか難しい問題なんですか?
787132人目の素数さん:02/05/17 00:00
>>786
未解決問題・・・
788784=786:02/05/17 00:07
>>787
未解決ってことは解けないってことですよね
何考えてんだうちの教師は…
>784
気休めだけど
ζ(s) = Σ(k=1, ∞) 1/k^s
とすると、
ζ(0) = 1/2
ζ(2) = π^2/6
ζ(4) = π^4/90

高校なのか?
s=4 と勘違いしたといって (以下略)
790132人目の素数さん:02/05/17 00:08
はぁ。アホか?
791132人目の素数さん:02/05/17 00:18
>>784
lim[n→∞]農[k=1,n]1/(k^3) 
この値が存在することの証明でしょ?
はぁ。アホか?
>>789
ζ(0) = -1/2 という罠
794789:02/05/17 00:27
>793
あうち。
やってしまった。風呂には逝ってきます。
ζ(-1) = -1/12だよね。
796132人目の素数さん:02/05/17 00:39
ζ(s)の、s≦0での値は、どのような考え方で
出されるものなんでしょうか。
普通に考えたら発散するとしか思えないんですけど。
r が 2以上の偶数のとき
Br を r-th ベルヌーイ数として
ζ(1-r) = -Br/r
なり。
あと正の偶数のときの公式もあるだ。
798スッドレ予想師。:02/05/17 00:43
>>574
勝手に、PN使うな!ボォケ。
800スッドレ予想師。:02/05/17 00:47
失礼。
>>798は、>>754のこと。

801796:02/05/17 00:56
>>799
ありがとうございます。読んでみます。
>>800
HNのこと?
使ってないじゃん。
騙られるのが嫌ならトリップ付けろ。
馬鹿
803132人目の素数さん:02/05/17 01:07
>>800
頭悪そぅ(ぷ
804132人目の素数さん:02/05/17 01:10
>>803
はいはい。厨房。w
「出しっこするもの寄っといで〜、出しっこしましょ〜:−」
>783
方針ではなく、キミの回答を見ないと添削できない。
ていうか、方針がわけわからんのだけど。
807魔羅師匠:02/05/17 11:24
      (\.  /⌒\  /)
      (\\(    .)//)
       (\\│   │//)
   / ̄ ̄  . │   │   ̄ ̄\    / ̄ ̄ ̄ ̄ ̄ ̄ ̄ ̄ ̄ ̄ ̄ ̄ ̄
  (/// ̄ ( ・∀・) ̄\\\)  < やぁみんな!魔羅師匠だよ!
     ( \) ⊂   つ(/  )     \_____________
      \\) │││ (//
       \) (_)__) (/

ドキュソ質問ですまないけど、二乗を使った式の操作がタブーなのは何故?
たとえば:
e^(πi)=-1
(e^(πi))^2=(-1)^2
e^(2πi)=1
ln(1)=2πi
ln(1)=0より、2πi=0
で成り立たなくなる
>>807
それ、2乗に関するパラドックスじゃないよ。e^(2πi)=e^0 だから、
複素数まで考えると e^x は、多対一の関数。
逆関数は一対多になってしまうからそういうことが起こるだけ。
tan(π/4+2π)=tan(π/4)=1 より π/4+2π=arctan(1)=π/4 とやっている
のと同じ。
指数法則に関するのパラドックスなら、次のようなやつで聞け。

(1) -1=(-1)^(1/3)=(-1)^(2/6)={(-1)^2}^(1/6)=1^(1/6)=1
(2) -1={√(-1)}^2={√(-1)}*{√(-1)}=√{(-1)*(-1)}=√1=1
809132人目の素数さん:02/05/17 16:04
整数係数の多項式f(x)=a[n]x^n+a[n-1]x^(n-1)+・・・+a[1]x+a[0]は
a[n]=a[0]=1,a[n-k]=a[k]>0(k=1,2,・・・,n-1)をみたす。
方程式f(x)=0が有理数解αを持つとき、1/αもf(x)の解であることを示せ。

がわかりません。
810( ○ ´ ー ` ○ ):02/05/17 16:14
>>809
計算するだけだべ( ● ´ ー ` ● )
>>809 係数が対称なので f(x) は (xの巾)*{(x+1/x)の多項式}
という形で必ずかける. x+1/x の組合せ x →1/x の入れ換えに対し
不変なので αが解なら逆数も解 といえる.
>>809
f(α)=0 とする。このとき、
f(1/α) = … = (1/α^n)f(α) = 0■

有理数という条件はいらないんじゃないの?
0以外の任意の複素数でOK。
813132人目の素数さん:02/05/17 19:46
これってパラドックスなのか? > -1={√(-1)}^2={√(-1)}*{√(-1)}=√{(-1)*(-1)}=√1=1
814132人目の素数さん:02/05/17 19:58
>>813
虚数の範囲でなければ、√(-1)は、定義に無いのでは?>高校生?
815132人目の素数さん:02/05/17 20:00
>>813
パラドクスだよ。さて、どこがおかしい?
816132人目の素数さん:02/05/17 20:02
{√(-1)}*{√(-1)}=√{(-1)*(-1)}

ここにきまっとうが
817132人目の素数さん:02/05/17 20:15

負の数に対しては許されていない根号計算をするのが
パラドックスなのか?
818132人目の素数さん:02/05/17 20:20
誰がパラドクスと言ったかね
819132人目の素数さん:02/05/17 20:31
>>818
言ったのは、>>815氏ですね。
>>818氏と同一人物かは不明ですが。

私見ですが、一見パラドックスに見えるけれでも、その実は、>>817氏の指摘する内容ですね。
820132人目の素数さん:02/05/17 20:39
パラドクス=逆説・逆理

1:一見矛盾しているようだが、実は真実をうがった命題。

2:論理的には正しそうに見えるのに、矛盾した結論が出てくる命題。
  (実は推論のどこかに誤りがある)

数学で用いられるそれは、ほとんどが2の意味である。
失敬。2は以下に訂正。

2:論理的には正しそうに見えるのに、矛盾した結論が出てくる証明や推論。
  (実は推論のどこかに誤りがある)
822132人目の素数さん:02/05/17 20:44
>>820
なるほど。。。
勉強になります。>本件では、>>820の2が該当しますね。
サンクス!
823132人目の素数さん:02/05/17 20:47
論理的に正しそうに見える? > {√(-1)}*{√(-1)}=√{(-1)*(-1)}

824822:02/05/17 20:55
>>823
その前に、√内が負の数であることに気が付き。。。
定義されていませんから。虚数範囲外では未定義ですよね?
825132人目の素数さん:02/05/17 22:10
>824
誰か実数と言ったかね?
826:02/05/17 22:56
>>782
サンキュウです、ということは2131−2099=32で、
何か数え忘れていたのでしょう。
ざんか式ですか。やはりそれしかないんですね。確率でもないとは
思ってましたが。
827132人目の素数さん:02/05/17 23:02
複素数平面上の点列P[1](z[1]),P[2](z[2]),…は
z[1]=1+i,z[n+1]=(1+i)z[n]+1(n=1,2,…)
をみたしている。
r=16^m(m=1,2,…)とするとき
O(0),A(r),B(r+ri),C(ri)を頂点とする正方形OABCの
周および内部に含まれるP[n]の個数を求めよ。

お願いします。
>827
自力でz[16]ぐらいまで計算して見ろ。
すぐ規則性見つかるから。
829132人目の素数さん:02/05/17 23:51
お願いします。

行列X'=[1,1,1,1],[4,-2,3,-5]
があるとき、
P=X(X'X)^-1X' と M=(I-P) を計算し、MP=0であることを示せ。

ヒント:Q=[1,3],[2,8]
・PとMを、Xの代わりにXQを用いて計算する。
・MとPの固有値は?

というものです。力技で計算したらMP=0にはなったんですが…
このQをどうやって使えばいいのかよく分かりません。
>>829
何だか意味不明。
831829:02/05/18 00:01
>>830
すんません、じゃあ英語の原文で。
For the matrix
X'=[[1,1,1,1],[4,-2,3,-5]]
compute P=X(X'X)^-1X' and M=(I-P).Verify that MP=0
Let Q=[[1,3][2,8]]

Hint:Show that M and P are idempotent.
a.Compute the P and M based on XQ instead of X.
b.What are the characteristic roots of M and P?

です。
832オラ来る:02/05/18 00:04
問題の意味がわからなくて困っています。

【問題】ベクトルA=i+2j+3k  と同じ向きの単位ベクトルの成分を求めよ。

原点からx軸に1つ、y軸に2つ、z軸に3つ進んだベクトルがありますよね。
それの単位ベクトルってどう求めればいいんですか?
成分を求めるってどういうことなんでしょうか??
833名無し:02/05/18 00:12
>>832
ベクトルAの各成分をAと原点との距離で割れば良いよ。
>>829=831
Xって何?

>>832
>原点からx軸に1つ、y軸に2つ、z軸に3つ進んだベクトルがありますよね。
(1,0,0) ,(0,2,0), (0,0,3)のこと?

>それの単位ベクトルってどう求めればいいんですか?
長さをもとめて、0でなければそれの逆数倍する。

>成分を求めるってどういうことなんでしょうか??
1.かっこのなかみを求めること。
2.指定方向ベクトル(1次元部分空間)への射影を行うこと。
835132人目の素数さん:02/05/18 00:14
y=e^x に対して点(0、A)から引ける接線の本数は何本か?
836829:02/05/18 00:15
>>834
冒頭のX'の転置行列です。
837名無し:02/05/18 00:17
>>835
Aが正なら0本、0以下なら1本。
>>837
Aが1以下なら1本でないの?
>>837
はぁ?何をどう勘違いしてるやら・・・
840名無し:02/05/18 00:22
失礼、838の言う通りです、書き間違えました。
>837-840
ん?
A>1 で 0
A=1 で 1
0< A<1 で 2
A<=0 で 1
では?
842オラ来る:02/05/18 01:02
お教え下さってありがとうございます。

ベクトルAの大きさは√14ですよね。
それの逆数倍するっていうことは1/√14が単位ベクトルの大きさですか。

成分については
>1.かっこのなかみを求めること。
>2.指定方向ベクトル(1次元部分空間)への射影を行うこと。

う〜んよくわからないです。。。
単位ベクトルってのは、大きさが1のベクトルのことだ。

それから成分というのは、たとえば(3,4,5)のx成分は3、
y成分は4、z成分は5、ということ。
844オラ来る:02/05/18 01:27
>>843
ということはベクトルAの大きさが√14だから単位ベクトルにするには
1/√14をかければいいですね。
その通り。
んで、成分はどうなる?
846オラ来る:02/05/18 02:00
だから、、成分は(1i/√14+2j/√14+3k/√14)かなぁ?
バカなんで間違ってたらゴメンナサイ。
>>846
激しく違います。
848オラ来る:02/05/18 02:06
そしたらえっと(1/√14,2/√14,3/√14)
スミマセンバカなんで・・・。
849132人目の素数さん:02/05/18 02:06
袋の中に、白球2個、黒球3、赤球4、が入ってる。
この袋から3個のたを取り出すとき、
少なくとも1個は赤球を含んでいる確立を求めよ。
これお願いします。
>>846
うん、それは求めるベクトル本体の式だね。
成分はちょっと違う。

ベクトル ai+bj+ck を成分表示すると、(a,b,c)。
表記法の違いだよ。
>>848
をを、書いてるうちに答えが。
正解。
852名無しゲノムのクローンさん:02/05/18 02:09
>>849
白黒5個から3つ選ぶ選び方を全ての選び方から引け。
853132人目の素数さん :02/05/18 02:12
一辺の長さが1の正方形の一つの対角線上に中心を持つ円をこの正方形内に
重ならないように2つ描くことを考える。円の面積の和が最大になるように
2つの円を描いたとき、2つの円の半径はそれぞれいくらか。

という問題なのですが、解くために2つの半径をそれぞれR,rとおきますよね。
そのあとなぜ、R≧rのように大小関係を設定するという発想が生まれてくる
必然性は何ですか?
また、大小関係を設定することによりどういう利点がうまれるのですか?



854オラ来る:02/05/18 02:15
えっともう一問聞いてもいいですか?

問. ベクトルA、Bについて│A+B│=│A-B│ならばAとBは直交することを示せ。

AとBのベクトルを実際に書いて試してみて、直交するとき(ま四角)
しか成り立たないのは感覚的にわかるんですけど、どうやって答えに
していけばいいのか方法がわからないんです。
「ベクトルAを(Axi+Ayj+Azk)、ベクトルBを(Bxi+Byj+Bzk)と定義する」
なんてところから始めるんですか?

855名無しゲノムのクローンさん:02/05/18 02:16
>>853
必然性については、正方形、円の対称性に起因する。
利点については、場合分けしなくて済むことと、解いてないから確かじゃないが、
多分、最大値を求める上で必ず必要になるんだと思う。
>854
│A+B│=│A-B│
ならば
│A+B│^2=│A-B│^2
なので
(A+B)・(A+B)=(A-B)・(A-B)
>854
示したいのは
A・B = 0。
で、 | A+ B| ^2 = ????
を使うとどうなる?
858857:02/05/18 02:21
うが。
やられた。>856
859オラ来る:02/05/18 02:21
>>856-857さん
あっ二乗すればいいんだ・・・。
やってみます!
860名無しゲノムのクローンさん:02/05/18 02:22
>>854
簡単なのは両辺を2乗して内積0を用いるのが簡単な方法。
一々成分表示してるのはきっとそういう解きかたに慣れさせたいからじゃ?
 
ところで、ベクトルをi,j,k表示する時、普通はボールドイタリックでしょうから、
ここで書くときは素直にA↑=(Ax,Ay,Az)みたいな書き方の方が分かりやすいんじゃないかな。
861132人目の素数さん:02/05/18 02:22
>>853
レスども。で答えが42ぶんの37か、9分の8か、84分の73
になりました。
答え教えてもらえませんでしょうか?
862オラ来る:02/05/18 02:30
│A+B│=│A-B│なのでそれぞれを二乗して
(A+B)(A+B)=(A-B)(A-B)
A・A+2AB+B・B=A・A-2AB+B・B
4A・B=0
A・B=0
ベクトルAとBの内積が0の時それぞれのベクトルは直交する
終わり

こんな感じでいいですか?
863オラ来る:02/05/18 02:32
>>860さん
は〜い、わかりました!
ベクトルAをどうやって書いたらいいのかわからないんで困ってました。
これからはA↑って書きます。ありがとうございます。
864856:02/05/18 02:32
×の時
○なので

日本語はちゃんとしないと。
>862
うむ。
866132人目の素数さん:02/05/18 02:33
>>855
>必然性については、正方形、円の対称性に起因する。

なぜ対称な図形だと大小関係を設定するというようになるのですか?
ちょっとわかりません。
867名無しゲノムのクローンさん:02/05/18 02:35
>>866
図形を半分に折ることが出きるから、としか言えないなぁ。
868132人目の素数さん:02/05/18 02:38
変数xを含むある四面体の体積Vは=x√(84-x^2)となったのですが、
次に、このような四面体が存在するようなxの範囲を求めよという問題です。

解答ではx>0,(84-x^2)>0 となっているのですが納得できません。
なぜルートの前のxをルート内にいれてx^2(84-x)>0という条件じゃないのですか?
変数は一カ所にまとめた方がいいと習ったのですが。
>>866
大・小の並びと小・大の並びが対称だから二者は同一視してよい。
だから大・小の並びのみを考える事で両者を包含していることに。
870名無しゲノムのクローンさん:02/05/18 02:40
>>868
ハァ?
x<0のときどうすんの??
871132人目の素数さん:02/05/18 02:40
>>867
大小関係を設定するって対称性を崩す行為じゃないのですか?
一方の場合だけ考えればいいからですか?
872名無しゲノムのクローンさん:02/05/18 02:42
870の補足だけど、
>>868さん、
x√(84-x^2)=√{x^2・(84-x^2)}
とか思ってるんですか?
873名無しゲノムのクローンさん:02/05/18 02:42
>>871
そうです、多分貴方の理解で正解だと思いますよ。
874132人目の素数さん:02/05/18 02:45
>>866
解いてみればわかるけど、わざわざ大小関係を設定するほどでもないよ。
というか、必然性は感じなかったけど。
強いていえば、拘束条件と目的関数がr、Rの対称式になり、大小関係を
設定すると目的関数の定義域を小さくできるから、かな。
>>853
やってみたけど、大小関係を
設定する必要は特になかったよ。
876868:02/05/18 02:47
>>870
そうですね。困りました。どう考えればいいのでしょうか?
う〜んx>0というのは辺だと設定しているじてんで範囲が自動的に
つきますね、忘れていました。そこからなんですが、
そのあと本では,(84-x^2)>0 となっているのですが、
なぜx^2(84-x)>0ではないのでしょうか?

877名無しゲノムのクローンさん:02/05/18 02:51
>>876
xが辺で、正であることが自明ならば、x^2(84-x)>0でもいいと思うけど、
どうせ両辺をx^2>0で割ったら同じことだから、84-x>0と書いた方がスマートです。
878866:02/05/18 02:51
>>869
なるほど、そういうカラクリだったのですね。わかりやすかったです!

>>874>>875
Rの範囲はどうなるんですか?あと答えはどうなりましたか?
879868:02/05/18 02:55
>>872
>x√(84-x^2)=√{x^2・(84-x^2)}
とか思ってるんですか?

あっそうか、x>0がいりますね。でもそれは辺の情報で考えてあるから
大丈夫だと思うんですけど。

>>877
ありがとうございます。やっとわかりました。助かりました。
880875:02/05/18 02:58
げっ、まさか、デカい円の半径が1/2の時に
細かい議論をするために、大小関係の設定が必要なんじゃ・・・

その場合は両円が接する時が最大、
というのは自明だとして考えもしなかったけど。
881名無しゲノムのクローンさん:02/05/18 03:02
866の問題に関してなんですが、それぞれ2つの円に外接する正方形を書くと、
その正方形は両方とも一辺が1の正方形の内部に含まれるわけですから、
2円の面積の和が最大になるとき、これらの外接する正方形の面積の和も最大になりますよね。
だから、この問題の場合、わざわざ円に拘る必要って無いんじゃないかな?
なんか個人的には四角形の面積関係の方が考えやすい・・・
>>881
二円が互いに接している状態を考えるので円に拘ります。
883オラ来る:02/05/18 03:10
度々すみません。

A×(B×C)=B(A・C)−C(A・B)
この恒等式を証明せよ。

なんでこんなのが成り立つんですか?
考え方からわからないです・・・。
884名無しゲノムのクローンさん:02/05/18 03:12
>>882
ん?
外接正方形は一辺1の正方形を90度回転したものと考えてもらうと、その問題はクリアできないかなぁ…
885132人目の素数さん :02/05/18 03:12
>>881
>2円の面積の和が最大になるとき、これらの外接する正方形の面積の和も最大に
なりますよね。

ちょっと言っていることがわかりません。「正方形の面積の和」ってどういうことですか?
正方形は一つですよ。


886875:02/05/18 03:12
>>880
両円が接する時が最大、というのも真面目に考えていたけど、必然性はないよ。

>>881
>866の問題に関してなんですが、それぞれ2つの円に外接する正方形を書くと、
>その正方形は両方とも一辺が1の正方形の内部に含まれるわけですから、
>2円の面積の和が最大になるとき、これらの外接する正方形の面積の和も最大になりますよね。
これはウソだよ。
887886:02/05/18 03:14
あ、漏れ874だ(w
888名無しゲノムのクローンさん:02/05/18 03:15
すいません、>>881は忘れてください。
889132人目の素数さん:02/05/18 03:21
袋の中に、白球2個、黒球3、赤球4、が入ってる。
この袋から3個のたを取り出すとき、
少なくとも1個は赤球を含んでいる確立を求めよ。
これお願いします。
852 :名無しゲノムのクローンさん :02/05/18 02:09
>>849
白黒5個から3つ選ぶ選び方を全ての選び方から引け。

861 :132人目の素数さん :02/05/18 02:22
>>853
レスども。で答えが42ぶんの37か、9分の8か、84分の73
になりました。
答え教えてもらえませんでしょうか?

お願い致します




>>883
3次元ベクトルがからむ「かけ算」には3種類がある。

・実数倍: 実数(かける)ベクトル = ベクトル
・内積:  ベクトル(かける)ベクトル = 実数
・外積:  ベクトル(かける)ベクトル = ベクトル

この3つの(かける)記号をちゃんと区別して書くことが重要。
>>883では、どれが何なのかわからないよ。
891オラ来る:02/05/18 03:25
>>890さん
説明不足でごめんなさい。
A×(B×C)=B(A・C)−C(A・B)
というのは
Aクロス(BクロスC)=B(AドットC)−C(AドットB)

ドットは「・」でいいかなって思うんですけど、
クロスは「×」じゃダメですか。
>>891

>クロスは「×」じゃダメですか。
「×」は、ベクトルの分野で外積(⇔内積)を表すんですよ。>知ってますよね。
893132人目の素数さん:02/05/18 03:37
883で充分わかると思うんだが・・・
>>891
ごめん。ちゃんと区別されてたのね。
じゃあ 外積× 内積・ 実数倍 無印
ということでいこう。

でもって答えは・・・考察中。
成分に持ち込むしかないのかな?
>>892
>「×」は、ベクトルの分野で外積(⇔内積)を表すんですよ。>知ってますよね。
⇔は数学では同値であることを表すんですよ。>知ってますよね。
896132人目の素数さん:02/05/18 03:40
895の勝ちだな
>>894
>成分に持ち込むしかないのかな?
成分なしだと場合分けが面倒なんで、そっちのほうが簡単だと思う。
898オラ来る:02/05/18 03:45
いろいろ調べていたんですけど、これってスカラー3重積の定理
として当たり前に様に使われているんですね。
でもどうしてこうなるのか見つけられなかったです・・・。
>>891
この式って、どの方面の研究から出来た式?
純粋に基礎的な?ベクトルの方面?、それとも、電磁気?
900オラ来る:02/05/18 03:55
>>899さん
物理の基礎としての数学だそうです。
物理板に行った方がいいですか?
>>891
897に従いやってみたけど、やっぱ成分だな。
x成分だけに着目して等しいことを確かめ、
あとは「y、zも同様」で片づけちまえ。
>>900
いや、それはわからない。
物理の基礎の理解を、容易にするための『難解』な式かもしれないし。。。
または、ホントに出所が、物理関係のトコかも。
903オラ来る:02/05/18 04:06
http://www.toyama-mpu.ac.jp/la/physics/pp2-4.pdf
ここにスカラー3重積のことが書いてあるんですけど、さっぱりです。
>>903
うちのPCでは見られない。アドビは入っているのに。
横スレしてゴメン
>>853の問題って、答えは、1/(2√2)と(2-√2)/8であってる?
>>905
多分ちがうわ、それ。
907853:02/05/18 06:13
>>905
それ違います。問題では(1)(2)で誘導がついていたので
いきなり(3)から解くのは難しいかもしれませんね。
それは(3)の問題です。
908132人目の素数さん:02/05/18 06:59
理系の家庭教師が不足しています。
大学生なら誰でもOKです!登録お願いします。

http://page.freett.com/syuusyoku
909132人目の素数さん:02/05/18 07:10
p∧q→p=T(真)
となるそうなんですが、途中の展開がわからないのです。
よろしくお願いします。
910エルビス&プレスリー:02/05/18 07:31
(P∧Q)→P
¬(P∧Q)∨P (含意消去)
(¬P)∨(¬Q)∨P (ドモルガン)
真 ((¬P)∨Pがトートロジー)
911132人目の素数さん:02/05/18 07:39
トートロジーってなんですか?
912132人目の素数さん:02/05/18 07:48
>910
ドモルガンの後に、それぞれを括弧でくくってやれば順番を
いれかえられるのですね。ありがとうございます。
それと、ほかの導き方ってありますか?
913829:02/05/18 08:00
どなたか>>831をお願いできないでしょうか…
厚かましくてすみません…
914エルビス:02/05/18 08:13
トートロジーはPが真であれ,偽であれ,常に真になるってこと。
というか問題のレベル的に(¬P)∨P=真を使ってよいのか疑問...
915132人目の素数さん:02/05/18 08:30
「問題のレベル的に」ってどういう意味なのですか?
使わない様にしたほうがいいのでしょうか?
916132人目の素数さん:02/05/18 09:09
平行6面体は直方体をつぶした図形だと思うのですが、なぜこの図形が
「6面体」と呼ばれるのですか?ぜんぜん正6面体とリンクしないのですが、
正6面体をどうすれば平行6面体になるのですか?

ところで正6面体って2つの正4面体を対称に組み合わせた図形ですよね?
917132人目の素数さん:02/05/18 09:20
>916
正6面体は立方体(さいころ)ですけど?
918ツPツRツQミlヨ?ツ?ムfミヤツ?ツ?:02/05/18 09:26
正四面体を2つ合わせても、1つの頂点に3つの面が集まるところと、4つの面が
集まるところができて全然「正」ではありません。
919132人目の素数さん:02/05/18 09:27
学校で寝てばっかりいたのでぜんぜんわかりません。
次の関数を微分しなさい。
1.y=2x√(x^2+1)
2.y=x/√(1-x^2)
3.y=√(1-x)/√(1+x)
4.y=x^2 sin(x+1)
5.y=sinx cos^2(x)
6.y=sin√(x^2-x+1)
7.sin^4(x) cos4x
8.√(1+cos^2(x))
9.y=cosx/(1-sinx)
10.y=(tanx+(1/tanx))

全く授業きいていなかったもので
公式程度しか覚えておりませぬ。。。
920919:02/05/18 09:27
7,8も「y=」です
921132人目の素数さん:02/05/18 09:35
>>917
えっそうなのですか!?

>>918
>正四面体を2つ合わせても、1つの頂点に3つの面が集まるところと、4つの面が
集まるところができて全然「正」ではありません。

すいません、意味がよくわかりません。上下に組み合わせるんですよ。
上と下に対称な角ができると思うのですが。
922132人目の素数さん:02/05/18 09:35
正四面体と球は側面と3点で接しますよね。そのうちの2点と正四面体の
頂点を含む断面で切ると、円が3辺に接している3角形が得られると思う
のですが、これは間違いですか?
ちょうど底面での切り口が「底面の3角形の中点連結定理の線」と一致する
とおもうのですが・・・。
923テcPテcRテcQテ~lテネ?テc?テタfテ~テトテc?テc?:02/05/18 09:43
>921
2つの正四面体を張り合わせたところの頂点はどうなっていますか。
分からなかったら実際に作って見てください。
924921:02/05/18 09:44
すいません、正8面体の仕切り板の4角形が正三角形になっている図形は
なぜ正6面体ではないのですか?6面あると思うのですが。
>829
R=XQとでも置いて、

X=RQ^(-1)で計算。
926まおまお:02/05/18 09:57
>>913
恐らく、Qに意味はないのではないか(違ってたら、ごめん)。
要するに、Qが正則でありさえすれば、Xの代わりにXQを用いても
結果であるPは不変。だから、都合の良いQを取れば、Pの計算が
簡単になるってことでしょう。Xの階数は明らかに2なんだから、
まあ、以下略。
>>924
正多面体では
 ○各面が合同な正多角形
 ○各頂点に集まる辺の数が同じ
でなくてはならない。
君のいう立体では
 「3辺が集まる頂点」と「4辺が集まる頂点」
が存在するので正多面体ではない。

928132人目の素数さん:02/05/18 10:02
>924
正何面体というのはどこから見ても同じ形。面の形が同じだけではダメです。
上にも書きましたが、どこの頂点から見ても同じ形です。
あなたの言っている立体では、1つの頂点に集まる辺(稜線)の数や面の数が
頂点によって違ってしまいます。
三角すいぐらい簡単に紙で作れるから、作って張り合わせてごらん。
929まおまお:02/05/18 10:03
ケコーン
930132人目の素数さん:02/05/18 10:05
コンドームの避妊率が99%であると過程して、
セクースの試行回数をxとおくと、任意のxにおける避妊率yは
 y=(99/100)^x とおける。
今ここでx=100(平均的なカップルにおける一年間のセクース実施回数)とすると
 y=(99/100)^100≒36.6%
また、x=200 であるときは、同様に考えて y=13.3%である。
 ∴コンドームでの避妊=(゚д゚)マズー

しかし、同様の条件のもとで、コンドームを使ったセクースでの妊娠率を1%と考えると、
セクースの試行回数をxとして、任意のxにおける妊娠率zは
 z=(1/100)^x となり、ここでx=100を代入すると
 z=(1/100)^100=1/1E+200
これはxが大きくなるに従って限りなく0に近似してゆく。
 ∴コンドームでの避妊=(゚д゚)ウマー


同一条件のもとで行われた施行に対する結果が矛盾してしまいます。
どうしてでしょうか?
あなたの求めた確率はそれぞれ、100回連続ヒットしない確率と
100回連続ヒットする確率です。
別に矛盾はしていません
932132人目の素数さん:02/05/18 10:17
アハーン?

つまり、結局のところコンドームでの避妊はウマー(゚д゚)なのかマズー(゚д゚)なのか?
マズーの顔は(+д+)だべ。
>>z=(1/100)^x, x=100のほう
おめーは100つ子をはらませるってのかYO!
935うきゃ@1年ぶり:02/05/18 12:04
>919
朝はネムー
教科書で「積の微分」「章の微分」「合成関数の微分」を探しましょう.
合成関数はちとむずいのでヒントを.公式見ながら考えてね
y=sin3x だったら,sinxのxに3xを代入した形と見て
f(x)=sinx,g(x)=3xとおくと
y'=(f(g(x)))'=f'(g(x))・g'(x)=cos3x・3=3cos3x
936うきゃ@1年ぶり:02/05/18 12:06
>889
まだ見てるかな・・・
答えが3つもあるってことは,まだはっきり分かってない証拠ですね
これが答えだってばーんと書き出してみて
937うきゃ@1年ぶり:02/05/18 12:09
>919
念のため言っておきます
たぶん授業中ねてた奴の相手なんて誰もしてくれないと思う
僕も次からは相手しないよー
938プレスリー:02/05/18 13:53
>>915に亀レス
公理と言うか,使っていい法則にP∧¬Pも入ってたか謎ということです.
もっといえば
P∧Q→P,P∧Q→Q(and除去)
っていうのも使っていいならこの問題は瞬殺なんで。
まぁ普通は
含意消去→ドモルガンで選言化→トートロジー
ってのが基本です。
P∧Q→P=Tっていうのがどういう場面ででてきた問題なのかが問題です.
939132人目の素数さん:02/05/18 14:24
a(n)がαに収束する⇔αはa(n)のただ1つの集積点であることを示せ。

ってのが分かりません。ただひとつってのはどうやって証明するんですか?

集積点の定義
∀ε>0に対し、#{n∈N|a(n)∈(x−ε,x+ε)}=∞
940132人目の素数さん:02/05/18 14:28
2つあったとして...
941ベホマズン ◆DEENepXc :02/05/18 14:54
a(n+2) = a(n) + 2^(n-1)
a(1) = 1
a(2) = 0
という数列の一般項が、
a(n) = 2/3{2^(n-2)+(-1)^(n-1)}
となる事を示せ

という問題がわかりません〜〜〜ToT
前もこのようなやつをここで聞いたんですが、いまだに・・・
一般項まで辿り着きません。

教えて下さい!
>>941
a(n+2)=a(n)+2^(n-1) ⇔ a(n+2)/2^(n-1)=1/4*a(n)/2^(n-3)+1

b(n)=a(n)/2^(n-3) と置き換えれば、
b(n+2)=1/4*b(n)+1

さらに、c(n)=b(n)-4/3 と置き換えれば、
c(n+2)=1/4*c(n)

あとは、n が偶数・奇数で場合分けして考えるか、
もしくは、x(n+2)+a*x(n+1)+b*x(n)=0 というタイプの
漸化式の a=0, b=-1/4 のケースと考えて解けばいいよ。
>>941-942
2のベキで割る前に、両辺からa(n+1)を引いておいた方がいいよ。
計算がやや楽になる。
944132人目の素数さん:02/05/18 15:35
新スレきぼん>132人目のともよちゃん
b(n)=2/3{2^(n-2)+(-1)^(n-1)}とおくと
b(1)=2/3{2^(-1)+(-1)^0}=2/3{1/2+1}=2/3*3/2=1
b(2)=2/3{2^0+(-1)^1}=2/3{1-1}=0
b(n+2)=2/3({2^n+(-1)^(n+1)}
b(n+2)-b(n)=2/3{2^n-2^(n-2)}(∵(-1)^(n+1)=(-1)^(n-1))
=2/3*2^(n-2){2^2-1}=2^(n-1)
答えとしてはこれでOK
>>945
それは、「予想して帰納法で証明」というやつだな。
問題なのは答えが予想できるかどうかだ・・・・
947ベホマズン ◆DEENepXc :02/05/18 16:11
解き方は、なんとなく分かるのですが、
最後の一般項まで導けないんです・・・
>>947
問題に既に一般解が出てるんじゃないんですか?
949ベホマズン ◆DEENepXc :02/05/18 16:16
その一般解まで導くのが問題なんです・・・
つまり行程が回答ということですね・・・

だれか、導けた方いらっしゃいますか?
950132人目の素数さん:02/05/18 16:21
>>949もっと簡単なのから始めた方が・・・
>>949
ちょっと待ってね
  a(n+2) = a(n) + 2^(n-1)
→ a(n+2)+a(n+1) =a(n+1)+ a(n) + 2^(n-1)
→ {a(n+2)+a(n+1)}/2^(n+2) =(1/2){a(n+1)+ a(n)}/2^(n+1) +1
→ {a(n+2)+a(n+1)}/2^(n+2)-1 =(1/2)[{a(n+1)+ a(n)}/2^(n+1)-1]

これで単純な等比数列になってると思います。

  a(n+2) = a(n) + 2^(n-1)
→ a(n+2)-a(n+1) =-a(n+1)+ a(n) + 2^(n-1)

でも同様に式変形し、2つの式から導けると思いますよ。
>>952
  a(n+2) = a(n) + 2^(n-1)
→ {a(n+2)+a(n+1)}/2^(n+2)-2 =(1/2)[{a(n+1)+ a(n)}/2^(n+1)-2]

の間違いかも。
検算はご自身で御願いします。
>>952
よく見ると式変形えらく間違っていました。
まあ、方針だけわかれば解けると思うので
めんどくさいしなおしません。
>>941 (方法その1) 高校生程度だと、下記のようにする。
n=2k-1 (奇数) と n=2k に分けて漸化式を整理すれば、
階数が一つ減り、一般項は等比級数の和として容易に求まる。
結局、

a(n)=(2/3)(2^(n-2)+2) (nが奇数の場合)
a(n)=(2/3)(2^(n-2)-2) (nが偶数の場合)

となる。この±を (-1)^n で書き直せば終り。
(あるいは奇数項に (1-(-1)^n/2をかけ、偶数項に
(1+(-1)^n/2をかけ、和をとると考えてもよい)
>>941 (方法その2) 大学初年級 = Knuthの本にあった方法
母関数を使う。zの関数で、

F(z)=a(0)+a(1)z+a(2)z^2+…

となっているものを考える。

F(z)-z^2 F(z)を考えると、z^2項から先が漸化式により
級数和となり、簡略化でき、F(z)が求まる。

F(z) = (1/2-(3/2)z)/((z+1)(2z-1)) ... (A)

ただし、ここで初項の値として a(1)=1のほかに a(0)=-1/2
を使った。このような問題の添字はゼロからはじめて
おいたほうが楽だよ。

で、Aを部分分数展開すると、

(A) = (-2/3)/(1+z) + (1/6)/(1-2z) ...(B)

1/(1+z) = 1-z+z^2-z^3+... = Σ(-1)z^k
1/(1-2z)=1+2z+(2z)^2+... = Σ2^k z^k

だから、(B)を整理すれば、

(A) = Σ(2^k/6-(2/3)(-1)^k)z^k

このz^k の係数が、求める一般項。
957132人目の素数さん:02/05/18 17:59
>>938
出所は、問題集にこのまま載っていたのです。展開しなさいと。
〜P∧P=偽
はこの問題集の公式のところにも一応かかれています。基本的なとき方(誰かに
つっこまれても大丈夫な)は>>910 の形となるのでしょうか?
958祭り:02/05/18 19:36
2ちゃんねらーの力でオールスター選手を誕生させましょう!
“不正投票無し”の人海戦術で2ちゃんねるの底力を見せましょう!地道に『1日5票、携帯も合わせて10票』で投票願います。
『2002サンヨーオールスターゲーム』
http://allstar.sanyo.co.jp/
2ちゃんねる党推薦候補は、
広島東洋カープ・キャッチャー・背番号63・鈴衛祐規さん

http://sports.2ch.net/test/read.cgi/base/1021303023/l50
959132人目の素数さん:02/05/18 19:37
nが奇数の時
a(n+2)-a(n)=2^(n-1)
a(n)-a(n-2)=2^(n-3)
..............
a(3)-a(1)=1

全部足すとa(n+2)-a(1)=2^(n-1)+2^(n-3)+....+2^0
右辺=2^0+2^2+2^4+...+2^(n-1):始項1で項比4の等比級数の和
1-4^{(n-1)/2+1}
-------------={2^(n+1)-1}/3
1-4

a(1)=1よりa(n+2)=1+{2^(n+1)-1}/3
n=m-2とおくとa(m)=1+{2^(m-1)-1}/3={3+2^(m-1)-1}/3=2/3*{1+2^(m-2)}(mは奇数)
で奇数の場合は出来た。偶数の場合はどうなるかにゃ?
960132番目のメルセンヌ数さん:02/05/18 20:47
移転完了。
話題が終了次第、
下記のURLへ逝って下さい。
http://science.2ch.net/test/read.cgi/math/1021721809/
961132人目の素数さん:02/05/19 16:15
>>957
教科書がそっちならドモルガンを逆に使って,
(P∧Q)→P
⇔¬(P∧Q)∨P (含意消去)
⇔¬(P∧Q∧¬P)(ドモルガン)
⇔¬(偽∧Q)
⇔¬偽
⇔真
みたいなのがいいかな.
偽∧Q⇔偽
が教科書に載ってるかは保証しないけど。
962961:02/05/19 16:21
ageてしまった...スマソ
おつかれさまです>>960
963829:02/05/20 00:13
かなり遅レスだけど、
>>925,>>926感謝。
これからやってみます。
964132人目の素数さん:02/05/20 06:35
>>961
ありがとうございます。
ちょっとレベルがあがってしまっているのですが。
もう一段階レベルを落としていただけないでしょうか?
965>>961:02/05/20 21:55
>>964
レベルあがってる??
>>910のドモルガンを逆に使ってるだけだよ。
むしろ教科書に載ってた前提の式を使ってるから
こっちのほうがどうみても基本的だと思うけど。
そろそろ君の中での使っていい式を並べてみてくれないかい?w
僕の中では
1.(P⇒Q)⇔¬P∨Q(含意除去)
2.(¬(P∧Q))⇔(¬P∨¬Q),(¬(P∨Q))⇔(¬P∧¬Q)(ドモルガン)
3.¬¬P⇔P(二重否定)
4.(P∨¬P)⇔真(トートロジー)
5.P∧偽⇔偽,P∧真⇔P,P∨真⇔真,P∨偽⇔P
これくらいだよ。
966132人目の素数さん:02/05/22 08:54
>>965
どうもありがとうございます。
いちおう、上に書かれているものは僕の本にも載っています。
その時々によって、使ってはいけないものなどもあるのでしょうか?
>>966
全部あげろと言われたら、例えば、
http://ysserve.cs.shinshu-u.ac.jp/Lecture/SymbolLogic/node18.html
とかね。(ここの教材の話ではだけど)

命題論理の本やってるんじゃないのかな...
そういう分野の本じゃなかったら公理系の書いてる節参照。
人によって違うし。

命題論理をやりたいのであれば意味論の項とか読んで自分で勉強してください
(とりあえずおさえておかないといけないので)
968132人目の素数さん:02/05/22 14:29
横レスだけど、論理式のシンタクスを扱うなら、
ゲンツェンのLKかNKが一番スッキリしてて
わかりやすいと思う。
969968:02/05/22 14:31
スマソ上げちまった
970教えてください!:02/05/22 22:28
今誰かいますか?
中学生から高校生くらいのレベルの問題なのですが、数学からしばらく遠ざか
っていたため、今日の授業にサッパリついていけませんでした。
数学の先生のように理解できるように教えてくれる人はいますか?

 問題 A剤の30%製剤をシロップに溶いて、15r/sの薬用量で体重5s
    の猫に1日3回5日分のシロップ剤を調製する場合、A剤の30%製
    剤は総量で何g必要か?
    また、これを溶解して100mlと150mlとした場合、それぞれ1回
    につき何mlずつ与える必要があるか?

30%製剤総量) 3回×5日×15r×5s=1125r
         1125r×100/30=3750r=3.75g

までは解かるんですが、その後の溶解が理解できません。
板違いだったらスマソ。でも数学だからここで教わりたいのです。
よろしくお願いします。                  
>>970
15回分で100mlなんだから、
1回分は 100/15 ml でいいんでないの?
972教えてください!:02/05/22 23:12
答えは100ml溶解 6.7ml
   150ml溶解 10ml ずつ投与。なんです。

どうしてそうなるのかわかりません。 
いや、だからさ、971を読んでくれよ。
974教えてください!:02/05/22 23:19
いや、理屈ではそうなるのですが、授業でやっていた途中の計算式が
理解できないんです。
1mlの含有量を出して、比例式で求めてるんです。
なんでそんなことするんですかね?
975132人目の素数さん:02/05/22 23:21
合同が同値関係であることを示したいのですが
どうすればいいでしょうか・・
976132人目の素数さん:02/05/22 23:26
反射律・対称律・推移律が成り立つことをしめせ。
>>975
合同って、平面図形のか?

図形の、「回転 o 裏返し o 平行移動」 が結合則を満たし、かつ
それぞれに逆変換が存在することを言えばいいんじゃないかと思う。
978132人目の素数さん:02/05/22 23:33
不特定多数の点があって、その最短ルートってどうもとめる?


979977:02/05/22 23:33
追記: そうすれば、>>976が示せる。
>>978
虱潰
981980:02/05/22 23:36
nanteyomun?
>981
Σιραμι τσβσι
983982:02/05/22 23:44
そういうアルゴリズムがあるの?
984132人目の素数さん:02/05/22 23:47
>>977
ありがとう!!!
985132人目の素数さん:02/05/22 23:55
lim[n→∞]3^n/n!の極限をはさみうちの定理を使って説く場合、何と比べればいいんですかね?
986次スレ誘導:02/05/23 00:14
◆ わからない問題はここに書いてね 32 ◆
http://science.2ch.net/test/read.cgi/math/1021721809/
987132人目の素数さん:02/05/23 00:14

ちょっと質問のポイントがずれてるが...

n≧3 のとき n!≧1・2・3^(n-3)・n 位でどうぢゃ?
988埋め立て開始:02/05/23 01:33
(・∀・)ウメー
989132人目の素数さん:02/05/23 01:34
無限級数lim_[n→∞]nΣ_[k=1,n]1/k^2の求め方が解りません。収束するらしいのですが…宜しくお願いします。
990:02/05/23 01:36
>>989
>>986に書いてあるスレへ Let's GO!
991メルセンヌ家の素数さん:02/05/23 01:41
>>981
蚤潰しw
とっとと埋めようぜ!
(・∀・)ウメー
994梅(・∀・)ウメー:02/05/23 01:48
α
995虱潰し:02/05/23 01:50
メール欄
996β:02/05/23 01:54
梅(・∀・)ウメー
β
997γ:02/05/23 01:55
(・∀・)ウメー
998ε-δ論法:02/05/23 01:57
最初はナニ言ってるかサパーリ分からなかったYO
(・∀・)ウメー
999しらみ:02/05/23 01:58
1000!!!!!!!
◆ わからない問題はここに書いてね 32 ◆
http://science.2ch.net/test/read.cgi/math/1021721809/
10011001
このスレッドは1000を超えました。
もう書けないので、新しいスレッドを立ててくださいです。。。